Uncommon Descent Serving The Intelligent Design Community

The Big Bang, The First Cause, and God

Share
Facebook
Twitter
LinkedIn
Flipboard
Print
Email

Over on a recent thread there has been much interesting discussion about a recent debate between theist philosopher Rabbi Daniel Rowe and atheist philosopher A.C. Grayling.  HeKS provided a review of the matter, focusing largely on his analysis of Jerry Coyne’s responses.

I agree with HeKS’s general observation that Coyne failed to adequately address the issues.  Indeed, it seems Coyne failed to adequately understand some of the issues, a situation that is all too common.

However, I want to focus in this post on a specific aspect of the discussion, namely, some of the points raised by sean samis, starting @37 on that thread.  In his comments, samis urges caution in drawing any conclusion from the Big Bang about deity’s existence or involvement.  I do not necessarily share all of his conclusions, but I think a number of his points are worthy of additional discussion.

First of all, let me apologize to HeKS for starting a new thread.  I initially began this as a comment to the prior thread, but it became long enough that it required a separate post.  Additionally, I want to focus on a specific issue that tacks in a slightly different direction than the prior thread.

If the Universe Had a Beginning, then What?

samis begins by addressing the question of the universe being created ex nihilo:

The proper response to the creation ex nihilo argument is that science does not believe or claim that our universe was created ex nihilo. The argument is a red herring.

This is an important point, and one on which the Big Bang arguments for God seem to flounder.  The fact that the universe had a beginning (and we should note here for accuracy’s sake that this is not a “fact” in an observational sense, but an inference), does not mean that whatever caused the universe had to be the First Cause or had to be God, in any sense of that word.  That the universe had a beginning just means that something caused the universe.  Nothing more; nothing less.

We can, indeed we must, approach claims of a multiverse or cosmic bubbles or some other universe-generating natural phenomenon with extreme skepticism.  There are many problems with such ideas, which have been well detailed previously in these pages.  But it simply does not follow that because the universe had a beginning that it must have been caused by the First Cause or that the First Cause has to be God.

Rather, what can be said is that: (a) no-one has any real observational evidence as to the cause of the universe; and (b) it is possible that the cause of the universe was the First Cause.  In addition, we might add that (c) it is possible that the First Cause had a plan, a purpose, an intent, a desire, a design – attributes similar to what we see ourselves possessing as rational, intelligent, individual, creative beings.

The foregoing is a more modest claim.  It is a reasonable claim, a supportable claim, a claim that is not at all challenged by the silly responses of the likes of Coyne & Co.  It is certainly as good of a claim – probably better from most rational points of view – than the contorted naturalistic explanations we are often treated to.

Yet we must acknowledge that it is still a claim based more on likelihood and inference, than on certainty and deduction.

samis later remarks:

That [the First Cause is spaceless, timeless and immaterial] does not follow unless we are careful to specify that whatever space, time, or material this “non-extensional something” might be composed of, it is not the space, time, or material which is part of our universe.

In other words, this “non-extensional something” can (and probably does) occupy space, experience time, and is composed of some material, but it is not of the space, time, or material of our universe.

Also a point worth considering.  Again, that the universe had a cause does not mean that the universe is all that there is or that the cause has no attributes similar to the attributes of our universe.  It is probably fair to say – definitionally so – that the cause of the universe exists outside the universe, but that does not speak directly to other attributes of that cause.

samis continues:

Much less is it given that this First Cause have attributes of intelligence (mind, intention, goals, wants, relationships, affection, etc.). Absent these this First Cause would not be any deity but a mere “thing” or “things”.

This is true up to a point.  Most of the attributes projected onto the First Cause flow not from any logical requirement of the First Cause itself, but from our personal beliefs and preferences about what we think that First Cause is, or should be.  That is well enough as a philosophical or religious matter, but it is not sustainable as a logical, scientific or deductive matter.

That said, there are some hints of purpose and goal-oriented activity and planning that strike any thoughtful observer of the cosmos.  Although not rising to the level of logical deduction, such hints certainly provide reasonable grounds to infer that the cause of the universe has certain attributes.

—–

How Far Can We Go?

It seems that with regard to the observable universe we have, at most, the following situation:

  1. An inference, from observable facts, that the universe had a beginning.
  2. A deduction that the universe had a cause.
  3. A deduction that the cause was not within the universe itself (i.e., existed outside of the universe, both spatially and temporally).
  4. An inference, from observable facts, that the universe has been finely tuned.
  5. A deduction that the cause was capable of producing the universe and of finely tuning the constants.

Most everyone is in agreement up to this point.  One additional item that everyone should agree on is the following:

  1. Ultimately, when traced back, there must be a First Cause – that which existed in and of itself, without a beginning.

It is true that whether the universe was caused by the First Cause or by some intermediate cause is entirely open to question.  However, at some point, we must regress to a First Cause.  We trust everyone is in agreement with this concept of a First Cause.

Identifying the First Cause, unfortunately, is a trickier matter.

The Nature of the First Cause

A number of proposals might be put forward, but let us focus on the two most common.

One proposal on the table is that the First Cause was a purely naturalistic phenomenon: some unidentified, never-before-seen, essentially indescribable, powerful phenomenon, that coincidentally (through sheer luck or sheer repetition over time) managed to produce the finely-tuned universe in which we find ourselves.

A second proposal on the table is that the First Cause is God.  The materialist will quickly argue that God is likewise unidentified, never-before-seen, and essentially indescribable.  Even if we grant this for purposes of discussion, this argument does not serve to strengthen the materialistic claim of a naturalistic First Cause, but only serves to put the God proposal on at least the same footing.

Yet they are not quite on the same footing.

We would be remiss if we did not acknowledge that many individuals have claimed (often at great risk to their reputation and physical safety) to have had a personal encounter with God and have tried, with varying degrees of completeness, to describe God.  This holds both for the rare visual experiences, as well as the less-concrete but far more common emotional or spiritual experiences.  The materialist may well argue that these individual accounts are disparate, unverified in some cases, and open to challenge.  That may well be true.  But the fact remains that there is some evidence, independent of the observations of the cosmos itself, of God’s existence, however scattered and fragmentary it may be.  It may not be much.  But it is more than can be said for the naturalistic proposal.

Furthermore, there is an additional aspect of the cosmos that even ardent materialists acknowledge demands an explanation: that of the finely-tuned constants and the apparent purposeful way in which everything works together to make our very existence possible  The universe, to put it bluntly and to borrow a phrase from Richard Dawkins uttered in the biological context, gives “the appearance of having been designed for a purpose.”

Now it may be that the materialist is right, that this apparent design is an illusion, that the existence of our universe is the result of a cosmic – or, shall we say, “extra-cosmic” – lottery.  That is one potential explanation, as a matter of sheer logical possibility.  But it is lacking in evidence, provides absolutely no intellectual comfort, and is certainly nothing to hang our hat on.

The concept of God at least has the benefit of positing a First Cause with the ability to make the purpose real, to fine tune for a purpose, to have a plan and a goal and an intended outcome; in other words, a First Cause that helps explain the apparent design in the universe, not one that tries to explain it away.

Finally, it is noteworthy – not definitive in any sense of the word, mind you, but noteworthy – that some of the very attributes attributed to God over the ages (tremendous power, vast intelligence, setting a plan in place, showing a personal interest in human affairs), have gained support centuries later in scientific discoveries.  If not at the level of deduction, then at least at the level of reasonable inference.

—–

Conclusion

So what are we left with?

The inference that the universe had a beginning does not allow us to identify the First Cause.  We cannot say, it seems to this author, as a matter of logic and deduction that the First Cause is God.  We cannot even say that the universe was caused by the First Cause, rather than some intermediate cause.  Indeed, as a matter of dispassionate objective scientific inquiry and reasoning, we can say but very little about the First Cause.

In that sense, the claim that the First Cause is God must be viewed with some caution.  But it must not be viewed with derision.  Rather, it should be seriously viewed as a live possibility, very much worthy of consideration.

Indeed, when compared against the materialistic claim, the proposal that the First Cause is God is eminently reasonable – being more consonant with the evidence, with our experience, and with the reasonable inferences that can be drawn from scientific inquiry.  While recognizing a significant lack of direct observational evidence on either side of the debate, the objective observer must at least consider the existence of God as a live possibility and, when weighed against the alternative, as the more rational and supportable possibility.

In the final analysis, the individual who holds to the idea that the First Cause is God should not go a bridge too far by attempting to shoehorn the observed attributes of our universe into a definitive, deductive claim for God’s existence.  Yet neither should he feel threatened by the materialistic claim, even more lacking as it is in evidence.  In the face of the materialistic mindset that so often rules the day, he can approach the debate with a healthy dose of humility, recognizing that his claim of God’s existence is based on inference (and hopefully personal experience), while at the same time feeling confidently grounded in the comparative strength of his position and feeling no need to apologize for the same.

Comments
I will not respond to your posts any further since it is obvious you place your own interpretation of scripture far above any empirical, or even scriptural, refutation I could bring forth. bornagain77
BA77 # 274: “mw you stated somewhere in your posts: ‘Who placed you judge over the Word of God at Sinai?’” You respond, “And you yourself are immune from this criticism how exactly?” __________________________________________________________________ I could say by exactly believing in the Ten Commandments. @ 270, I said, “Who placed you judge over the Word of God at Sinai? What gives you a clear scriptural right to say I do violence to the strict Word, the unalterable Word?” Surely, God is immune from criticism in His divine law, otherwise He is untrue and unreliable. Not the best way to inculcate faith you must agree? And we all see through a dark glass dimly (1 Cor 13:12) do we not? Still, divine light we have set in stone. You say, “As well, it is obvious, at least to me, that you have doctrinally placed belief in YEC on a level that is almost as necessary as belief in Christ himself for salvation.” As for the level of divine law in the Ten Commandments they are on a level with Christ, for in terms of the Holy Trinity He spoke them with Yahweh. Relative to Catholics; ref the Catechism of the Catholic Church and the Ten Commandments: 2072 “Since they express man's fundamental duties towards God and towards his neighbour, the Ten Commandments reveal, in their primordial content, grave obligations. They are fundamentally immutable, and they oblige always and everywhere. No one can dispense from them. The Ten Commandments are engraved by God in the human heart.” 2081 “The Ten Commandments, in their fundamental content, state grave obligations. However, obedience to these precepts also implies obligations in matter which is, in itself, light.” http://www.vatican.va/archive/ccc_css/archive/catechism/p3s2.htm Today, there are very few Catholics who take divine law as “fundamentally immutable.” Mutated is six-day creation from the Big Bang Theory to monkeys and back. You imply you can refute my arguments scripturally, but I will not listen; not the case, but then you close down. Yet, in my opinion, you still have not demonstrated which scripture trumps an unalterable divine law and commandment, all ten of which Jesus fulfilled to the “letter” and “stroke of a letter” (Matt 5:17-19); saying “heaven and earth” would pass away before the law of Sinai, when it will be no longer needed in heaven, as there will be only one Will; God’s. Still, I have had more than my share of comments at UD in this OP, of which I am very grateful and wish you well BA77 for your many valid and respected contributions throughout UD. I just thought these last comments would wrap it up for me. mw
StephenB @210: Apologies for the late response.
Michael Augros, in his book “Who designed the designer,” makes my point using different words. Perhaps it will resonate with you. “Two first causes would have to share a common nature."
This claim follows from what, exactly?
They could thus be distinguished only by some addition to that nature in at least one of them.
Whether or not they can be distinguished from each other, meaning whether they are exactly alike or not, is quite a separate question from whether they both exist.
Their common nature exists with the distinctive feature in the one case and not in the other. The common nature is, therefore, in itself indifferent to the add-on and hence enters into a combination with it only through a prior cause, a combiner.
Sorry, but this does not resonate with me. What I can discern, unfortunately, is that he is playing word games here. The question is whether an uncaused cause can exist. If it does, then it will inevitably have some characteristics – however we want to define them. It simply does not follow that because it has a particular characteristic, that the characteristic must have come about through a prior cause. The only reason his example appears to make sense, at first blush, is because he has led us down the path of starting with characteristic X and then (conveniently, through some unknown cause) adding characteristic Y. Then he proclaims that because characteristic Y was "added" to what already existed, that there must have been a “cause” for characteristic Y that did the "adding." But it simply doesn’t follow in the context of the discussion. There is no logical reason why we can’t have an uncaused cause with characteristic X and an uncaused cause with characteristic Y (or with both characteristics X and Y, for that matter). There is no principle of reason that requires characteristic Y to have been caused by some “combiner” while accepting that characteristic X could exist independent of a prior cause. His argument is based on a semantic game and definitely does not resolve the question of whether there could be more than one uncaused cause. Eric Anderson
mw you stated somewhere in your posts: "Who placed you judge over the Word of God at Sinai?" And you yourself are immune from this criticism how exactly? mw, I will not respond to your posts any further since it is obvious you place your own interpretation of scripture far above any empirical, or even scriptural, refutation I could bring forth. As well, it is obvious, at least to me, that you have doctrinally placed belief in YEC on a level that is almost as necessary as belief in Christ himself for salvation. bornagain77
Violence to Scripture, Hell and the Jolly Roger: (Part 4 of 4) Below is an extensive quote from scripture, I am sorry to labour the point, but I think needs must; it highlights that God means business to those who deflect His law. They were literal threats, for literal commands, otherwise, He is injustice itself. If God exaggerates, and six days means six long ages with as many days as befits the Big Bang Theory; then clearly, He cannot claim truth, accuracy, purity of word, or that He as an exact holy map to follow the Way! Big Bang Theory is better, as it saves the word of God from ridicule: or makes atheists out of us. ________________________________________________________ “But if you will not obey the LORD your God by diligently observing all his commandments and decrees, which I am commanding you today, then all these curses shall come upon you and overtake you: Cursed shall you be in the city, and cursed shall you be in the field. Cursed shall be your basket and your kneading-bowl. Cursed shall be the fruit of your womb, the fruit of your ground, the increase of your cattle, and the issue of your flock. Cursed shall you be when you come in, and cursed shall you be when you go out. The LORD will send upon you disaster, panic, and frustration in everything you attempt to do, until you are destroyed and perish quickly, on account of the evil of your deeds, because you have forsaken me. The LORD will make the pestilence cling to you until it has consumed you from the land that you are entering to possess. The LORD will afflict you with consumption, fever, inflammation, with fiery heat and drought, and with blight and mildew; they shall pursue you until you perish. The sky over your head shall be bronze, and the earth under you iron. The LORD will change the rain of your land into powder, and only dust shall come down upon you from the sky until you are destroyed. The LORD will cause you to be defeated before your enemies; you shall go out against them one way and flee before them seven ways. You shall become an object of horror to all the kingdoms of the earth. Your corpses shall be food for every bird of the air and animal of the earth, and there shall be no one to frighten them away. The LORD will afflict you with the boils of Egypt, with ulcers, scurvy, and itch, of which you cannot be healed. The LORD will afflict you with madness, blindness, and confusion of mind; you shall grope about at noon as blind people grope in darkness, but you shall be unable to find your way; and you shall be continually abused and robbed, without anyone to help. You shall become engaged to a woman, but another man shall lie with her. You shall build a house, but not live in it. You shall plant a vineyard, but not enjoy its fruit. Your ox shall be butchered before your eyes, but you shall not eat of it. Your donkey shall be stolen in front of you, and shall not be restored to you. Your sheep shall be given to your enemies, without anyone to help you. Your sons and daughters shall be given to another people, while you look on; you will strain your eyes looking for them all day but be powerless to do anything. A people whom you do not know shall eat up the fruit of your ground and of all your labours; you shall be continually abused and crushed, and driven mad by the sight that your eyes shall see. The LORD will strike you on the knees and on the legs with grievous boils of which you cannot be healed, from the sole of your foot to the crown of your head. The LORD will bring you, and the king whom you set over you, to a nation that neither you nor your ancestors have known, where you shall serve other gods, of wood and stone. You shall become an object of horror, a proverb, and a byword among all the peoples where the LORD will lead you. You shall carry much seed into the field but shall gather little in, for the locust shall consume it. You shall plant vineyards and dress them, but you shall neither drink the wine nor gather the grapes, for the worm shall eat them. You shall have olive trees throughout all your territory, but you shall not anoint yourself with the oil, for your olives shall drop off. You shall have sons and daughters, but they shall not remain yours, for they shall go into captivity. All your trees and the fruit of your ground the cicada shall take over. Aliens residing among you shall ascend above you higher and higher, while you shall descend lower and lower. They shall lend to you but you shall not lend to them; they shall be the head and you shall be the tail All these curses shall come upon you, pursuing and overtaking you until you are destroyed, because you did not obey the LORD your God, by observing the commandments and the decrees that he commanded you. They shall be among you and your descendants as a sign and a portent for ever. Because you did not serve the LORD your God joyfully and with gladness of heart for the abundance of everything, therefore you shall serve your enemies whom the LORD will send against you, in hunger and thirst, in nakedness and lack of everything. He will put an iron yoke on your neck until he has destroyed you. The LORD will bring a nation from far away, from the end of the earth, to swoop down on you like an eagle, a nation whose language you do not understand, a grim-faced nation showing no respect to the old or favour to the young. It shall consume the fruit of your livestock and the fruit of your ground until you are destroyed, leaving you neither grain, wine, and oil, nor the increase of your cattle and the issue of your flock, until it has made you perish. It shall besiege you in all your towns until your high and fortified walls, in which you trusted, come down throughout your land; it shall besiege you in all your towns throughout the land that the LORD your God has given you. In the desperate straits to which the enemy siege reduces you, you will eat the fruit of your womb, the flesh of your own sons and daughters whom the LORD your God has given you. Even the most refined and gentle of men among you will begrudge food to his own brother, to the wife whom he embraces, and to the last of his remaining children, giving to none of them any of the flesh of his children whom he is eating, because nothing else remains to him, in the desperate straits to which the enemy siege will reduce you in all your towns. She who is the most refined and gentle among you, so gentle and refined that she does not venture to set the sole of her foot on the ground, will begrudge food to the husband whom she embraces, to her own son, and to her own daughter, begrudging even the afterbirth that comes out from between her thighs, and the children that she bears, because she is eating them in secret for lack of anything else, in the desperate straits to which the enemy siege will reduce you in your towns. If you do not diligently observe all the words of this law that are written in this book, fearing this glorious and awesome name, the LORD your God, then the LORD will overwhelm both you and your offspring with severe and lasting afflictions and grievous and lasting maladies. He will bring back upon you all the diseases of Egypt, of which you were in dread, and they shall cling to you. Every other malady and affliction, even though not recorded in the book of this law, the LORD will inflict on you until you are destroyed. Although once you were as numerous as the stars in heaven, you shall be left few in number, because you did not obey the LORD your God. And just as the LORD took delight in making you prosperous and numerous, so the LORD will take delight in bringing you to ruin and destruction; you shall be plucked off the land that you are entering to possess. The LORD will scatter you among all peoples, from one end of the earth to the other; and there you shall serve other gods, of wood and stone, which neither you nor your ancestors have known. Among those nations you shall find no ease, no resting-place for the sole of your foot. There the LORD will give you a trembling heart, failing eyes, and a languishing spirit. Your life shall hang in doubt before you; night and day you shall be in dread, with no assurance of your life. In the morning you shall say, ‘If only it were evening!’ and at evening you shall say, ‘If only it were morning!’—because of the dread that your heart shall feel and the sights that your eyes shall see. The LORD will bring you back in ships to Egypt, by a route that I promised you would never see again; and there you shall offer yourselves for sale to your enemies as male and female slaves, but there will be no buyer.” (Deuteronomy 28:15-68) ________________________________________________________ Of course, Jesus said, “Father forgive them for they know not what they do” (Lk 23:34). But in justice, He also said, we will not come out of jail until paid back is the last farthing (Matt 5:26). Compare the authoritative words of Deuteronomy to the ‘expertise’ of Bill Nye and evolutionistic consensus science; it’s a laugh if nothing else: http://www.lutheranscience.org/site/cpage.asp?cpage_id=180082464&sec_id=180015283 As for the big bang being the flagship of Christian evangelisation, think otherwise. We only need to look at: - http://crev.info/category/space/, https://biblescienceforum.com/, https://biblescienceforum.com/2016/08/16/quasar-exhibit-no-time-dilation-and-still-defy-a-big-bang-explanation/#more-5480, and http://open-site.org/Science/Physics/Cosmology_Problems_Big_Bang/, for example. Indeed, some may think, by making such an evangelical claim, BA77, you sail under the flag of the Jolly Roger. In conclusion, by your version of His personal word at Sinai, He is a lame duck God in need of Big Bang resuscitation through the good Dr Darwin. Though on second thoughts, the good doctor said, His was a “damnable religion.” Impossible to shock evolutionary sense into Him. Well, that's off my chest. I have a book to write. Time is pressing. The matter is one of faith: to take God at His word when all seems lost and you are scoffed at when to God all things are possible. Could it possibly be; His word is absolutely correct? All the best, mw. mw
Violence to Scripture, Hell and the Jolly Roger: (Part 3 of 4) BA77, you have not answered one of my philosophical and theological arguments. Are you afraid to admit the least possibility that the divine law is actually pure, perfect and true, beyond your understanding of any miracle? You or I cannot even find the abode of the spirit, let alone the place of heaven. You simply dismiss me as doing violence to scripture. Therefore, in honesty and truth, how can you say, but you know how God created? Have you seen God with His Hand on the trigger of a cosmic big bang bomb proving Him a liar? Therefore, how can you, in all honesty, say, 'I, BA77, testify against the word of God, that He did not create in six days.' Your soul BA77, on the matter? Yet you trash my belief with claims of violence to scripture built on the very word of the Judaeo-Christian God. Then, surely, it must be me who is going to hell in your belief system for doing violence to scripture? Well then, I shall be in good company with the Father, Son and Holy Spirit; God of Sinai, who said, every seven days to remember He created in six days; not six ages, as that’s what we will be doing! It appears, BA77, that you will not allow the smallest concession to a believer in strict divine law because you are afraid it may eventually undermine your view? Surely, if yours is that perfect, it can take on all comers without recourse to some form of Christian abuse, as it seems to me? Do you not realise, implied in your belief is one which actually makes out to save the word of God at Sinai, and hence by extension, saviour of our Savour, because you herald it as “one of the greatest apologetic tools in the arsenal of present day Christians” Therefore, the Word of God is insufficient to stand on its own, God’s word needs propping up by the Big Bang Theory! If so, then, down a black hole we go. Yet, your system of belief, which, make no mistake, I accept, because we are all at different stage posts on the way to the kingdom of heaven, appears to trash mine; contemptible and weak against the powerful and intellectual battering ram of consensus science, but not the word of the God of Sinai verbatim, which we will be judged by. Nevertheless, made perfect is God’s strength in weakness (2 Cor 12:9). And boy, does not creation in six days fit the bill? Belief in creation in six days is a test of pure faith, absolute trust in the God of Sinai. It is not blind faith, as Moses spoke face to face with God, even though hidden. Jesus came to set captives free, now we captivate and bind Jesus and the Father with evolutionism in its various forms. Times are about to change. Jesus fulfilled the law and the prophets. The law starts in Genesis. Jesus died for that law, as well as the Divine Law at Sinai. If not He died for believing a distortable law, the Father planted rotten fruit by not saying six days means ages and ages. In addition, Paul preaches myth. The law to eat of Godless knowledge, based on the beguilement God surely did not mean He literally created in six days, as theory tells us different, therefore, we think we become equal to God by determining how long creation took, and its method, yet without recourse to the knowledge of miracles. It seems to me, that if we cannot live the hardest teaching of God, we will weaken a Royal Personal Commandment, on which the whole of everyday worship originated for Judaeo-Christians, it is ourselves who do violence to the word of the Almighty, for whom nothing is impossible. Instead, we plant the White Flag over the mystery of Sinai. Instead, do we not rather adore the big bang theory, which to me, the only reasonable claim to any truth, if we are accurately honest, is that creation occurred. Jesus says, He is “the truth”, “the way,” and by Him keeping the law to the dot and tittle, to creation in six days. However, all scripture is dissolved in the acid of evolutionism, and Jesus is not the way; the Big Bang Theory and Darwinism is! I eventually came to believe God must have created in six days, because He Personally testified in stone His Command is Divine Law, that He created so. Stone is not a flexible medium symbolically; its use by God speaks volumes. Surely, more humility, more faith, much less theory, and less talk about violence! mw
Violence to Scripture, Hell and the Jolly Roger: (Part 2 of 4) Again, BA77, I must draw your attention to the matter that the Father’s commandments are essential in spiritual warfare, not just believing in Jesus. Satan will soon sift a person not attempting to keep to scripture and divine law: - “Then the dragon was angry with the woman, and went off to make war on the rest of her children, those who keep the commandments of God and hold the testimony of Jesus” (Rev 12:17). Without a doubt, by cohabiting evolutionism with divine law (ref Rev 12:9) Satan, “has deceived the whole world.” You may believe mathematically, everything physical can be reduced into a singularity; a dot containing no space and therefore in no place and no space, then included a cosmic pump to inflate theory and matter into no space in perpetual motion and energy; which frankly, is another big bang fudge. Whereas, great difficulties arise in people by not believing that everything was from everlasting in the mind of God before He created at will through the Word in six days. Judaeo-Christians believe God/Jesus created six large jars of water into mature wine instantly. Just a little example of creating a mature substance from a different substance; both of which He created anyway. Therefore, do you agree or disagree that the Almighty God could or could not have created on a bigger scale in six plain days, as His plain word said in His Testimony, which basically says: “I swear by Almighty God (Myself in this case) that the evidence I shall give on creation shall be the truth, the whole truth and nothing but the truth:’ (similar to an oath taken in England when a witness stands before the judge in court). A bet, your version of creation BA77, with the whole of big bang scientists against the word of God of Sinai? Would you bet your soul? Please answer this question. BA77, leave room for those compelled into the kingdom of heaven by the really narrow way, through the door which says, enter all who believe God created in six days. Whereas, you seem to say that inflation theory and big bang theory are superior means for believing the word of God at Sinai. Still, to meet your belief system you have to first inflate the Word itself! Your belief; God/Christ took ages and ages and ages and ages to create. Totally of course against a plain reading of divine, moral and ethical law. Surely to appease the god of evolutionism, besides ages and ages, chance and luck is also interspaced with a few cosmic mutations here and there, inflated with a Cosmic Natural Selector: as in Darwinism so in the Big Bang. You or I, BA77, have not the slightest inkling on how to produce a miracle at will. We cannot speak from experience of turning water to wine, let alone creating a cosmos from nothing, nor understand how such a maturing miracle may affect data. At least leave open the possibility, that some, need to believe the word of God, no matter how impossible. Catholics are one of the best at doing it, and at the same time the worst. Why, because we believe God becomes a Biscuit, or Droplets of Wine and multiplies Himself as many times as He wills. However, too many Catholics can't seem to believe that the Biscuit God we swallow and digest, also created in six days!!! Clearly, that's too much to swallow! Therefore, we end up with theory being superior to the plain teaching of God. If that is the case, the word of God does indeed bow to evolutionism. Made to bow is Christ before evolutionism. And to Catholics, Orthodox and indeed a few number of Protestants, with a UD orientation, I would ask, how old the God of transubstantiation, when the Biscuit God will test as made recently? mw
Violence to Scripture, Hell and the Jolly Roger: (Part 1 of 4). BA77, your comment at #237 deserves more attention: “Funny, I consider the big bang one of the greatest apologetic tools in the arsenal of present day Christians.” ______________________________________________________ In heaven, seen is the Ark of the Covenant (Rev 11:19). Of course, nothing unclean can enter either heaven or the Ark were the Testimony of the Word of God was placed. The Big Bang Theory makes the Genesis Sabbath Commandment unclean, not totally pure in truth. God’s law must speak plainly because His warnings are extremely plain. You claim, BA77, that I do violence to scripture (“whatever that means”). If that is the case, the Holy Trinity does “violence” to us by commanding us to believe He created in six days; initially on pain of death, therefore making the matter one of moral sin for Himself if you prove Him wrong by the unprovable Big Bang Theory, because your apologetic tool makes the God of Sinai inaccurate in speech. By believing in the Big Bang Theory as superior science to light the way for the God of Sinai, and the true Light of the world (Jn 8:12), we make Genesis wrong; wrong the world Flood; wrong the divine law; wrong the teaching of The Father; wrong the teaching of Christ on the Flood and the Fathers commandments; wrong, the teaching of Paul on Adam; wrong becomes the teaching by the Holy Spirit on the genealogy of Jesus; Wrong the totality of scripture inspired by the Holy Spirit; wrong our salvation based on unclear and imprecise law. Surely you are not saying keeping to divine law is violence to scripture? By altering one tittle of a divine law given by the personal word of God, the whole edifice of divine instruction will eventually crumble. Does not your apologetic tool advocate a superior false idol made in the image of theory, and by spreading its word: somewhat murders the true word of God and by extension faith in divine law: bears false witness against the Testimony and clear word of the God at Sinai: adulterates the word of God, making it inferior to a theory: loves the word of theory above the word of the God of Sinai; steals the genuine worship due to God, that He could not possibly have created in six days; and it rather covets the word of the big bang? If so, what an evangelisation: disfiguration of the Ten Commandments. Indeed, to cast out or disfigure plain divine speech not coated in riddles in order to theoretically evangelise for God is surely beguilement, which will eventually cripple Christ as He took all His instructions from the Father, God of Sinai. Nevertheless, I do accept your method of apologetics attracts customers. However, what the Holy Trinity did not say anywhere in scripture, nor is it even implied, that creation took ages. Time and time again you use scripture to say in the beginning was the Word. Yes, and the Word should know by now how to use words! You imply, in the beginning, was an uncertain, unclear Word. Who placed you judge over the Word of God at Sinai? What gives you a clear scriptural right to say I do violence to the strict Word, the unalterable Word? God could easily have said He created over a long time, and we are to remember it every seven days. Clearly, He did not. He laid down strict consequences for disbelieving and disobeying His Divine Law, as we shall come to in part 4. To scoff and accuse fellow Christians of doing violence to scripture by keeping to a divine law, means, eventually, you preach lawlessness. How can you say to another Man's servant, who keeps to the Master's teaching, which is, He fulfilled the law to the dot and tittle (Matt 5:17-19), say I do violence to the Master's teaching? Clearly, your version of believing in such a God, inadvertently cuts His Godliness, as God must be absolutely God in every word of God. You cannot then say absolutely, your version of God is God in everything, as He cannot be God of a literal six-day creation as His Literal God Word said. The word of the Judaeo-Christian God is vastly more important to some than unprovable theory. The strict word of God as an apologetic tool, cannot be improved on. It seems your God needs improving, BA77, hence you use the Big Bang Theory to improve a divine law. mw
one last, last, word: Out of curiosity, I checked the technical literature for their corrected model to see how robust it was. Let the reader decide for themselves if their model is robustly constructed:
The lithium isotopic ratio in very metal-poor stars?,?? K. Lind1,2, J. Melendez3, M. Asplund4, R. Collet4 and Z. Magic1 1 Max Plack Institute for Astrophysics, Karl-Schwarzschild-Strasse 1, 857 41 Garching bei München, Germany 2 Institute of Astronomy, University of Cambridge, Madingley Road, Cambridge, CB3 0HA, UK e-mail: klind@ast.cam.ac.uk 3 Departamento de Astronomia do IAG/USP, Universidade de São Paulo, Rua do Matão 1226, Cidade Universitària, 05508-900 São Paulo, SP, Brazil 4 Research School of Astronomy & Astrophysics, Australian National University, Cotter Road, Weston Creek, ACT 2611, Australia Received: 4 March 2013 Accepted: 24 May 2013 Abstract Context. Un-evolved, very metal-poor stars are the most important tracers of the cosmic abundance of lithium in the early universe. Combining the standard Big Bang nucleosynthesis model with Galactic production through cosmic ray spallation, these stars at [Fe / H] < ? 2 are expected to show an undetectably small 6Li / 7Li isotopic signature. Evidence to the contrary may necessitate an additional pre-galactic production source or a revision of the standard model of Big Bang nucleosynthesis. It would also cast doubts on Li depletion from stellar atmospheres as an explanation for the factor 3–5 discrepancy between the predicted primordial 7Li from the Big Bang and the observed value in metal-poor dwarf/turn-off stars. Aims. We revisit the isotopic analysis of four halo stars, two with claimed 6Li-detections in the literature, to investigate the influence of improved model atmospheres and line formation treatment. Methods. For the first time, a combined 3D, non-local thermodynamic equilibrium (NLTE) modelling technique for Li, Na, and Ca lines is utilised to constrain the intrinsic line-broadening and to determine the Li isotopic ratio. We discuss the influence of 3D NLTE effects on line profile shapes and assess the realism of our modelling using the Ca excitation and ionisation balance. Results. By accounting for NLTE line formation in realistic 3D hydrodynamical model atmospheres, we can model the Li resonance line and other neutral lines with a consistency that is superior to LTE, with no need for additional line asymmetry caused by the presence of 6Li. Contrary to the results from 1D and 3D LTE modelling, no star in our sample has a significant (2?) detection of the lighter isotope in NLTE. Over a large parameter space, NLTE modelling systematically reduces the best-fit Li isotopic ratios by up to five percentage points. As a bi-product, we also present the first ever 3D NLTE Ca and Na abundances of halo stars, which reveal significant departures from LTE. Conclusions. The observational support for a significant and non-standard 6Li production source in the early universe is substantially weakened by our findings. http://www.aanda.org/articles/aa/abs/2013/06/aa21406-13/aa21406-13.html
bornagain77
The measurements line up using the corrected models.
"Fudged" models you mean?
Since you have done this ‘honest omission’ twice now, of the most important part no less, I’m out of here and will let the unbiased reader judge who is being ‘dogmatic’ and who is dealing forthrightly with the evidence. The last word is all yours.
How could you possibly say I omitted and ignored it? I quoted and bolded the relevant section with respect to my posts for crying out loud. Your excuses are exceedingly pathetic though typical of your average OEC. They seem to suffer from selective blindness on a fundamental level. Oh well. Vy
* The Traditional Cosmological Lithium Problem: The lithium isotope ratio problem described just above is in addition to the traditional cosmological "lithium problem" which has been updated in a 2014 paper in the Monthly Notices of the Royal Astronomical Society. The traditional cosmological lithium problem is that, regardless of isotopes, the amount of observed lithium where theory attributes it to the big bang itself is inconsistent with big bang nucleosynthesis (BBN). Earlier, a secondary assumption was that the inconsistency was possibly a "local problem", perhaps only manifesting itself in our own or similar galaxies. So the authors asked:
...is the Li problem a local problem, limited to our Galaxy, or is it independent of the environment? The analysis of the RGB stars in M54 confirms the findings in ? Centauri (Monaco et al. 2010), considered as the remnant of an accreted dwarf galaxy: the Li problem seems to be an universal problem, regardless of the parent galaxy.
Thus Mucciarelli, et al., conclude:
Our result shows that this discrepancy is a universal problem concerning both the Milky Way and extra-galactic systems. Either modifications of BBN calculations, or a combination of atomic diffusion plus a suitably tuned additional mixing during the main sequence, need to be invoked to solve the discrepancy. MNRAS, 2014
I guess this is wrong also because [insert excuse]. Vy
You dishonestly keep leaving this part of my paper out:
Using observations of ancient stars with W. M. Keck Observatory’s 10-meter telescope and state-of-the-art models of their atmospheres has shown that there is no conflict between their lithium-6 and lithium-7 content and predictions of the standard theory of Big Bang nucleosynthesis, restoring thus the order in our theory of the early universe.
The measurements line up using the corrected models. Since you have done this 'honest omission' twice now, of the most important part no less, I'm out of here and will let the unbiased reader judge who is being 'dogmatic' and who is dealing forthrightly with the evidence. The last word is all yours. bornagain77
Aside from the misuse of the term "observations" to refer to a belief of what happened in the past, note the helpful admission that the alleged elements confirmation was "critical" to acceptance of big bang theory. Before the era of "precision cosmology", long ago in history back to the year 1990, a handful of scientists were determined to state for the record that the big bang theory had not predicted the relative abundances of hydrogen, helium, and lithium. Rather, they argued, big bang proponents were adjusting the theory's parameters to match already existing observations:
It is commonly supposed that the so-called primordial abundances of D (Deuterium, i.e., heavy hydrogen, N+P), 3He (Helium N+2P), and 4He (2N+2P) and 7Li (Lithium 3P+4N) provide strong evidence for Big Bang cosmology. But a particular value for the baryon-to-photon ratio needs to be assumed ad hoc to obtain the required abundances." -H. C. Arp et al., 1990 Nature 346, pp. 807-812
Vy
Vy, you are lost, I showed where they corrected the measurements, you showed where they experimentally confirmed the model.
That's what you got from this:
My paper says that the measurements for lithium were off, and when corrected they line up. Your paper, merely confirms that the lithium 6 and 7 should be such and such amount if big bang is correct. I don’t see where they disproved the earlier paper I cited solving the lithium problem, nor even where they acknowledged reading it.
Are you even reading your own posts? Here’s your quote:
One of the most important problems in physics and astronomy was the inconsistency between the lithium isotopes previously observed in the oldest stars in our galaxy, which suggested levels about two hundred times more Li-6 and about three to five time less Li-7 than Big Bang nucleosynthesis predicts. This serious problem in our understanding of the early Universe has invoked exotic physics and fruitless searches for pre-galactic production sources to reconcile the differences.
Now here’s mine:
With these new results, what is known as the “lithium problem” remains a hard nut to crack: on the one hand, now all laboratory results of the astrophysicists suggest that the theory of primordial nucleosynthesis is correct. On the other hand, many observations of astronomers show that the oldest stars in our Milky Way contain only half as much lithium-7 as predicted. Sensational reports by Swedish researchers, who discovered clearly more lithium-6 in such stars than predicted, must also likely be checked again based on the new LUNA data. Bemmerer says, “Should unusual lithium concentrations be observed in the future, we know, thanks to the new measurements, that it cannot be due to the primordial nucleosynthesis.”
Are you really not seeing it?
? Your paper hand-waves away the lithium problem by fudging values based on supposedly state-of-the-art models claiming the lithium problem doesn't exist. My more recent paper shows that the lithium does exist. You say they don't contradict each other. Wow!
You are not even trying to understand.
If only you weren't referring to yourself.
You paper does not even acknowledge my paper.
So every science paper on topic X acknowledges every other science paper on topic X. Awesome.
Try harder.
Please do! Vy
Vy, you are lost, I showed where they corrected the measurements, you showed where they experimentally confirmed the model. You are not even trying to understand. Your paper does not even acknowledge my paper, and when the two papers are taken holistically, the papers dovetail each others conclusions. Try harder. bornagain77
IMHO you are toying with an anomaly. So what!
Lol. The tune has changed.
Even if true it does not help you establish YEC as true. What is your prediction for lithium amounts using YEC predictions?
The dogma is strong with this one. Vy
Using observations of ancient stars with W. M. Keck Observatory’s 10-meter telescope and state-of-the-art models of their atmospheres has shown that there is no conflict between their lithium-6 and lithium-7 content and predictions of the standard theory of Big Bang nucleosynthesis, restoring thus the order in our theory of the early universe. bornagain77
Vy, the ‘dogmatic’ sentence you highlighted was your own words with YEC inserted
Ya think?
They are calling for new measurements? And this shores up your position how?
You seem to be lost. You (as per your article) claimed Li-6 and Li-7 abundances were as predicted. I showed that's false. Where did you see YEC? Vy
They are calling for new measurements? And this shores up your position how? "must also likely be checked again based on the new LUNA data. Bemmerer says, “Should unusual lithium concentrations be observed in the future, we know, thanks to the new measurements, that it cannot be due to the primordial nucleosynthesis.” IMHO you are toying with an anomaly. So what! Even if true it does not help you establish YEC as true. What is your prediction for lithium amounts using YEC predictions? bornagain77
Vy, the 'dogmatic' sentence you highlighted was your own words with YEC inserted, i.e. pot meet kettle. bornagain77
My paper says that the measurements for lithium were off, and when corrected they line up. Your paper, merely confirms that the lithium 6 and 7 should be such and such amount if big bang is correct. I don’t see where they disproved the earlier paper I cited solving the lithium problem, nor even where they acknowledged reading it.
Are you even reading your own posts? Here's your quote:
One of the most important problems in physics and astronomy was the inconsistency between the lithium isotopes previously observed in the oldest stars in our galaxy, which suggested levels about two hundred times more Li-6 and about three to five time less Li-7 than Big Bang nucleosynthesis predicts. This serious problem in our understanding of the early Universe has invoked exotic physics and fruitless searches for pre-galactic production sources to reconcile the differences.
Now here's mine:
With these new results, what is known as the "lithium problem" remains a hard nut to crack: on the one hand, now all laboratory results of the astrophysicists suggest that the theory of primordial nucleosynthesis is correct. On the other hand, many observations of astronomers show that the oldest stars in our Milky Way contain only half as much lithium-7 as predicted. Sensational reports by Swedish researchers, who discovered clearly more lithium-6 in such stars than predicted, must also likely be checked again based on the new LUNA data. Bemmerer says, "Should unusual lithium concentrations be observed in the future, we know, thanks to the new measurements, that it cannot be due to the primordial nucleosynthesis."
Are you really not seeing it? Vy
Vy, Your dogmatic adherence to the YEC seems to have rendered you incapable of considering any evidence against it.
Lol. If only. I started this argument with you because I was dogmatic. Very reasonable. The BB Lithium problem is one out of a plethora of evidences against the fudge theory and yet all you offer is a now disproved claim that was based on supposedly state-of-the-art "models". What was that about being dogmatic? Vy
My paper says that the measurements for lithium were off, and when corrected they line up. Your paper, merely confirms that the lithium 6 and 7 should be such and such amount if big bang is correct. I don't see where they disproved the earlier paper I cited solving the lithium problem, nor even where they acknowledged reading it. Perhaps, since it so important for you to believe in YEC, (even though this still does not help you even if it pans out), you can dig through the technical literature and find where they addressed the paper. bornagain77
Vy, Your dogmatic adherence to the YEC seems to have rendered you incapable of considering any evidence against it. bornagain77
Read carefully.
It would be nice if you followed your advice. Your reference:
International team strengthens Big Bang Theory June 6, 2013
Mine:
Measurement at Big Bang conditions confirms lithium problem Date: August 27, 2014
Your evidence is like digging up a paper from centuries ago claiming to prove phlogiston while ignoring all the recent counter-evidences. Vy
Read carefully. Using observations of ancient stars with W. M. Keck Observatory’s 10-meter telescope and state-of-the-art models of their atmospheres has shown that there is no conflict between their lithium-6 and lithium-7 content and predictions of the standard theory of Big Bang nucleosynthesis, restoring thus the order in our theory of the early universe. bornagain77
Your dogmatic adherence to the big bang seems to have rendered you incapable of logically discussing its merits or even considering any evidence against it. I wonder why you're not as close-minded about evolution. It has way more evidence i.e. zero. Vy
Your post is from 2013, mine is from 2014. Do the math. Vy
One of the most important problems in physics and astronomy was the inconsistency between the lithium isotopes previously observed in the oldest stars in our galaxy, which suggested levels about two hundred times more Li-6 and about three to five time less Li-7 than Big Bang nucleosynthesis predicts. This serious problem in our understanding of the early Universe has invoked exotic physics and fruitless searches for pre-galactic production sources to reconcile the differences. The team, led by Karin Lind of the University of Cambridge, has proven the decades-old inventory relied on lower quality observational data with analysis using several simplifications that resulted in spurious detections of lithium isotopes. Using observations of ancient stars with W. M. Keck Observatory's 10-meter telescope and state-of-the-art models of their atmospheres has shown that there is no conflict between their lithium-6 and lithium-7 content and predictions of the standard theory of Big Bang nucleosynthesis, restoring thus the order in our theory of the early universe. Read more at: http://phys.org/news/2013-06-international-team-big-theory.html#jCp
bornagain77
Vy, denial of denial does not constitute evidence for YEC.
You're not even trying. How could I deny the universe had a beginning? I'm a YEC! It's like saying Atheists believe God exists. Weird. Vy
You claim I'm denying evidence. Well let's see, from your quotes:
The predictions of Big Bang nucleosynthesis have been one of the main successes of the standard Big Bang model,” said lead author Lind
Reality:
There’s no escape; new measurements show far less lithium than predicted by the big bang, and more fine tuning than would be expected by chance. Measurements made by Italians deep underground confirm an old problem in cosmology: not enough lithium-7, but too much lithium-6. Science Daily explains:
With these new results, what is known as the “lithium problem” remains a hard nut to crack: on the one hand, now all laboratory results of the astrophysicists suggest that the theory of primordial nucleosynthesis is correct. On the other hand, many observations of astronomers show that the oldest stars in our Milky Way contain only half as much lithium-7 as predicted. Sensational reports by Swedish researchers, who discovered clearly more lithium-6 in such stars than predicted, must also likely be checked again based on the new LUNA data. Bemmerer says, “Should unusual lithium concentrations be observed in the future, we know, thanks to the new measurements, that it cannot be due to the primordial nucleosynthesis.
National Geographic sums up the problem simply: “That curious deficiency suggests that astrophysicists either don’t fully understand the big bang, they suggest, or else don’t fully understand the way that stars work.” Such a quandary suggests they could understand neither. “The most radical solution to the problem is that the big bang theory is incomplete,” said Brian Fields at the University of Illionois. “But less radical solutions haven’t yet solved the problem.”
The only way it makes sense is fudging. If only you were reading them. Sheesh! Vy
Vy, denial of denial does not constitute evidence for YEC. bornagain77
Denial of evidence does not constitute dealing with the evidence forthrightly.
This plus the fact that you just posted that OEC yom misinterpretation article proves to me you either don't read my posts or don't understand it. Vy
Old Earth Creation Science Word Study: Yom By Greg Neyman © 2007, Old Earth Ministries Published 16 March 2005 (This article can be freely copied and distributed, as long as it is unaltered and a link back to the original article appears on the page) The Hebrew word for “day” is the word “Yom.” Young earth creationists have always argued that the word used for the days of creation can only mean a 24-hour day. In this article, we will examine the uses of Yom in the Old Testament, and show that it can mean a wide variety of time periods. First, one must understand that the Hebrew language is not nearly as diverse as our English language. Whereas our vocabulary is around half a million, the Hebrew language has only 8,700 words. The French language, one of the poorest modern languages in vocabulary and the language of choice for diplomats, has just about 40,000 words or over 4 times the amount of words that Ancient Hebrew has. Many of the Hebrew words could be considered duplicates with only slight differences. Thus, words which contain multiple meanings are common. Such is the case with the word Yom. Hebrew Dictionaries Let’s start with the possible meanings of Yom; The Theological Wordbook of the Old Testament (1980, Moody Press) “It can denote: 1. the period of light (as contrasted with the period of darkness), 2. the period of twenty-four hours, 3. a general vague “time,” 4. a point of time, 5. a year (in the plural; I Sam 27:7; Ex 13:10, etc.).” Strong’s Exhaustive Concordance of the Bible (symbols omitted) from an unused root meaning to be hot; a day (as the warm hours), whether literal (from sunrise to sunset, or from one sunset to the next), or figuratively (a space of time defined by an associated term), [often used adv.]:–age, + always, + chronicles, continually (-ance), daily, ([birth-], each, to) day, (now a, two) days (agone), + elder, end, evening, (for)ever(lasting), ever(more), full, life, as long as (…live), even now, old, outlived, perpetually, presently, remaineth, required, season, since, space, then, (process of) time, as at other times, in trouble, weather (as) when, (a, the, within a) while (that), whole (age), (full) year (-ly), younger As you can see, Hebrew dictionaries attest to the fact that the word Yom is used for anywhere from 12 hours up to a year, and even a vague “time period” of unspecified length. Other Uses of Yom Day is not the only translation for the word Yom. Here are some other uses. Time It is interesting to note that in 67 verses in the Old Testament, the word Yom is translated into the English word “time.” For instance, in Genesis 4:3, it says “And in process of time it came to pass, that Cain brought of the fruit of the ground an offering unto the Lord.” In this instance, Yom refers to a growing season, probably several months. Again, in Deuteronomy 10:10, it refers to a “time” equal to forty days. In I Kings 11:42, it says “And the time that Solomon reigned in Jerusalem over all Israel was forty years.” In this case, Yom translated as the word “time” is equivalent to a 40 year period. In Isaiah 30:8, it says “Now go, write it before them in a table, and note it in a book, that it may be for the time to come for ever and ever.” In this case, Yom is equal to “forever.” How long is forever? An infinite number of years…billions upon billions upon billons of years. If Yom can equal trillions of years here, then why not billions of years in Genesis? Year Four times in the Old Testament Yom is translated “year.” In I Kings 1:1, “David was old and stricken in years…” In 2 Chronicles 21:19, “after the end of two years” and in the very next verse “Thirty and two years old.” Finally, in Amos 4:4, “…and your tithes after three years.” In each case, Yom represents years, not days. Age Eight times in the Old Testament Yom is translated “age.” These range from sentences like “stricken in age,” meaning old age (Genesis 18:11 and 24:1; Joshua 23:1 and 23:2), and other times it says “old age” (Genesis 21:2, Genesis 21:7). Genesis 47:28 refers to “the whole age of Jacob,” therefore yom here refers to an entire lifetime. In Zechariah 8:4, it says old men and women will sit in the streets of Jerusalem, “each with cane in hand because of his age.” Ago One time Yom is translated “ago.” 1 Samuel 9:20 says “As for the donkeys you lost three days ago, …” Always Four times yom is translated as “always,” in Deuteronomy 5:29, 6:24, 14:23, and in 2 Chronicles 18:7. Always here can be interpreted as a lifetime…for instance, we are to keep the commandments of the Lord always (Deut. 5:29). Season Three times yom is translated “season.” In Genesis 40:4, “…and they continued a season in ward.” Again, in Joshua 24:7, “dwelt in the wilderness a long season,” and in 2 Chronicles 15:3, “…a long season Israel hath been…”. In each case yom represents a multi-month period. Chronicles When used in conjunction with the word dâbâr, yom is translated “chronicles” (27 times). Continually When used in conjunction with kôwl, yom is translated as “continually” (11 times). Once, in Psalm 139:16, it is translated continuance (without the kôwl). Ever Ever is used to represent a long period of time, such as in Deuteronomy 19:9, “to walk ever in his ways.” Nineteen times Yom is translated “ever.” The old testament uses “for ever” instead of the word forever. In sixteen cases of use of the word ever, for is placed before it, indicating a infinite period of time. I will not list them all (consult Strong’s Concordance for a full listing) but here is an example. In Psalm 23:6, it says “Surely goodness and mercy shall follow me all the days of my life; and I will dwell in the house of the Lord for ever.” Here Yom is translated as the final word of this verse, ever. Thus, Yom in this verse, and 16 others, represents eternity. Evermore In one instance, when yom is used in conjunction with kôwl, Yom is translated “evermore.” Deuteronomy 28:29, “…and thou shalt be only oppressed and spoiled evermore;” thus representing either a lifetime or eternity. Word Usage in the Old Testament As you can see, Yom is used in a wide variety of situations related to the concept of time. Yom is not just for days…it is for time in general. How it is translated depends on the context of its use with other words. Yom in the Creation Account Even within the creation account, Yom is used to represent four different time periods. Genesis 1:5 “And God called the light Day, and the darkness He called Night.” Here, Moses uses Yom to indicate a 12-hour period Genesis 1:14 “And God said, “Let there be lights in the firmament of the heaven to divide the day from the night, and let them be for signs, and for seasons, and for days, and years.” Here, Moses uses Yom to indicate 24-hour days Genesis 2:4 “…in the day that the Lord God made the earth and the heavens.” Here, Moses uses Yom to indicate the entire creative week. The fourth usage of Yom in the creation account is in the summary for each of the six creation days, “and there was morning and evening the first day”. Yom is used to represent a finite, long period of time, usually either millions or billions of years. To show support for this, consider the uses of Yom by Moses. Moses Other Uses of Yom Moses, the author of the first five books of the Bible, and of Psalm 90, used Yom in many different ways. Genesis 4:3 “And in process of time it came to pass, that Cain brought of the fruit of the ground an offering unto the Lord.” In this instance, Yom refers to a growing season, probably several months. Genesis 43:9 “…then let me bear the blame for ever.” Here, Moses uses Yom to represent eternity Genesis 44:32 “…then I shall bear the blame to my father for ever.” Again, Moses uses Yom to represent eternity Deuteronomy 4:40 “…that thou mayest prolong thy days upon the earth, which the Lord thy God giveth the, for ever.” Here Yom represents a physical lifetime Deuteronomy 10:10, “Now I stayed on the mountain forty days and nights, as I did the first time,…” Here, Yom is a “time” equal to forty days. Deuteronomy 18:5 “…to stand to minister in the name of the Lord, him and his sons for ever.” Again, Yom is translated as eternity Deuteronomy 19:9 “…to love the Lord thy God, and to walk ever in His ways…” Here, Yom represents a lifetime. As long as we live we are to walk in his ways As you can see, Moses used the word Yom to represent 12-hours, 24 hours, the creative week, forty days, several months, a lifetime, and eternity. Common Young Earth Arguments To get around the obvious conclusion that Yom in Genesis 1 can mean millions of years, young earth theorists have come up with several arguments, none of which is supported by common Hebrew grammatical rules according to Hebrew experts (such as Dr. Walter Kaiser). These rules were created by Hebrew language experts who are young earth creationists to begin with, thus their viewpoint is obviously biased. They have a specific agenda they are trying to prove, and thus cannot be objective. Ordinals/Cardinals Young earth creationists say that whenever Yom is used with an ordinal or cardinal number (1st, 2nd, 1,2, etc) that it always represents a 24 hour day. However, this is not true. In Zechariah 14:7-9, the “one day” refers to a period of time when the Lord shall be king over the earth. In other places, some say that Isaiah and Hosea have numbers with the word day which are figurative (External Link). Some young earth theorists, including Jonathan Sarfati in his book Refuting Compromise, have addressed this verse in Zechariah an Hosea. Although his argument sounds impressive, you have to recognize it for what it is…he is arguing for his young earth agenda, thus any rules that he espouses must be examined by true Hebrew scholars who are impartial. Hebrew scholars do not recognize this fabricated rule.1 What Sarfati thinks is not important…what is important, as Dr. Walter Kaiser points out, is the intentions of the author. We should not create rules that support our own agendas, but should strive to understand the author’s intended meaning outside of rules. Evening/Morning Construction In Genesis 1 Moses says “and there was evening and morning the xx day”. Does the use of evening and morning indicate a sunrise and sunset for each creative day? First, let’s look at what evening and morning are not. They are not actual evening and mornings, as this requires a sunrise and sunset. According to young earth theory, the Sun was not created until Day Four, thus there could be no sunrise or sunset for the first three days of creation. However, God uses the terms evening and morning for those first three days. Therefore, they cannot be actual evenings and mornings. We are left with only one option. The words for Evening and Morning can only represent the beginning and ending of the creative period, and not actual sunrise and sunsets. Scripture itself sets this pattern for us. Morning and evening are used figuratively in Psalm 30:5, Psalm 49:14,15, Psalm 90:6. Thus, the evening and morning of creation can mean the start and end of the creative process that is attributed to that creation period. Young earth advocates counter that traditionally, church fathers have always held that sunrise and sunsets do not constitute a day, and they accepted the sun creation on Day Four with no hint of the first three days being anything other than 24-hour days. For instance, Sarfati in Refuting Compromise mentions Luther and Calvin (page 84-86). However, Luther and Calvin did not have the means of modern science at their disposal. At the time, geocentricity was still accepted! Don’t fall into the trap of following the teachings of our church fathers. For more, read Church Fathers. Literal/Figurative Argument This argument says that you cannot use a word figuratively until after you have used it literally (see this Answers in Genesis article). The author gives two examples, which appear to be correct and follow this rule. However, is this rule valid? I see no reason to suppose that it is. You have to be careful with young earth claims about biblical interpretation methods. Again, they will invent rules that support their cause, when there is no basis for their rule in Hebrew. In this case, it makes no difference which order the word Yom appears in, i.e. literal before figurative or vice versa. Yes, these are the first words of the Bible, but they are not the first words of mankind. All the time from Adam to Moses, men were speaking in their own languages, thus the literal interpretation via spoken language would already have been established. There was no need to suppose a literal/figurative structure. If God’s Creation Was Billions of Years Old… If God’s creation was billions of years old, how would He have written the creation account in Genesis? One thing is certain…God is good at telling us exactly what we need to know. When God refers to a large number, He uses picture stories, such as Abraham’s descendants being as numerous as the sand. Why does He do this? If God had said, “You will have millions of descendants,” Abraham would have asked, “What is a million?” When considering the creation, if we broke it down into days, that would be 5,000,500,000,000 days, or roughly 13.7 billion years. Do we need an account for each day of creation…of course not. God in His infinite wisdom, saw fit to tell us the creation story by breaking it down into creative segments, each of which was attributed to a specific creative act or acts. We need to give the early Hebrews of Genesis a break…they didn’t have calculators like we do! One must also consider that time with God has no meaning. To Him, 10 billion years is like a day. Thus, it is no problem for God to put billions of years into one of His days. Dr. Hugh Ross puts it best in his determination that the frame of reference for creation is the surface of the earth. Genesis 1:2 puts the witness of creation on the surface. But who is witnessing these events? It is God himself. During the first 5.99 days of creation, God is the only one present. Thus, human time does not matter…no humans were there to witness the passage of time. What matters is how God sees time! Thus, a billion year day is only a passing moment in God’s eyes. The creation account is written in such a manner for all people to understand it. The issue is not how long creation took…the issue is that God did it, and that’s all that matters in the end. Conclusion With such a wide usage of the word Yom for many different time periods, it cannot be claimed that Yom in the Old Testament only represents a 24-hour period. During the creation account alone, Yom represents four different time periods. Rules of Hebrew, created by young earth Hebrew scholars, are invalid. Because of their biased position, they are trying to prove their own agenda. Since humans did not witness creation, our own concept of a 24-hour day does not apply. The only thing that matters is God’s concept of time. Thus, the only evidence we have to accurately assess the age of creation is the creation itself. Since the rocks and stars say we are billions of years old, that must be the truth. This fits perfectly with a literal interpretation of Genesis, and an inerrant Bible, and does not impact any other Biblical doctrines. 1 Television Show and Transcript, “Are the Genesis Creation Days 24 Hours or Long Periods of Time,” The John Ankerberg Show, 2005. http://www.oldearth.org/word_study_yom.htm
Question: Do you, like mw apparently does in his heart of hearts, believe only YECs go to heaven? Supplemental notes:
Special and General Relativity compared to Heavenly and Hellish Near Death Experiences https://www.youtube.com/watch?v=TbKELVHcvSI&list=PLtAP1KN7ahia8hmDlCYEKifQ8n65oNpQ5&index=1 Albert Einstein vs “The Now” of Philosophers and “The Now” of Quantum Mechanics https://www.youtube.com/watch?v=dwyHUxoKWNM&list=PLtAP1KN7ahia8hmDlCYEKifQ8n65oNpQ5&index=3 ‘In the ‘spirit world,,, instantly, there was no sense of time. See, everything on earth is related to time. You got up this morning, you are going to go to bed tonight. Something is new, it will get old. Something is born, it’s going to die. Everything on the physical plane is relative to time, but everything in the spiritual plane is relative to eternity. Instantly I was in total consciousness and awareness of eternity, and you and I as we live in this earth cannot even comprehend it, because everything that we have here is filled within the veil of the temporal life. In the spirit life that is more real than anything else and it is awesome. Eternity as a concept is awesome. There is no such thing as time. I knew that whatever happened was going to go on and on.’ In The Presence Of Almighty God – The NDE of Mickey Robinson – video (testimony starts at 27:45 minute mark) https://www.youtube.com/watch?v=s66DchGhhD0 ‘When you die, you enter eternity. It feels like you were always there, and you will always be there. You realize that existence on Earth is only just a brief instant.’ Dr. Ken Ring – has extensively studied Near Death Experiences ‘Earthly time has no meaning in the spirit realm. There is no concept of before or after. Everything – past, present, future – exists simultaneously.’ – Kimberly Clark Sharp – Near Death Experiencer ‘There is no way to tell whether minutes, hours or years go by. Existence is the only reality and it is inseparable from the eternal now.’ – John Star – NDE Experiencer
as to:
"All your big bang “evidence” is dealt with in the article I linked to."
Denial of evidence does not constitute dealing with the evidence forthrightly. bornagain77
All your big bang "evidence" is dealt with in the article I linked to. Vy
It means that if you want to read the bible ‘consistently’ then an OEC view is what you must take.
Didn't we already do this? - "Yom" as used in Genesis is obviously not referring to long periods but normal 24 hour days. From my post at #194:
I’d love to know which of the following verses, when considered in light of what YECs actually based their interpretation of ‘Yom’ as used in the creation, you’re referring to when you say “what about all the times Yom is clearly used referring to a long period of time?”: ———— ‘First day’ – 22 occurences ‘Second day’ – 13 exact matches ‘Third day’ – 44 occurences ‘Fourth day’ – 7 exact matches ‘Fifth day’ – 4 exact matches ‘Sixth day’ – 6 exact matches ‘Seventh day’ – 45 occurences
But don't worry, you can dance around this. - Being a YEC does not mean reading every part of the Bible literally. It never did. Some OECs don't seem to get that. Now, would you mind explaining reading the bible "consistently"?
Denial of the evidence for a beginning of the universe?
What are you talking about? :/ Are you assuming because I disbelieve in the fudge theory that is the big bang, I must believe the universe had no beginning? Vy
"What does it even mean?" It means that if you want to read the bible 'consistently' then an OEC view is what you must take.
,,, ‘And if you’re curious about how Genesis 1, in particular, fairs. Hey, we look at the Days in Genesis as being long time periods, which is what they must be if you read the Bible consistently, and the Bible scores 4 for 4 in Initial Conditions and 10 for 10 on the Creation Events’ Hugh Ross – Latest Scientific Evidence for God’s Existence – video 56:14 minute mark https://youtu.be/d4EaWPIlNYY?t=3374
as to
"Too bad it’s a theory of unobserved and unobservable fudge factors."
Denial of the evidence for a beginning of the universe? I would expect as much from atheists, but it really is surprising that some Christians would be as bad as atheists, even worse, in their denial of the evidence for the beginning of the universe:
Big Bang Exterminator Wanted, Will Train - Denyse O'Leary - October 20, 2013 Excerpt: "Perhaps the best argument in favor of the thesis that the Big Bang supports theism is the obvious unease with which it is greeted by some atheist physicists. At times this has led to scientific ideas, such as continuous creation or an oscillating universe, being advanced with a tenacity which so exceeds their intrinsic worth that one can only suspect the operation of psychological forces lying very much deeper than the usual academic desire of a theorist to support his/her theory." Cosmologist Christopher Isham http://www.evolutionnews.org/2013/10/big_bang_exterm077961.html Big Bang Theory - An Overview of the main evidence Excerpt: Steven Hawking, George Ellis, and Roger Penrose turned their attention to the Theory of Relativity and its implications regarding our notions of time. In 1968 and 1970, they published papers in which they extended Einstein's Theory of General Relativity to include measurements of time and space.1, 2 According to their calculations, time and space had a finite beginning that corresponded to the origin of matter and energy."3 Steven W. Hawking, George F.R. Ellis, "The Cosmic Black-Body Radiation and the Existence of Singularities in our Universe," Astrophysical Journal, 152, (1968) pp. 25-36. Steven W. Hawking, Roger Penrose, "The Singularities of Gravitational Collapse and Cosmology," Proceedings of the Royal Society of London, series A, 314 (1970) pp. 529-548. http://www.big-bang-theory.com/ International team strengthens Big Bang Theory Jun 06, 2013 Excerpt: The fundamental observations that corroborate the Big Bang are the cosmic microwave radiation and the chemical abundances of the light elements described in the Big Bang nucleosynthesis theory. "The predictions of Big Bang nucleosynthesis have been one of the main successes of the standard Big Bang model," said lead author Lind. "Our findings remove much of the stark tension between 6Li and 7Li abundances in stars and standard BBN, even opening up the door for a full reconciliation. This further consolidates a model resting heavily on the pillars of the cosmic microwave background and the expanding Universe." http://phys.org/news/2013-06-international-team-big-theory.html#nwlt Evidences For The Big Bang - Michael Strauss – video (4:50 - mark - main evidences) (14:30 mark - unscientific speculations involving quantum Planck time persist) https://vimeo.com/9195703 Evidence Supporting the Big Bang http://www.astronomynotes.com/cosmolgy/s7.htm Direct (Distance) Measurements Place Universe’s Age at 13.79 Billion Years – Hugh Ross – May 2013 – podcast http://www.reasons.org/podcasts/science-news-flash/direct-measurements-place-universe-s-age-at-13.79-billion-years The Megamaser Cosmology Project. V. An Angular Diameter Distance to NGC 6264 at 140 Mpc; http://arxiv.org/abs/1207.7292
bornagain77
"Well, does not God do violence to us by first commanding us to believe He created in six days; initially on pain of death?" So mw, do you or do you not believe that only YECers go to heaven? You can't have it both ways. You can't say in one instance that you don't believe only YECers go to heaven and then in the next instance say you believe God commands us the believe in YEC 'initially on pain of death'. Does 'initially' in this instance mean that not believing in YEC was a sin that Christ had to atone for? Do I need to repent of OEC in your belief system? Need I remind, I find that your YE interpretation of the bible to be seriously misconstrued?
My cat ate your dogma - picture https://s-media-cache-ak0.pinimg.com/736x/a6/e4/1a/a6e41ab2a11d89228a9ee93253d2e7a6.jpg
bornagain77
... and still resolutely think your YE interpretation does violence to scripture overall.
You keep on harping on about this. What does it even mean?
Funny, I consider the big bang one the greatest apologetic tools in the arsenal of present day Christians
No surprise there. Too bad it's a theory of unobserved and unobservable fudge factors. Vy
"HMMM mw, so you don’t believe in the ‘big bang’ since it contradicts your YEC? Funny, I consider the big bang one the greatest apologetic tools in the arsenal of present day Christians." _____________________________________________ Really: I consider the greatest apologetic tool in the arsenal of Judaeo-Christianity is the personal Testimony of the Holy Trinity through Yahweh. The only scripture the Divinity ever wrote. Thus, it has the approbation of heaven. Name one big bang scientist in the same league. If Sinai is true, the big bang is one of the biggest cons in the history of Satan. Still, the big bang theory appears to knock the stuffing out of him also. Yes, your quotes are very impressive, which I read with interest, and use in some cases. None of which matches the word of God of Sinai. Therefore, I will stick to what God said in an historical, witnesses and recorded event. Jesus verified the Commandments, which He fulfilled, and did not destroy one tittle. Whereas, with respect  BA77, the world appears to side with you, which however, spins or destroys a commandment Jesus upheld to the letter. Yet, there are scientists who do reject the big bang theory, even less who believe in a Hildegard system, but certainly a reasonable few believe God created in six days. That is a miracle in itself. You are persistent BA77, that I do violence to scripture. Well, does not God do violence to us by first commanding us to believe He created in six days; initially on pain of death? Indeed, does not your logic imply, in the Ten Commandments, God works violence. Unless His word is totally true. There cannot be shades of truth and shades of salvation. mw
HMMM mw, so you don't believe in the 'big bang' since it contradicts your YEC? Funny, I consider the big bang one the greatest apologetic tools in the arsenal of present day Christians
The best data we have [concerning the Big Bang] are exactly what I would have predicted, had I nothing to go on but the five books of Moses, the Psalms, the bible as a whole. Dr. Arno Penzias, Nobel Laureate in Physics - co-discoverer of the Cosmic Background Radiation - as stated to the New York Times on March 12, 1978 “Certainly there was something that set it all off,,, I can’t think of a better theory of the origin of the universe to match Genesis” Robert Wilson – Nobel laureate – co-discover Cosmic Background Radiation “There is no doubt that a parallel exists between the big bang as an event and the Christian notion of creation from nothing.” George Smoot – Nobel laureate in 2006 for his work on COBE "Now we see how the astronomical evidence supports the biblical view of the origin of the world. The details differ, but the essential elements in the astronomical and biblical accounts of Genesis are the same: the chain of events leading to man commenced suddenly and sharply at a definite moment in time, in a flash of light and energy." Robert Jastrow – Founder of NASA’s Goddard Institute – Pg.15 ‘God and the Astronomers’ ,,, 'And if you're curious about how Genesis 1, in particular, fairs. Hey, we look at the Days in Genesis as being long time periods, which is what they must be if you read the Bible consistently, and the Bible scores 4 for 4 in Initial Conditions and 10 for 10 on the Creation Events' Hugh Ross - Latest Scientific Evidence for God's Existence – video 56:14 minute mark https://youtu.be/d4EaWPIlNYY?t=3374 Science and Creation - Dr Michael Strauss - video (at approx 17:00 minute mark the old earth interpretation of Genesis 1 is discussed) https://youtu.be/EZJozX3sbE0?t=1017 Dr. Michael Strauss (Professor of Physics at Oklahoma) gives a lecture and Q&A about Science and Creation at the University of Nebraka-Lincoln (March 2015).
Do you also reject Qauntum Mechanics since it says the universe did not exist 10^-43 seconds ago and that would also obviously conflict with your YEC worldview?
“Look, we all have fun ridiculing the creationists who think the world sprang into existence on October 23, 4004 BC at 9AM (presumably Babylonian time), with the fossils already in the ground, light from distant stars heading toward us, etc. But if we accept the usual picture of quantum mechanics, then in a certain sense the situation is far worse: the world (as you experience it) might as well not have existed 10^-43 seconds ago!” – Scott Aaronson – MIT associate Professor quantum computation - Lecture 11: Decoherence and Hidden Variables
As well, Do you believe the earth orbits the sun? Or do you 'scripturally' believe that the Earth is the center of the universe? It might surprise you to find out that I do!
https://uncommondescent.com/intelligent-design/materialists-do-not-believe-that-the-earth-orbits-the-sun-is-objectively-tr/#comment-615600
bornagain77
Sorry, spelt "new" wrong. mw
I new something else was left unsaid. "Perhaps if you get a couple of people in the atheistic crowd at TSZ to convert to your YEC beliefs, or at least become Christians, then I would be more receptive to your argument(s)?" Surely, that is like saying, unless I can feel the wounds in His hands, I will not believe (Jn 20:25); or rather, 'unless I can speak to God face to face and ask Him, "did you create in six days or not," I will not believe?' mw
BA77, I am absolutely not the one to convince anyone. How in the world did God get anyone to believe He created in six days? By a show of power and a death law, plus continuous daily miracles over 40 years. Today, who cares? Anyway, you have not answered any relevant argument I have made, scripturally that is. The world is mainly reflected in your response. You continue to bring up hell, when all manner of sins and blasphemies will be forgiven. Heavens above, I have done my fair share! It appears, that the worst enemy from your point of view is a fellow Christian,  those who profess to keep wholeheartedly to the Word, certainly the Divine Law Commandments. But, with respect BA77, you have not won your position scripturally, and you cannot authenticate the Big Bang. Perhaps at the entrance to heaven, you will have rehearsed what you will say to the God of Sinai, 'Lord you made a big mistake by saying creation took six days, but I took notice of some fine scientists." And as for Jesus on His return, will he salute the Big Bang Theory, and institute a divine correction, or teach the Father's divne law, which he died keeping for our sake, as instructed in obedience to the Father? I think we will agree to disagree, and leave it at that, as I have other places to be scoffed at. We learn nothing by not allowing our beliefs to be tested. And as for being directed elsewhere, it speaks volumes, does it not? Better to be tested with the hardest arguments. Christians trample best the word of God underfoot, not atheists. This, I hope, is my final comment on this thought provoking thread. By all means have the last word; against the personal testimony of God! (Forgive me for such a cheap shot). All the very best, mw mw
So mw, are you saying that you do believe I am going to hell for not being a young earther? You say you don't in one sentence but your supposed clarifications of the matter seem to directly contradict your denial. If it is of any comfort for you, I am quite comfortable with an old earth interpretation of scripture, and still resolutely think your YE interpretation does violence to scripture overall. None of your pontificating on YEC has nudged my belief in OEC in the least. Perhaps you should focus your energy elsewhere, besides trying to convert me, a Christian, into YEC since your efforts have been so futile in that regards? Perhaps if you get a couple of people in the atheistic crowd at TSZ to convert to your YEC beliefs, or at least become Christians, then I would be more receptive to your argument(s)? http://theskepticalzone.com/ bornagain77
BA77, with respect, you appear to be obsessed with you and others going to hell for disbelieving that God created in six days. You may cite as many citations you wish, which I read with interest, but you will not cite the accurate and truthful personal Testimony of God, one which you cannot accept at this moment. If His Testimony is not good enough for you, what is? As you elasticate the word of God, where or when does your method of believing stop? You cannot clearly address my last comment, nor of those embedded in the whole of my writings in this post. Therefore you bring in lesser fallen human consideration, and on top of that, you appear to make out that I condemn such science. Please do not keep following your own misleading train of thought. I for one accept your belief system, but I have right to argue against it according to Scripture. Jesus, against Satan, teaches volumes, "we live by every word from the mouth of God" (Matt 4:4). It appears you cannot agree with Jesus or the Father at Sinai, both who are one God (Jn 10:30). I believe, God must take account of the times we are in; such a beguiled time, such a disbelieving time, and all through the dragon in scientific clothing; through evolutionism in its various forms, God is eclipsed. However, I expect much trouble is on the way to change the face of the earth, and that we shall all taste it in some measure. mw
"Still, the one who sent him said he created in six days!" So in your belief system, since I do not believe in a six literal 24 hour day creation, but in six long periods of time creation, then I am not 'hearing God' according to you and am therefore not going to heaven in your belief system? Do you think Kelvin is in hell for believing in Old Earth creation?
"We have the sober scientific certainty that the heavens and earth shall ‘wax old as doth a garment’.... Dark indeed would be the prospects of the human race if unilluminated by that light which reveals ‘new heavens and a new earth.’" Sir William Thomson, Lord Kelvin (1824 – 1907) – pioneer in many different fields, particularly electromagnetism and thermodynamics. “But I may be permitted to remark that a correction of this kind cannot be said to be unimportant in reference to biological speculation. "The limitation of geological periods, imposed by physical science, cannot, of course, disprove the hypothesis of transmutation of species; but it does seem sufficient to disprove the doctrine that transmutation has taken place through "descent with modification by natural selection” Sir William Thomson, Lord Kelvin (1824 – 1907) https://books.google.com/books?id=wCNGAQAAMAAJ&pg=PA532
bornagain77
Indeed, BA77, and I am up to speed at last. You say rightly: ".....whoever hears my word and believes him who sent me has eternal life and will not be judged but has crossed over from death to life." Still, the one who sent him said he created in six days! The message of Jesus is twofold and clear - "and believes him who sent me". "And no one goes to the Father but through me" (Jn 14:6), that's another problem. mw
"Do only young earthers go to heaven in your belief system?” ,,, to be very clear, no I don’t." Okie Dokie.
John 5:24 "Very truly I tell you, whoever hears my word and believes him who sent me has eternal life and will not be judged but has crossed over from death to life.
bornagain77
Violence to Scripture-Addendum response to BA77. Sorry BA77, that I have taken a week to respond to “mw, apparently, from all the energy you’ve put into your responses, you think that a young earth interpretation is very important. So, is or is not the young earth interpretation essential for salvation? i.e. Do only young earthers go to heaven in your belief system?” However, I answered that question at #219 (or so I thought). So to be very clear, no I don’t. Many who are “first will be last and the last first” (Matt 19:30). Still, every knee will have to bow to Jesus, and His divine law from Sinai. As this post is about the Big Bang (Urknall). A few last thoughts. The letter below was send by me to the editor of the Catholic Times (England) last weekend: Fr --------- (Credo, The Catholic Times, 16th September) appears not to see a black hole in Mgr Lemaître’s calculations about a theoretical big bang over 13.7 billion years. Lacking are equations on the super science of miracles generated at instantaneous speed and at God’s will. The Big Bang Theory cannot address any dimension of the spirit. His calculations do not account for the personal almighty power of the infinite God, and therefore, are unable to disprove or prove how God created from nothing physical. God speaks, and it its done (Ps 33:9): work that out! For example, calculate by what way God created Himself from the dead instantly, repeat the process and join it with the Big Bang Theory. Not only that Jesus generated a new species of human, instantly; one which could traverse between spirit and matter. No Darwinian theory needed. However, if Mgr Lemaître said his calculations are the best natural science can do, but adhered to must be divine law, as Jesus said, that is a different matter. The Ten Commandments are the only scripture written by the Holy Trinity, that speaks volumes. God/Jesus said, “Do not think that I have come to abolish the law or the prophets; I have come not to abolish but to fulfil. For truly I tell you, until heaven and earth pass away, not one letter, not one stroke of a letter, will pass from the law until all is accomplished” (Matt 5:17-19). Therefore, Jesus fulfilled the divine law that God/He created in six days. Against the words of Jesus, it is delusional and unscriptural for Fr ------, normally a worthy and fine priest, to claim we can stand next to Jesus in “bliss,” with Lemaître, for destroying divine law. ________________________________________________________ Clearly, what I am saying is that any other interpretation to clear and plain scripture surely does violence to scripture? You may say the Bible is not scientific. No, it is only the word of God we profess to believe in! More so, it is the word of God written by Him personally at Sinai. In addition, the whole of scripture is inspired by the Holy Spirit (2 Tim 3:16). Pull one sweet apple away and the whole tree of faith turns sour or crinkled and would eventually rot if it were not for some divine intervention. If there was no Self-Revelation by God at Sinai, and in the Flesh, we would all be big bangers and atheists. The Divine Ten Commandments contain all essential moral and ethical laws. If one is deemed grossly in error, then Judaeo-Christians cannot moralise to anyone, as we demonstrate we cannot keep to believing our own ethical standards as given by Jesus/God when He created through Christ in six days, and said every seven days to remember that feat of almighty power and will and instantly. _______________________________________________________ However, in relation to a cosmic theory of creation, I came across the writings of Hildegard of Bingen; probably it will seem more foolish for me for mentioning it. Though I must say, as a Catholic, from experience, the best scoffers are Catholics! Saint Hildegard of Bingen, on 7 October 2012, the feast of the Holy Rosary, Pope Benedict XVI named her a Doctor of the Church. He called her “perennially relevant” and “an authentic teacher of theology and a profound scholar of natural science and music.” Her apparently divinely revealed celestial mechanics certainly opposes the Big Bang Theory, and provides room for investigation. A new low cost e-book book by Helmut Posch, is worth noting: The True Conception of the World according to Hildegard von Bingen, (2015). http://kolbecenter.org/store-2/#!/Books-PDF/c/13115134/offset=0&sort=addedTimeDesc He writes: “A few years ago a mathematical institute in Switzerland demonstrated that one can also look at the circulation of the planets as a centripetal effect of an eddy field and then make calculations that are just as exact” (as the Copernicus system). A very limited summary of her revelation is - In the Hildegard system, the whole cosmos is bounded as a cell, nothing goes beyond those bounds of space, and space does not move. The Cosmic Cell (I have just Christened it so) is a perpetual motion machine into which God supplies energy for all movement and life leading apparently to some form of cosmic winds and waves resulting in pressure differential and the explanation for gravity. Jesus allegedly said “I am fiery force hidden in these. They blaze up out of me!” (Divine Words, 42). The outer shell may alter shape somewhat. In simplistic terms, four elements make up the creation. Humans are interconnected, and our behaviour effects the whole. Distant stars and a cosmic winds/waves even effect even our cloud formation. Everything is interconnected elementally. The system is not simple. There is also a local system amidst universal space currents. The system may be called the “Spirit of Science.” The system is human centred and earth centred. Therein, under present conditions it takes some believing. Still, what is prime faith based on, it is not consensus science that is for sure. Nevertheless, apparently, after the judgment, the Cosmos will no longer rotate, as in Paradise, and there will be perpetual day. All the best, mw. mw
HeKS @223
HeKS: Anything that exists within a larger context is necessarily contingent upon its context. This is true whether the context is space, or time, or constraining laws, or variable motion, or probabilistic outcomes, or material components, or creative agency, or all of the above, or anything else.
This is a very important ontological point. Equally crucial IMHO is that persons have the ability to, temporarily, accept a wide variety of contexts. We can choose the act within an arbitrary context and its arbitrary rules — chess, society, fantasy scenarios and arguably this very universe. If the term ‘temporarily contingency’ would make sense, then it would apply here.
HeKS: The entirety of the context that informs a physical object’s existence, state of being and activity would need to already exist, each part in proper relation to the other, before the object itself could exist. This would apply all the way back to the beginning of the universe itself and even any quantum regime that may or may not have existed at that point. Physicality, at all points, is contingent upon composition with respect to its own parts, the spacetime it occupies, and the laws that govern its behavior. An electron does not escape this fact, even if one wants to consider it “one thing”. It is still “one thing” that is contingent upon its larger context.
I fully agree. I notice your reluctance to consider an electron “one thing”. Indeed, from a strict ontological point of view the universe, its laws and every physical thing in it would be a better candidate for the title “one thing”. But since it is contingent it also fails to qualify.
HeKS: So no physical object could possibly qualify as an uncaused cause.
Only what is truly “one thing” can qualify as an uncaused cause. Origenes
mw, apparently, from all the energy you've put into your responses, you think that a young earth interpretation is very important. So, is or is not the young earth interpretation essential for salvation? i.e. Do only young earthers go to heaven in your belief system? bornagain77
Violence to Scripture: To balance a Catholic interpretation of "Yom," cited # 224, which, while offering reasoned views, holds the sun was created before day four, which is not entirely satisfactory to some, therefore, one final article, cited from Answers in Genesis is offered: https://answersingenesis.org/creationism/young-earth/reply-to-bruce-gordons-biblical-critique-of-young-earth-creationism/ mw
Violence to Scripture – I thought at least one scholarly counter to BA77’s reference to “Yom” may be useful: http://kolbecenter.org/meaning-yom-genesis/ mw
Origenes
Suppose that someone claims that an electron is one thing and not a composite whole. What would be your response?
Personally, I'd say what I said in #214:
Anything that exists within a larger context is necessarily contingent upon its context. This is true whether the context is space, or time, or constraining laws, or variable motion, or probabilistic outcomes, or material components, or creative agency, or all of the above, or anything else. The entirety of the context that informs a physical object’s existence, state of being and activity would need to already exist, each part in proper relation to the other, before the object itself could exist. This would apply all the way back to the beginning of the universe itself and even any quantum regime that may or may not have existed at that point. Physicality, at all points, is contingent upon composition with respect to its own parts, the spacetime it occupies, and the laws that govern its behavior.
An electron does not escape this fact, even if one wants to consider it "one thing". It is still "one thing" that is contingent upon its larger context. This would seem to have some implications for the possible number of uncaused causes. Anything that is physical is contingent upon its context, and anything that is contingent cannot be uncaused. Furthermore, no single physical object, in the absence of other objects, time and governing laws, can actually cause anything. So no physical object could possibly qualify as an uncaused cause. Instead, we need something that is not contingent on any external context ... something that embodies existence in a pure and necessary way. It could not be reliant upon an external space-time, because that would again make it contingent upon a greater context, and therefore not an uncaused ultimate cause. But this seems to create an issue for any attempts to multiply uncaused causes. After all, if we have 2 red blocks on a table, or 3, or 11, how is it that we distinguish them in terms of their identity so as to establish that there are multiple blocks rather than just one? How do we determine that Block 1 != Block 2, and Block 7 != Block 10? Isn't it because they are separated in space and thereby capable of having distinct identity? If the blocks all occupied exactly the same space, could we even really say that there were multiple blocks? If reality were exhausted by the presence of one such block, could there be two blocks? If an uncaused cause must exist in the absence of any external context like space, time or governing laws then, in a literal sense, there cannot be any room for multiple uncaused causes, or any possibility for distinctions in the uncaused cause. An uncaused cause would absolutely "embody" and exhaust existence in the absence of any contingent effects it causes. HeKS
Violence to Scripture: part 4 of 4 As for the word “Yom,” God said many times in scripture, we must not add or subtract to words. Do you not ignore that advice? If we cannot keep to simple rules given for our good, what then? Paul gifted with the Holy Spirit, teaches the good rule, “nothing beyond what is written,” (1 Cor 4:6). Time and time again, God gives sound advice throughout scripture. Do not add or subtract to scripture (Deut 4:2), (1:3), (4:14), and, “Do not add to his words, or else he will rebuke you, and you will be found a liar.” (Prov 30:6) God said, “You must diligently observe everything that I command you; do not add to it or take anything from it” (Deut 12:32). The reason God is so adamant on this is because “The sum of your word is truth; and every one of your righteous ordinances endures forever. (Psalms 119:160) With respect, a major problem with your many examples of the word “Yom,” which I believe you cite in order to undermine the first impression of the plain and clear reading of the Genesis Sabbath Commandant, is you first must throw out scriptural standards and teaching. You appear to set “Yom” to any number to follow a God of long ages. I can only conclude by such a method (overtly straining at the word “Yom”) cast out are God given scriptural commands given for our good on how to act ethically and handle scripture containing the word of God. Ok, it is not a salvation issue. There again it is becoming one because the more and more people disregard the plain and first impression of scripture, which is not one of evolutionism in any form; eventually, we create a God of our own liking, as Darwin did, or none at all. Surely, your version of the word of God is that a day, as set down in Divine Law, leads to the belief that the Commandments harbours gross error, and hence untrue. In actual fact, “Yom” can mean as many days or fewer days as you want when boot strapped to the Big Bang Theory: everything bar what a plain reading says! God could have easily written He created over a long time and commanded us to rest every seven days and to remember the fact. However, He did not. Therefore, your interpretation cannot be true. Your reasoning surely leads to the conclusion that the sabbath is acutely imperfect as a Divine Law, a day is not a day when embedded by the finger of God as His Testimony and made Divine Law. Surely, that is a recipe for the eventual dismantling the kingdom of heaven. Yet God remembers the seventh day as the finality of the Creator act. Intended or not, I suggest, your version of God renders Him an amnesiac, and He must then at least bear false witness against the neighbour. The reliability and credibility of His word suspect. Let’s imagine BA77, quantum mechanics somehow transported you back to the stoning of the man for picking up sticks for firewood on the sabbath. You interrupt the crowed, stones in hand, saying: ‘I am from the future, where theory says, God did not create in six days, and the sabbath day is not a true or an exact day, therefore this act commanded by God is a murderous act and unjust.’ Phew! I will leave the rest to your imagination, remembering that the only big bang to consider in this case, is when Yahweh banged the heads of Miriam and Aaron together for believing they had a God given right to speak over Moses. Overall, in this context, I also suggest you render the Divine Law questionable and imperfect, therefore making unholy Divine Law (a meaning of holy, “exalted or worthy of complete devotion as one perfect in goodness and righteousness”), bearing in mind, a day in your version of events actually represents still an imprecise number of billions of year days. If God cannot count basic numbers and pass them on as perfect information, what is your version of God in this matter playing at! Yahweh said: “I call heaven and earth to witness against you today that I have set before you life and death, blessings and curses. Choose life so that you and your descendants may live” (Deut 30:19). God did not call Darwin or the Big Bang Theory as a witness for His defence. The heavens of the Big Bang Theory declare God wrong! Rather, ‘Our Father who art in heaven, who does not know how they go, who has been telling us a load of codswallop.’ What a way to encourage faith in the word of God! “The heavens are telling the glory of God; and the firmament proclaims his handiwork.” (Ps 19:1). More codswallop if the heavens do not declare He created in six days. Who is to judge with righteousness, accuracy, truth and holiness, when He said the heavens were created miraculously over four days? You may say, ‘I will only believe if God wrote in stone again the same statement.’ As Jesus said, “Blessed or those who have not seen but believe.” (Jn 20:29) Take note scholars of the power of the Holy Spirit of truth, sent to remind us of truth, including Divine Law: “Now when they saw the boldness of Peter and John and realized that they were uneducated and ordinary men, they were amazed and recognized them as companions of Jesus.” (Acts 4:13) “Jesus Christ is the same yesterday and today and for ever” (Heb 13:8). God changes not (Mal 3:6). Therefore, will God/Jesus on His return, come back and apologise to the big bang Judaeo-Christians, or go to a place or worship, to remember, the Father created in six days, through Him (Col 1:15-19). Bearing in mind, God created deliberately, in six days; created in such a manner because he knew of the falling away in pride of fallen knowledge today. A creation deliberately made to appear foolish to confound the wise (1 Cor 1:27). God does not lie, or fiddle the scriptural books. “God is not a human being, that he should lie, or a mortal, that he should change his mind. Has he promised, and will he not do it? Has he spoken, and will he not fulfil it?” (Num 23:19). How can He ever fulfil truthfully and accurately His word by the Big Bang Theory? He told us how long creation took, no matter what human calculations and measurements say. Can we measure God with our calculations and see Him through a radio-telescope or a microscope? Have we strict evidence that God played a part in the Big Bang. If we accuse God of being a liar, at least make sure our evidence is irrefutable before we meet God and we end up being rebuked. Have any of us seen the hand of God, as strict evidence He created by the Big Bang? Nevertheless, we have His word how long creation took. Present fallen natural scientific consensus is no match for the miraculous science and word of the Almighty. Yet, we know best. Anyway, BA77, I am glad you say to me you only “think” [my] “YEC interpretation does violence to the scripture overall.” Nevertheless, what a violent lot those who take the Creationist God at His Word! Ah well, God/Jesus did say, “I come not to bring peace but a sword” (Matt 10:34). More faith less theory. I hope I have fully answered your concerns, I for one could not be clearer. Sincerely, all the best, mw. mw
Violence to Scripture: part 3 of 4 The Ten Commandments do not contain science, they contain knowledge of a super scientific event, the knowledge of God to command miracles at will. Six-day creation was a super scientific event, the one big bang of miracles complete. Then natural law proceeded, as today. We cannot really on backward extrapolation which can never take account of a miracle to either prove God or disprove God. We take God at His word and that should be enough. Sadly, it is not. How long one of the six days of creation, the same length as the sabbath day, otherwise the sabbath becomes rather meaningless and makes God a murder and liar. God created to fit in with human activity and our need for rest. God stated as a Divine Law what six human days reflects when conjoined to a normal sabbath day. The Decalogue contains a replica test as given to Adam and Eve: a command containing a test of pure faith. A pure test of faith to believe in the Almighty by word alone. The tree of knowledge still resides in the Genesis Sabbath Commandment, this time, the world has almost fallen in pride, rejected is God information. Of course, we know we are too sophisticated in intelligence to fall for that old cherry of the Fall. For our good, God wills worship; remembered for honest truth and Almighty power, that He created in six days; accurate deserved worship for a monumental act of power. Clearly, at this moment in time, many are unable to do that. If He did not create in six days, He wants false worship. That is silly. Are you saying BA77, creation in six days is impossible, or that it is impossible by a theory which will never be proved by humanity? Or, is it more a matter of, “You of little faith” (Matt 6:8), when all things are possible with the Father (Mk 14:36). The truth of Yahweh sanctifies us (Jn 17:17), Jesus said so as the “truth” (Jn 14:6), who teaches He learned all in His Humanity from the Father. Believing God created in six days sanctifies us: it is only then that all scripture makes clear sense, easy to understand; but then expect stick and scoffing. It seems BA77, an outcome of your belief is that your version of God must do violence to us by literally commanding us, on pain of death as in one case, to believe He created in six days, when you strongly suggest the Law is not precise, rather, Divine Law is fallible by the standards of the Big Bang Theory. The Big Bang Theory becomes a golden standard, a god. A type of a wager BA77. God’s six-day Testimony against every word of every fallen scientist, who makes the Creator Saviour an inaccurate sinner to His Own Divine Law. Surely, on the basis of proof of an intelligent divine pattern of legal words spoken by God in Exodus, a real pattern of informational truth was given at Sinai. If we cannot see true informational intelligent design in Divine Law, how can we profess to be experts to see intelligent design? When you say to me, “I think your YEC interpretation does violence to the scripture overall,” I perceive you really mean, I do violence to your belief in consensus science and in relation to long ages and Hugh Ross? Was he by the way at Sinai, does he have knowledge of miracles or the power of God? He surely then is guessing; betting against the cut word of God, as you must also do. mw
Violence to Scripture: part 2 of 4 Sinai, not Genesis, is the major historic starting point of the Judaeo-Christian faith because rooted and linked is seven-day worship to a historically witnessed event. The following scripture speaks volumes of God’s plain and clear Testimony, “a sign forever:” _______________________________________________________ “The LORD said to Moses: You yourself are to speak to the Israelites: ‘You shall keep my sabbaths, for this is a sign between me and you throughout your generations, given in order that you may know that I, the LORD, sanctify you. You shall keep the sabbath, because it is holy for you; everyone who profanes it shall be put to death; whoever does any work on it shall be cut off from among the people. For six days shall work be done, but the seventh day is a sabbath of solemn rest, holy to the LORD; whoever does any work on the sabbath day shall be put to death. Therefore the Israelites shall keep the sabbath, observing the sabbath throughout their generations, as a perpetual covenant. It is a sign for ever between me and the people of Israel that in six days the LORD made heaven and earth, and on the seventh day he rested, and was refreshed.’ (Exodus 31:12-18) ________________________________________________________ That command is frighteningly plain: do a simple style Rorschach inkblot test on that pattern of divinely intelligent words Yahweh spoke. What immediate pattern do we see, “a sign forever” or a bad sign? A historical miraculous event directed by the finger of God, Lasered in stone tablets four times, pointing to Genesis and, in one sense, confirmed at the Transfiguration when Jesus spoke to Moses. The basis of Judaeo-Christian morality is the Ten Commandments. We agree on that. The Decalogue is the divine Testimony of the Holy Trinity; giving divine intelligent information; information which only the creator had true knowledge of. God surely gave true witness, the truth; accurate information to the letter. The Ten Commandments contain the witness statement of God on origins. Today, too many fallen scientists are on the road to agnosticism and atheism, having made a god out of the Big Bang Theory and Darwinism, while too many Christians feed the golden calf of evolutionism with shredded scripture. It appears BA77, in order to fulfil your belief in long ages, wrong in some measure is God: at least unclear and inaccurate; therefore, possibly giving highly misleading or contaminated information at Sinai, but set in a judgmental Divine Law. We have become embarrassed to believe God created in six days, and the command for us to remember creation took six days, every seven days. Heavens above, God as a tongue in his head (so to speak), and knows how to intelligently design a pattern of sound words reflecting a designer cosmos to aid us better to combat the battering ram of beguilement in spiritual warfare. The consequences of the elastication of Divine Law, is that God casts out his own moral laws. First he lied, and then he murders some Israelite in the wilderness for picking sticks on the Sabbath; for strictly not keeping to a major Divine Law. Is it not you who does violence to scripture on this point by making out God to be full of consensus scientific error. Accurate must be the judgement of God. We expect to be judged fairly and with accuracy. Therefore, was the stoning commanded by God unjust and inaccurate. If so, we may as well cast out the whole Ten Commandments, and your version of scripture and it is the word of God that needs saving by fallen theory! If one law is inaccurate and therefore verging on immoral, where then is the basis for morality, if one law is wrong, the whole batch is wrong by contamination. The Holy Trinity demonstrated time and time again in scripture the power to create from dead matter. To change six jars of water into mature wine. How old the wine, who determined its age? God wanted to create a mature cosmos, as a mature Adam and Eve. Surely BA77, you are not suggesting the existence of better knowledge than that written clearly by God in stone twice over? Well, yes you are. Has any creature ever created a universe in six days, clearly not. Do you have experience working miracles and know of their effects on data? Then why should I rely on your views before God’s? Surely, you cannot expect me to. Established is the New Testament on the Hebrew scripture (Matt 5:17-19); “his works were finished at the foundation of the world” (Heb 4:3). Therefore, I for one am not advising anyone to break a Commandment that incorporates a test of pure belief; that God created in six days. Surely, you do not say, at the beginning, the Big Bang Theory was a finished foundation. Too many are shoehorning scripture onto the Big Bang theory. The faith is dying because of lack of true faith in the word of God. Those who do so are scoffed at regularly by fellow Christians. How un-Christ-like, scoffing actually at the Divine Law of Jesus and the Father. It was Yahweh who was angered when he summoned together two people to bang their heads together when thinking God spoke to them above Moses. ______________________________________________________ “Suddenly the LORD said to Moses, Aaron, and Miriam, ‘Come out, you three, to the tent of meeting.’ So the three of them came out. Then the LORD came down in a pillar of cloud, and stood at the entrance of the tent, and called Aaron and Miriam; and they both came forward. And he said, ‘Hear my words: When there are prophets among you, I the LORD make myself known to them in visions; I speak to them in dreams. Not so with my servant Moses; he is entrusted with all my house. With him I speak face to face—clearly, not in riddles; and he beholds the form of the LORD. Why then were you not afraid to speak against my servant Moses?’ And the anger of the LORD was kindled against them, and he departed. When the cloud went away from over the tent, Miriam had become leprous, as white as snow. And Aaron turned towards Miriam and saw that she was leprous.” (Num 12:4-10) _______________________________________________________ Creation took six days; it is not a riddle. Plain words, simple, clear cut in stone. Historic in witnessed evidence. In this day and age, it appears that a gift of grace is needed to believe the impossible. Still, many accept the grace, and pay a price. Today, too many Christians take a pot shot at the Father to see if we can knock him off Sinai. Lemaître for one: all in a scientific manner of course and very respectable. mw
Violence to Scripture: part 1 of 4. You say, BA77, I did not answer you in my response at 184-187. Here is another go, though it was my aim to leave it at that due to having a lot on my plate at the moment; though I am obliged, it seems, to respond further. First, BA77; you ask a question, “do you think only YECs go to heaven?” What is a bright and scriptural person as you doing asking such a question? No doubt we will both hope to eventually meet in heaven. Nevertheless, you will remember, there are scripturally two ways to heaven: and there is always instant divine mercy, as the thief on the cross found. First, “And this is his command: to believe in the name of his Son, Jesus Christ, and to love one another as he commanded us.” (1 Jn 3:23) Second: “So Jesus said to them, ‘Very truly, I tell you, unless you eat the flesh of the Son of Man and drink his blood, you have no life in you. Those who eat my flesh and drink my blood have eternal life, and I will raise them up on the last day; for my flesh is true food and my blood is true drink. Those who eat my flesh and drink my blood abide in me, and I in them. Just as the living Father sent me, and I live because of the Father, so whoever eats me will live because of me. This is the bread that came down from heaven, not like that which your ancestors ate, and they died. But the one who eats this bread will live for ever.’ (John 6:53-58) However, ------------------------------------------------------------------------------------- You say BA77: “For the record, I still disagree with your YEC interpretation of the scripture and think that the best fit for the word ‘Yom’ is a long period of time. I think your YEC interpretation does violence to the scripture overall.” ------------------------------------------------------------------------------------- Is it not your version of God whom you make violent by having Him command murder on a person for picking sticks on the seventh day by making out that God did not create in seven days? ____________________________________________________ “When the Israelites were in the wilderness, they found a man gathering sticks on the sabbath day. Those who found him gathering sticks brought him to Moses, Aaron, and to the whole congregation. They put him in custody, because it was not clear what should be done to him. Then the LORD said to Moses, ‘The man shall be put to death; all the congregation shall stone him outside the camp.’ The whole congregation brought him outside the camp and stoned him to death, just as the LORD had commanded Moses.” (Numbers 15:32-36) _____________________________________________________ God/Jesus, personally ordered the stoning to death of a man. Surely, such an extreme penalty must highlight the supreme importance of a Divine Law, which was not to be broken by an act of work or an act of disbelief. Of course, God wants people saved, but heavens above BA77, the God of Sinai had just given the Divine Law in miraculous acts and daily provided miraculous acts to reinforce his power and protection, and people still disbelieved! His power and command to believe could not be more clear through acts of divine power. God means what He says, and acted so. The Father was the teacher of Christ in his Humanity. The Father gave Him the commandment what to say (Jn 12:49-50), Jesus being obedient to death, keeping all the Father’s commandments to the letter, as Christians should try to (Matt 5:17-19), and not by being a hypocrite and elasticating them to make six days equal to billions of days in order to please Big Bang theorists (ref Matt 15:3-9), hence making the Genesis Sabbath Commandment at Sinai doubtful and eventually of no effect. Still, we will be judged by the words of the Father (Jn 12:48). If He can’t count, God help us. The accuracy of divine law is of prime importance. Truth does not have degrees of accuracy, truth is the whole truth and nothing but the truth, not shades of truth to our liking, or lack of belief. Strict Divine Law is one reason for the Saviour. “But when the fullness of time had come, God sent his Son, born of a woman, born under the law, in order to redeem those who were under the law, so that we might receive adoption as children” (Gal 4:4-5). Why, did not Jesus in relation to divorce, adultery and lust, make a stricter law out of the Decalogue (Matt 5:28). Do we not now commit spiritual adultery with the Big Bang Theory and Darwinism against Divine Law, intellectually crucifying Divine Law and Christ, and doing our selves no good whatsoever? The law must be accurate or we have a Saviour Creator God who is inaccurate. What then His judgements? Did not Jesus say, we will be judged to the last farthing is paid (Matt 5:26). Surely, we expect to be judged accurately. How, by what God plainly said. You are making the case BA77, that the Genesis Sabbath Divine Law is doubtful, subject to further interpretation. Therefore, Divine Law is inaccurate: ‘I appeal Judge to be removed from hell, because your law is unclear.’ Satan would have a field day. However, Jesus teaches a valuable lesson in the wilderness against Satan: “But he answered, ‘It is written, “One does not live by bread alone, but by every word that comes from the mouth of God.”’ (Matt 4:4) Notice, Jesus said to Satan, “it is written.” And Jesus said it was hypocrisy to try to get around what is written, especially when written by God at Sinai (Matt 15:3-9). Therefore, BA77, no more of one implication to your statement, ‘I do violence to scripture by keeping to scripture.’ I have never heard such, as we shall see next. mw
HeKS,
I’d say we can even extend this principle out to additional levels. Anything that exists within a larger context is necessarily contingent upon its context. This is true whether the context is space, or time, or constraining laws, or variable motion, or probabilistic outcomes, or material components, or creative agency, or all of the above, or anything else. The entirety of the context that informs a physical object’s existence, state of being and activity would need to already exist, each part in proper relation to the other, before the object itself could exist. This would apply all the way back to the beginning of the universe itself and even any quantum regime that may or may not have existed at that point. Physicality, at all points, is contingent upon composition with respect to its own parts, the spacetime it occupies, and the laws that govern its behavior.
Yes, your extended analysis makes sense to me and points to a broader principle, perhaps something on the order that complexity implies dependence. Further, it shows that the logic which proves [a] a physical thing is always contingent is similar to the logic that proves [b] only one first cause is possible. StephenB
Origenes @216 If someone said that an electron is one thing and not a composite whole, I would first ask if, for him, an electron is an example of a thing or the definition of a thing. Then, I would remind him that I provided both a definition of a thing (composite whole) and an example (automobile). If he said that an electron was an example of a thing, I would ask him to provide the definition that informed his example. If he said it was the definition, I would ask him why it is so narrow that it excludes almost everything that most people would characterize as a thing, such as an automobile. Further, I would disclose my own understanding of an electron in the context of my own definition, that is, I would characterize the electron, nucleus, and atom as the basic raw materials which make up the parts and wholes in things. Finally, I would ask him to provide the context or the reason why he is providing a definition in the first place. For example, the reason that I provided my definition was to show that a "thing" cannot be a first cause. I would need a similar context from him. Until he disclosed this kind of information, I would consider his assertion to be meaningless. StephenB
StephenB @215, Suppose that someone claims that an electron is one thing and not a composite whole. What would be your response? Origenes
Origines
Can you please elucidate? Do you mean to say that every physical thing is infinitely “cuttable”? Or do you mean to say that every physical thing cannot exist distinct from laws, space and time and is therefor a composite whole?
All I mean to say is that there is one aspect of physical things --[they exist as composite wholes]-- that, when considered, will provide proof of their contingent nature. I am assuming that everyone already agrees with me that physical things do, indeed, exist as composite wholes. StephenB
StephenB
I have said that a physical thing cannot be the first cause. A physical thing is a composite whole, that is, a whole that is made up of parts. The important consideration is that the whole depends on the parts for its existence; the parts must exist before the whole can exist. Think of an automobile: The crankcase, engine, and carburetor must exist before the automobile can exist. That means that a prior cause must already be in place to form the parts into a whole. Thus, a physical thing must be contingent. By virtue of being a composite whole, it cannot be the first cause.
I'd say we can even extend this principle out to additional levels. Anything that exists within a larger context is necessarily contingent upon its context. This is true whether the context is space, or time, or constraining laws, or variable motion, or probabilistic outcomes, or material components, or creative agency, or all of the above, or anything else. The entirety of the context that informs a physical object's existence, state of being and activity would need to already exist, each part in proper relation to the other, before the object itself could exist. This would apply all the way back to the beginning of the universe itself and even any quantum regime that may or may not have existed at that point. Physicality, at all points, is contingent upon composition with respect to its own parts, the spacetime it occupies, and the laws that govern its behavior. HeKS
Eric Anderson @209 :
I’m certainly open to learning about whatever qualities we think must exist in an uncaused cause (so far, the description of that uncaused cause has been pretty lean, consisting primarily of a claim of being “simple”) ....
How about adding the quality "enormously powerful" to the list, since the First Cause created all matter and energy. And how about "intelligent", in order to explain the fine tuning of the universe and life? On the concept of "simple"... It is, at least to me, a gift that keeps on giving. It provides me with a deeper understanding of "being", e.g. in the sense that "simple" (oneness) is uncuttable (indestructible). Also the concept elucidates "freedom" and "autonomy", in the sense that "simple" cannot be caused. Origenes
StephenB, thank you for your comments. Regarding your claim that a physical thing is a composite whole, would you care to respond to my question in post #191? Origenes
Eric, you write,
A physical object being necessarily contingent at many levels does indeed appear to be the claim, yet I still feel some intellectual uneasiness accepting such a claim, as I haven’t heard a good reason why a physical object is necessarily contingent. Again, people seem to keep defining what is outside of our universe as having the non-characteristics of what is in our universe, but that becomes circular and doesn’t follow logically or as a matter of evidence.
I have said that a physical thing cannot be the first cause. A physical thing is a composite whole, that is, a whole that is made up of parts. The important consideration is that the whole depends on the parts for its existence; the parts must exist before the whole can exist. Think of an automobile: The crankcase, engine, and carburetor must exist before the automobile can exist. That means that a prior cause must already be in place to form the parts into a whole. Thus, a physical thing must be contingent. By virtue of being a composite whole, it cannot be the first cause. StephenB
Eric, Michael Augros, in his book “Who designed the designer,” makes my point using different words. Perhaps it will resonate with you. “Two first causes would have to share a common nature. They could thus be distinguished only by some addition to that nature in at least one of them. That distinguished addition, since the common nature is indifferent to it, would belong to its possessor as a cause. *(A distinguishing feature might be a god who has intelligence distinct from its, and added to, its self- existent nature. The distinguishing feature cannot be something trivial since it is the only way the two causes can be two and not one in the same thing. Their common nature exists with the distinctive feature in the one case and not in the other. The common nature is, therefore, in itself indifferent to the add-on and hence enters into a combination with it only through a prior cause, a combiner. Whichever of our first causes possesses this difference, and is distinguished by it alone therefore, has its distinct existence only through that prior cause having provided it with its distinctive feature). Therefore, the possessor of the distinguishing addition would not be a first cause after all. “ StephenB
HeKS @207: Thanks for the comments.
First, using different colors as an example of qualities suggests that you think a physical object can be uncaused, but a physical object is necessarily contingent at many levels. I suspect that StephenB was referring to qualities that would be deemed essential rather than superficial.
The example of color was just for simplicity's sake to wake us up as to the specific claim that was being made (namely, that we can't have uncaused causes with different characteristics, and ergo, the claim goes, we can only have one first cause). I don't care whether we talk about color, size, shape, non-corporeal aspects, spirit, personality or whatever. That we can't have two uncaused causes because they will be different from each other just doesn't follow in my mind. Nor, frankly, does it follow that they must be different from each other. More to the substantive point of your comment: A physical object being necessarily contingent at many levels does indeed appear to be the claim, yet I still feel some intellectual uneasiness accepting such a claim, as I haven't heard a good reason why a physical object is necessarily contingent. Again, people seem to keep defining what is outside of our universe as having the non-characteristics of what is in our universe, but that becomes circular and doesn't follow logically or as a matter of evidence. I'm certainly open to learning about whatever qualities we think must exist in an uncaused cause (so far, the description of that uncaused cause has been pretty lean, consisting primarily of a claim of being "simple") and how those qualities cannot exist in more than one uncaused cause. That might all be true, to be sure. But I don't see how it holds together as a matter of deductive logic.
Second, even if it turned out that it could be possible for more than one uncaused cause to exist, Occam’s Razor would prevent us from multiplying such causes unnecessarily.
Well, I don't feel any obligation to bow down to Occam's Razor in general, used as it often is to dismiss unpalatable arguments without addressing them on the merits. Even so, a rational application of Occam's Razor would depend on some kind of knowledge about the actual entity or entities in question. One cannot simply invoke Occam's Razor to win the argument by fiat. With no knowledge about whether there are, or should be, or might be, multiple uncaused causes, Occam's Razor simply operates as a rhetorical device to win the day. We can't rationally apply Occam's Razor unless we have some knowledge and basis for doing so. More importantly, even if we were to apply Occam's Razor to argue that one uncaused cause is better than two, that is a far weaker and far more tentative argument than claiming that there can only be one uncaused cause as a matter of logic. That is much of my point. Again, there may well be only one uncaused cause. I'm certainly open to that. But I haven't yet seen how that can follow as a matter of logic.
So unless evidence or argument could be brought forth necessitating multiple uncaused entities there would be no basis for postulating their actual existence merely because you think it might be logically possible.
See Querius @93. That alone is certainly more than enough to prevent Occam's Razor from being applied broad brush to this issue. Furthermore, as mentioned already, part of the very point I am making is about logical possibility, which some have argued does not exist. That is precisely part of what I am questioning. Eric Anderson
Eric, Yes, I'm aware of the different ways of measuring time, and you're actually illustrating my point. o Time is measured by something that cycles, such as the oscillation frequency of a Cesium atom, a light-dark cycle (I won't conclude it's a rotation cycle, although that's possible as well), or an orbital cycle. The standard is arbitrary. o A "day" is defined in Genesis as a single light-dark cycle. o The passage of time is influenced by velocity and gravity. I'm sure you're familiar with the twins paradox. This thought experiment was scientifically verified in 1971 with the Hafele–Keating experiment. o A "day" of time is not absolute. There's no way of determining how long that light-dark cycle is. It's simply "a day." Thus, it makes no sense referring to "a day of 24 hours" because an hour is not absolute either. o Since about 1908, spacetime (or Minkowski space) is recognized as a single manifold. This means that it equally makes no sense to refer to "a foot of twelve inches." Genesis was not intended to be a science book, but it is compatible with scientific thought. It doesn't include dismemberment and re-purposed body parts of gods and goddesses. The sun and moon are merely lamps, not deities. Hope this clarifies what I meant. -Q Querius
Eric, Just two quick points. First, using different colors as an example of qualities suggests that you think a physical object can be uncaused, but a physical object is necessarily contingent at many levels. I suspect that StephenB was referring to qualities that would be deemed essential rather than superficial. Second, even if it turned out that it could be possible for more than one uncaused cause to exist, Occam's Razor would prevent us from multiplying such causes unnecessarily. So unless evidence or argument could be brought forth necessitating multiple uncaused entities there would be no basis for postulating their actual existence merely because you think it might be logically possible. HeKS
StephenB: Thanks, Stephen, for your comment. I'm confused about your claim here:
There cannot be two uncaused causes. The very fact that they are different in some way (having different identities) indicates that one of them lacks some quality that the other has, meaning that its existence is contingent on something that lacks nothing—in which case it is disqualified from being the uncaused cause.
The fact that Cause B has a quality different than Cause A does not mean that Cause B's existence is contingent on something else, while Cause A's existence is uncaused. It simply doesn't follow. If one of the qualities of A is "red" and B, instead, has the quality "blue" it doesn't mean that B is somehow contingent, but A isn't. There is no principle of logic or practical reason that would preclude two identities from being uncaused. Now, if what you are making is a more narrow claim, namely that a specific Event X is associated with some particular Cause A, rather than Cause B, that well may be the case. But (i) it is a factual question what the cause of Event X is (it could be both A and B); and (ii) even the existence of a clear, complete, and comprehensive link between Cause A and Event X does not mean that Cause B does not exist -- only that it did not cause Event X. Eric Anderson
As to those who hold that Genesis should be taken not only literally in a linguistic sense, but as some kind of scientific treatise laying out the details of creation with precision ...
Hmm, maybe so. IMO, it's more like a historical description of what happened, when and broadly, how.
... inquiring minds presumably might want to know precisely which kind of “day” they are referring to.
I think in discussions like this "24 hour day" is more or less placeholder text for "however long it was then". Perhaps a full day was 20 hours long or 30 hours long, either way (from bornagain's quote):
"Genesis 1:5: “And there was evening and there was morning — the first day.” Here, the word does indeed mean [an x]-hour day."
:)
There cannot be two uncaused causes. The very fact that they are different in some way (having different identities) indicates that one of them lacks some quality that the other has, meaning that its existence is contingent on something that lacks nothing—in which case it is disqualified from being the uncaused cause.
Awesome stuff. :D Vy
Eric, Origenes, Thank you for your probing analysis and well-thought-out questions. There cannot be two uncaused causes. The very fact that they are different in some way (having different identities) indicates that one of them lacks some quality that the other has, meaning that its existence is contingent on something that lacks nothing—in which case it is disqualified from being the uncaused cause. StephenB
Eric Anderson @202,
EA:
SB: Yes. By physical thing, I mean a composite whole. ... All physical things I can conceive of are composite wholes.
Based on this, you conclude that the First Cause must be “simple,” whatever that means in this context.
Your question is addressed to StephenB, but I would like to offer my thoughts. The First Cause must be simple, because if it were the case that the First Cause consists of distinct parts, then those distinct parts are logically prior to the First Cause, which is incoherent. From this StephenB goes on to argue, if I understand him correctly, that the First Cause cannot be physical, since all physical things consist of distinct parts — are "composite wholes."
EA: On what basis can we assume that only one thing is eternal and self-sustaining and without cause? I agree there is a strong argument that there must be a “First Cause,” in the sense of an ultimate cause for whatever has brought about our existence. What I don’t understand is the assumption — usually unstated, never explained — that there can only be one such entity.
Excellent question. I would like to know as well. Origenes
Stephen B @188:
Yes. By physical thing, I mean a composite whole. I assume that this is also what my adversaries mean when they mistakenly posit a physical first cause. All physical things I can conceive of are composite wholes.
Based on this, you conclude that the First Cause must be "simple," whatever that means in this context. On what basis can we assume that only one thing is eternal and self-sustaining and without cause? I agree there is a strong argument that there must be a "First Cause," in the sense of an ultimate cause for whatever has brought about our existence. What I don't understand is the assumption -- usually unstated, never explained -- that there can only be one such entity. Eric Anderson
Querius @200:
From a physics point of view, ALL days have 24 hours, all hours have 60 minutes, all minutes have 60 seconds . . ."
Not really. No current calculations in astronomy view a day as being equal to 24 hours made up of 60-minute hours, and 60-minute seconds. The length of a day depends on whether we are talking about a solar day, a sidereal day, a mean (solar or sidereal) day, or any other number of nuances and permutations. It is true that for standard civil timekeeping (a political and practical convenience, rather than a scientific one), we treat our days as 24, 60-minute hours, made up of 60-second minutes. But then we have to adjust this convenient fiction with leap years; centennial, 400-year, and 1,000-year adjustments; as well as the occasional leap second, and on and on. As to those who hold that Genesis should be taken not only literally in a linguistic sense, but as some kind of scientific treatise laying out the details of creation with precision, inquiring minds presumably might want to know precisely which kind of "day" they are referring to. :) Eric Anderson
A couple of thoughts.
A day of 24 hours . . .
From a physics point of view, ALL days have 24 hours, all hours have 60 minutes, all minutes have 60 seconds regardless of how long or short they are in comparison to each other. According to Genesis, the periodic episodes of light and dark defined time. A single one of these periods defined the length of a day. In an absolute sense, the length of time in a day could be shorter or longer depending on your speed. Physicists and cosmologists now generally agree with the Bible that - The universe had a beginning, - Time is relative, and that 1,000 years could pass in a day. - That God stretched out the heavens. - That light existed before the sun. -Q Querius
Question: Do only YECs go to heaven? bornagain77
A brief reading of your quote in 195 leaves me laughing. This especially:
In Genesis 1 Moses says “and there was evening and morning the xx day”. Does the use of evening and morning indicate a sunrise and sunset for each creative day? First, let’s look at what evening and morning are not. They are not actual evening and mornings, as this requires a sunrise and sunset. According to young earth theory, the Sun was not created until Day Four, thus there could be no sunrise or sunset for the first three days of creation. However, God uses the terms evening and morning for those first three days. Therefore, they cannot be actual evenings and mornings.
Are you kidding me??! Revelation 21:23
And the city had no need of the sun, neither of the moon, to shine in it: for the glory of God did lighten it, and the Lamb is the light thereof.
Apparently God didn't know how long the sun He created needed to offer light to represent daytime. Pathetic.
When God refers to a large number, He uses picture stories, such as Abraham’s descendants being as numerous as the sand. Why does He do this? If God had said, “You will have millions of descendants,” Abraham would have asked, “What is a million?”
Awwwwwwww! Whoever wrote this article must be God's mouthpiece or His PA to know exactly what would have happened had God used a word invented in the 17th century in a conversation with Abraham. Silly God. How dare He use 'big' grammar and confuse those dunderheads, right?
When considering the creation, if we broke it down into days, that would be 5,000,500,000,000 days, or roughly 13.7 billion years. Do we need an account for each day of creation…of course not. God in His infinite wisdom, saw fit to tell us the creation story by breaking it down into creative segments, each of which was attributed to a specific creative act or acts.
Wow, just wow.
We need to give the early Hebrews of Genesis a break…they didn’t have calculators like we do!
Really? Wouldn't wanna stress their cavemen brains now would we?
Thus, the only evidence we have to accurately assess the age of creation is the creation itself. Since the rocks and stars say we are billions of years old, that must be the truth. This fits perfectly with a literal interpretation of Genesis, and an inerrant Bible, and does not impact any other Biblical doctrines.
Whatever makes you sleep well at night. Vy
@ba in 195, you do realize I could post 5 CMI articles in response to yours, right? Vy
Why I Reject A Young Earth View: A Biblical Defense of an Old Earth – Jonathan M. – 2011 Excerpt: If, therefore, it may be considered legitimate to take the seventh day as representative of a much longer period of time, then whence the mandate for supposing a commitment to interpreting the other six days as representative of 24-hour periods?
Considered legitimate by who based on what? Evodelusionists think it 'may be considered legitimate' to conflate adaptation, speciation and whatnot with evodelusionary probablymaybecouldness on a daily basis.
Fourth, there is the multiple-usage of the word “day” in Genesis 1.
You don't say? Kinda like this: 'The day I went to [insert place] was the first time I realized how long a day can seem to be.' Another You seem to think Jonathan has discovered some new unheard of knowledge here. 'Day' can clearly mean a myriad of things even in a single sentence.
Let’s take a look at the manner in which the word “day” is used in the Genesis 1 (up to 2:4) narrative alone: 1. Genesis 1:5a: “God called the light Day, and the darkness he called Night.” Here, “day” is contrasted with “night”: Thus, a 24-hour day is not in view, but rather “day” in the sense of “daytime” (i.e. 12 hours).
Shocka!!! Whuda thunk it?
2. Genesis 1:5b: “And there was evening and there was morning — the first day.” Here, the word does indeed mean a 24-hour day.
Ya think?
3. Genesis 2:3: “By the seventh day God had finished the work he had been doing; so on the seventh day he rested from all his work. 3 Then God blessed the seventh day and made it holy, because on it he rested from all the work of creating that he had done.” To this, I have already alluded — the key point here is the absence of “evening” and “morning”, which denotes all of the previous six days.
Grasping at straws. Big ones!
4. The correct rendering of the Hebrew with respect to Genesis 2:4 is “This is the account of the heavens and the earth in the day they were created, when the LORD God made the earth and the heavens.”
So? Vy
Old Earth Creation Science Word Study: Yom By Greg Neyman © 2007, Old Earth Ministries Published 16 March 2005 (This article can be freely copied and distributed, as long as it is unaltered and a link back to the original article appears on the page) The Hebrew word for “day” is the word “Yom.” Young earth creationists have always argued that the word used for the days of creation can only mean a 24-hour day. In this article, we will examine the uses of Yom in the Old Testament, and show that it can mean a wide variety of time periods. First, one must understand that the Hebrew language is not nearly as diverse as our English language. Whereas our vocabulary is around half a million, the Hebrew language has only 8,700 words. The French language, one of the poorest modern languages in vocabulary and the language of choice for diplomats, has just about 40,000 words or over 4 times the amount of words that Ancient Hebrew has. Many of the Hebrew words could be considered duplicates with only slight differences. Thus, words which contain multiple meanings are common. Such is the case with the word Yom. Hebrew Dictionaries Let’s start with the possible meanings of Yom; The Theological Wordbook of the Old Testament (1980, Moody Press) "It can denote: 1. the period of light (as contrasted with the period of darkness), 2. the period of twenty-four hours, 3. a general vague "time," 4. a point of time, 5. a year (in the plural; I Sam 27:7; Ex 13:10, etc.)." Strong's Exhaustive Concordance of the Bible (symbols omitted) from an unused root meaning to be hot; a day (as the warm hours), whether literal (from sunrise to sunset, or from one sunset to the next), or figuratively (a space of time defined by an associated term), [often used adv.]:--age, + always, + chronicles, continually (-ance), daily, ([birth-], each, to) day, (now a, two) days (agone), + elder, end, evening, (for)ever(lasting), ever(more), full, life, as long as (...live), even now, old, outlived, perpetually, presently, remaineth, required, season, since, space, then, (process of) time, as at other times, in trouble, weather (as) when, (a, the, within a) while (that), whole (age), (full) year (-ly), younger As you can see, Hebrew dictionaries attest to the fact that the word Yom is used for anywhere from 12 hours up to a year, and even a vague "time period" of unspecified length. Other Uses of Yom Day is not the only translation for the word Yom. Here are some other uses. Time It is interesting to note that in 67 verses in the Old Testament, the word Yom is translated into the English word "time." For instance, in Genesis 4:3, it says "And in process of time it came to pass, that Cain brought of the fruit of the ground an offering unto the Lord." In this instance, Yom refers to a growing season, probably several months. Again, in Deuteronomy 10:10, it refers to a "time" equal to forty days. In I Kings 11:42, it says "And the time that Solomon reigned in Jerusalem over all Israel was forty years." In this case, Yom translated as the word "time" is equivalent to a 40 year period. In Isaiah 30:8, it says "Now go, write it before them in a table, and note it in a book, that it may be for the time to come for ever and ever." In this case, Yom is equal to "forever." How long is forever? An infinite number of years...billions upon billions upon billons of years. If Yom can equal trillions of years here, then why not billions of years in Genesis? Year Four times in the Old Testament Yom is translated "year." In I Kings 1:1, "David was old and stricken in years..." In 2 Chronicles 21:19, "after the end of two years" and in the very next verse "Thirty and two years old." Finally, in Amos 4:4, "...and your tithes after three years." In each case, Yom represents years, not days. Age Eight times in the Old Testament Yom is translated "age." These range from sentences like "stricken in age," meaning old age (Genesis 18:11 and 24:1; Joshua 23:1 and 23:2), and other times it says "old age" (Genesis 21:2, Genesis 21:7). Genesis 47:28 refers to "the whole age of Jacob," therefore yom here refers to an entire lifetime. In Zechariah 8:4, it says old men and women will sit in the streets of Jerusalem, "each with cane in hand because of his age." Ago One time Yom is translated "ago." 1 Samuel 9:20 says "As for the donkeys you lost three days ago, ..." Always Four times yom is translated as "always," in Deuteronomy 5:29, 6:24, 14:23, and in 2 Chronicles 18:7. Always here can be interpreted as a lifetime...for instance, we are to keep the commandments of the Lord always (Deut. 5:29). Season Three times yom is translated "season." In Genesis 40:4, "...and they continued a season in ward." Again, in Joshua 24:7, "dwelt in the wilderness a long season," and in 2 Chronicles 15:3, "...a long season Israel hath been...". In each case yom represents a multi-month period. Chronicles When used in conjunction with the word dâbâr, yom is translated "chronicles" (27 times). Continually When used in conjunction with kôwl, yom is translated as "continually" (11 times). Once, in Psalm 139:16, it is translated continuance (without the kôwl). Ever Ever is used to represent a long period of time, such as in Deuteronomy 19:9, "to walk ever in his ways." Nineteen times Yom is translated "ever." The old testament uses "for ever" instead of the word forever. In sixteen cases of use of the word ever, for is placed before it, indicating a infinite period of time. I will not list them all (consult Strong's Concordance for a full listing) but here is an example. In Psalm 23:6, it says "Surely goodness and mercy shall follow me all the days of my life; and I will dwell in the house of the Lord for ever." Here Yom is translated as the final word of this verse, ever. Thus, Yom in this verse, and 16 others, represents eternity. Evermore In one instance, when yom is used in conjunction with kôwl, Yom is translated "evermore." Deuteronomy 28:29, "...and thou shalt be only oppressed and spoiled evermore;" thus representing either a lifetime or eternity. Word Usage in the Old Testament As you can see, Yom is used in a wide variety of situations related to the concept of time. Yom is not just for days...it is for time in general. How it is translated depends on the context of its use with other words. Yom in the Creation Account Even within the creation account, Yom is used to represent four different time periods. Genesis 1:5 "And God called the light Day, and the darkness He called Night." Here, Moses uses Yom to indicate a 12-hour period Genesis 1:14 "And God said, "Let there be lights in the firmament of the heaven to divide the day from the night, and let them be for signs, and for seasons, and for days, and years." Here, Moses uses Yom to indicate 24-hour days Genesis 2:4 "...in the day that the Lord God made the earth and the heavens." Here, Moses uses Yom to indicate the entire creative week. The fourth usage of Yom in the creation account is in the summary for each of the six creation days, "and there was morning and evening the first day". Yom is used to represent a finite, long period of time, usually either millions or billions of years. To show support for this, consider the uses of Yom by Moses. Moses Other Uses of Yom Moses, the author of the first five books of the Bible, and of Psalm 90, used Yom in many different ways. Genesis 4:3 "And in process of time it came to pass, that Cain brought of the fruit of the ground an offering unto the Lord." In this instance, Yom refers to a growing season, probably several months. Genesis 43:9 "...then let me bear the blame for ever." Here, Moses uses Yom to represent eternity Genesis 44:32 "...then I shall bear the blame to my father for ever." Again, Moses uses Yom to represent eternity Deuteronomy 4:40 "...that thou mayest prolong thy days upon the earth, which the Lord thy God giveth the, for ever." Here Yom represents a physical lifetime Deuteronomy 10:10, "Now I stayed on the mountain forty days and nights, as I did the first time,..." Here, Yom is a "time" equal to forty days. Deuteronomy 18:5 "...to stand to minister in the name of the Lord, him and his sons for ever." Again, Yom is translated as eternity Deuteronomy 19:9 "...to love the Lord thy God, and to walk ever in His ways..." Here, Yom represents a lifetime. As long as we live we are to walk in his ways As you can see, Moses used the word Yom to represent 12-hours, 24 hours, the creative week, forty days, several months, a lifetime, and eternity. Common Young Earth Arguments To get around the obvious conclusion that Yom in Genesis 1 can mean millions of years, young earth theorists have come up with several arguments, none of which is supported by common Hebrew grammatical rules according to Hebrew experts (such as Dr. Walter Kaiser). These rules were created by Hebrew language experts who are young earth creationists to begin with, thus their viewpoint is obviously biased. They have a specific agenda they are trying to prove, and thus cannot be objective. Ordinals/Cardinals Young earth creationists say that whenever Yom is used with an ordinal or cardinal number (1st, 2nd, 1,2, etc) that it always represents a 24 hour day. However, this is not true. In Zechariah 14:7-9, the "one day" refers to a period of time when the Lord shall be king over the earth. In other places, some say that Isaiah and Hosea have numbers with the word day which are figurative (External Link). Some young earth theorists, including Jonathan Sarfati in his book Refuting Compromise, have addressed this verse in Zechariah an Hosea. Although his argument sounds impressive, you have to recognize it for what it is...he is arguing for his young earth agenda, thus any rules that he espouses must be examined by true Hebrew scholars who are impartial. Hebrew scholars do not recognize this fabricated rule.1 What Sarfati thinks is not important...what is important, as Dr. Walter Kaiser points out, is the intentions of the author. We should not create rules that support our own agendas, but should strive to understand the author's intended meaning outside of rules. Evening/Morning Construction In Genesis 1 Moses says "and there was evening and morning the xx day". Does the use of evening and morning indicate a sunrise and sunset for each creative day? First, let's look at what evening and morning are not. They are not actual evening and mornings, as this requires a sunrise and sunset. According to young earth theory, the Sun was not created until Day Four, thus there could be no sunrise or sunset for the first three days of creation. However, God uses the terms evening and morning for those first three days. Therefore, they cannot be actual evenings and mornings. We are left with only one option. The words for Evening and Morning can only represent the beginning and ending of the creative period, and not actual sunrise and sunsets. Scripture itself sets this pattern for us. Morning and evening are used figuratively in Psalm 30:5, Psalm 49:14,15, Psalm 90:6. Thus, the evening and morning of creation can mean the start and end of the creative process that is attributed to that creation period. Young earth advocates counter that traditionally, church fathers have always held that sunrise and sunsets do not constitute a day, and they accepted the sun creation on Day Four with no hint of the first three days being anything other than 24-hour days. For instance, Sarfati in Refuting Compromise mentions Luther and Calvin (page 84-86). However, Luther and Calvin did not have the means of modern science at their disposal. At the time, geocentricity was still accepted! Don't fall into the trap of following the teachings of our church fathers. For more, read Church Fathers. Literal/Figurative Argument This argument says that you cannot use a word figuratively until after you have used it literally (see this Answers in Genesis article). The author gives two examples, which appear to be correct and follow this rule. However, is this rule valid? I see no reason to suppose that it is. You have to be careful with young earth claims about biblical interpretation methods. Again, they will invent rules that support their cause, when there is no basis for their rule in Hebrew. In this case, it makes no difference which order the word Yom appears in, i.e. literal before figurative or vice versa. Yes, these are the first words of the Bible, but they are not the first words of mankind. All the time from Adam to Moses, men were speaking in their own languages, thus the literal interpretation via spoken language would already have been established. There was no need to suppose a literal/figurative structure. If God's Creation Was Billions of Years Old... If God's creation was billions of years old, how would He have written the creation account in Genesis? One thing is certain...God is good at telling us exactly what we need to know. When God refers to a large number, He uses picture stories, such as Abraham's descendants being as numerous as the sand. Why does He do this? If God had said, "You will have millions of descendants," Abraham would have asked, "What is a million?" When considering the creation, if we broke it down into days, that would be 5,000,500,000,000 days, or roughly 13.7 billion years. Do we need an account for each day of creation...of course not. God in His infinite wisdom, saw fit to tell us the creation story by breaking it down into creative segments, each of which was attributed to a specific creative act or acts. We need to give the early Hebrews of Genesis a break...they didn't have calculators like we do! One must also consider that time with God has no meaning. To Him, 10 billion years is like a day. Thus, it is no problem for God to put billions of years into one of His days. Dr. Hugh Ross puts it best in his determination that the frame of reference for creation is the surface of the earth. Genesis 1:2 puts the witness of creation on the surface. But who is witnessing these events? It is God himself. During the first 5.99 days of creation, God is the only one present. Thus, human time does not matter...no humans were there to witness the passage of time. What matters is how God sees time! Thus, a billion year day is only a passing moment in God's eyes. The creation account is written in such a manner for all people to understand it. The issue is not how long creation took...the issue is that God did it, and that's all that matters in the end. Conclusion With such a wide usage of the word Yom for many different time periods, it cannot be claimed that Yom in the Old Testament only represents a 24-hour period. During the creation account alone, Yom represents four different time periods. Rules of Hebrew, created by young earth Hebrew scholars, are invalid. Because of their biased position, they are trying to prove their own agenda. Since humans did not witness creation, our own concept of a 24-hour day does not apply. The only thing that matters is God's concept of time. Thus, the only evidence we have to accurately assess the age of creation is the creation itself. Since the rocks and stars say we are billions of years old, that must be the truth. This fits perfectly with a literal interpretation of Genesis, and an inerrant Bible, and does not impact any other Biblical doctrines. 1 Television Show and Transcript, "Are the Genesis Creation Days 24 Hours or Long Periods of Time," The John Ankerberg Show, 2005. http://www.oldearth.org/word_study_yom.htm
Question: Do only YECs go to heaven? Supplemental notes:
Special and General Relativity compared to Heavenly and Hellish Near Death Experiences https://www.youtube.com/watch?v=TbKELVHcvSI&list=PLtAP1KN7ahia8hmDlCYEKifQ8n65oNpQ5&index=1 Albert Einstein vs "The Now" of Philosophers and "The Now" of Quantum Mechanics https://www.youtube.com/watch?v=dwyHUxoKWNM&list=PLtAP1KN7ahia8hmDlCYEKifQ8n65oNpQ5&index=3 'In the 'spirit world,,, instantly, there was no sense of time. See, everything on earth is related to time. You got up this morning, you are going to go to bed tonight. Something is new, it will get old. Something is born, it's going to die. Everything on the physical plane is relative to time, but everything in the spiritual plane is relative to eternity. Instantly I was in total consciousness and awareness of eternity, and you and I as we live in this earth cannot even comprehend it, because everything that we have here is filled within the veil of the temporal life. In the spirit life that is more real than anything else and it is awesome. Eternity as a concept is awesome. There is no such thing as time. I knew that whatever happened was going to go on and on.' In The Presence Of Almighty God – The NDE of Mickey Robinson – video (testimony starts at 27:45 minute mark) https://www.youtube.com/watch?v=s66DchGhhD0 'When you die, you enter eternity. It feels like you were always there, and you will always be there. You realize that existence on Earth is only just a brief instant.' Dr. Ken Ring - has extensively studied Near Death Experiences 'Earthly time has no meaning in the spirit realm. There is no concept of before or after. Everything - past, present, future - exists simultaneously.' - Kimberly Clark Sharp – Near Death Experiencer 'There is no way to tell whether minutes, hours or years go by. Existence is the only reality and it is inseparable from the eternal now.' - John Star - NDE Experiencer
bornagain77
@ba in 192, you claim it's the YECs interpretation of 'Yom', not the OECs in their varied forms from theistic Darwinist to the less popular framework hypothesis champions, that are responsible for "violence to the scripture overall". OK. Your assumption that YECs base their interpretation of creation week as normal days on the presence of 'Yom' is obviously a strawman. It is of course based on several things but also on this:
Naturally this lead to a debate regarding the word yom, which I defended should be properly translated as “day” by its context and plain reading. I explained that every other place in scripture which contained the combination of yom and certain qualifiers such as numbers, or morning and evening, etc., universally rendered yom as a normal day. Moreover, I alleged that God’s creative week formed the basis of the Jewish Sabbath—6 days of work plus 1 day of rest.
But of course you knew that already. I'd love to know which of the following verses, when considered in light of what YECs actually based their interpretation of 'Yom' as used in the creation, you're referring to when you say "what about all the times Yom is clearly used referring to a long period of time?": ------------ 'First day' - 22 occurences 'Second day' - 13 exact matches 'Third day' - 44 occurences 'Fourth day' - 7 exact matches 'Fifth day' - 4 exact matches 'Sixth day' - 6 exact matches 'Seventh day' - 45 occurences ------------ A cursory look at them doesn't indicate that there's a magical difference in meaning of 'third day' in the Bible and 'third day' in everyday life. You're free to ignore the Genesis verses (those ones seem to give several OECs nightmares). Let's not forget there's "evening and morning" in Genesis. :D EDIT: Just went through your previous posts and saw your Jon. M., Hugh Ross etc. quotes and I find it stunning that theistic Darwinists are more honest when it comes to the meaning of 'day' in Genesis when read plainly without (Darwinian) pseudoscientific presuppositions. Vy
I still disagree with your YEC interpretation of the scripture and think that the best fit for the word ‘Yom’ is a long period of time. I think your YEC interpretation does violence to the scripture overall.
Shocka!!!
To flip your main defense on its head, If the Bible means a literal 24 hour day every time the word Yom is used in scripture then what about all the times Yom is clearly used referring to a long period of time?
Except of course no YEC says that, ever. Even in the English language, 'day' doesn't always mean a "a literal 24 hour day". It's sad your arguments against YEC on this site tend to be riddled with strawmen. And not even original ones at that.
Are we to somehow, to be consistent Biblical purist, to force fit those square pegs of long periods into your round holes of 24 hours so as to fit your preferred interpretation? Or is it only on this one hill that you want to die for your preferred YEC interpretation of Yom in the Bible?
Even the theistic Darwinists are honest enough to admit that Genesis is referring to normal days if you don't hold any presuppositional mystical meaning of what "first day, second day ... sixth day" should mean here and here:
It is apparent that the most straightforward understanding of Genesis, without regard to the hermeneutical considerations suggested by science, is that God created the heavens and the earth in six solar days, that man was created on the sixth day, and that death and chaos entered the world after the fall of Adam and Eve, and that all fossils were the result of the catastrophic deluge that spared only Noah’s family and the animals therewith
Next time, try to make the projection a little less obvious. Vy
mw, I did not see your responses to me at 184-187. and For the record, I still disagree with your YEC interpretation of the scripture and think that the best fit for the word 'Yom' is a long period of time. I think your YEC interpretation does violence to the scripture overall. i.e. To flip your main defense on its head, If the Bible means a literal 24 hour day every time the word Yom is used in scripture then what about all the times Yom is clearly used referring to a long period of time? Are we to somehow, to be consistent Biblical purist, to force fit those square pegs of long periods into your round holes of 24 hours so as to fit your preferred interpretation? Or is it only on this one hill that you want to die for your preferred YEC interpretation of Yom in the Bible? Question, do you think only YECs go to heaven? That is not a 'gotcha' question, but is a question aimed at focusing on what is truly the hill worth dying on in scripture. Did the Apostles Really Die as Martyrs for their Faith? By Sean McDowell http://magazine.biola.edu/article/13-fall/did-the-apostles-really-die-as-martyrs-for-their-f/ bornagain77
StephenB: All physical things I can conceive of are composite wholes.
Can you please elucidate? Do you mean to say that every physical thing is infinitely "cuttable"? Or do you mean to say that every physical thing cannot exist distinct from laws, space and time and is therefor a composite whole? Origenes
StephenB @188 I just dropped in here and saw your post and found it interesting as daveS and KF and I have been having a very similar discussion about very similar points over in the Quotes of the Day thread with respect to the characteristics of a necessary being. I agree with all your points here. HeKS
Querius @93:
What is intriguing to think about is that since our consciousness, which seems to have the power of collapsing wavefunctions, does have a small amount of transcendent control, it might actually belong to the extra- super- existence outside our universe! Maybe our spirits do sit in heavenly places!
Yes, interesting indeed! The nature of consciousness is one of the great open questions in all of existence. Eric Anderson
Origines
If by “physical thing” you mean “physical composite whole”, then I agree.
Yes. By physical thing, I mean a composite whole. I assume that this is also what my adversaries mean when they mistakenly posit a physical first cause. All physical things I can conceive of are composite wholes.
The first cause cannot depend for its existence on internal parts, because it must be simple.
Right, though I would express it the other way: The first cause must be simple because it cannot depend for its existence on internal parts.
At the bottom of post #97 I argue that there cannot be something external to the first cause. From this I conclude that the first cause must be outside space, time and laws. I would appreciate your comment
. I fully agree. So much so, that I will express the same point using different words, which means that your statement is true from any perspective. The first cause cannot depend on anything external to itself. It is, by necessity, self-existent. Thus, it is prior to all other things, which necessarily depend on it. Put another way, the self-existent being is being, which means that it gives being to other things which can only receive it and would not otherwise have it. Thus, the self-existent being is necessary and everything else is contingent. The first cause must be prior to (and the cause of) anything outside of it. The law-giver must be outside the physical law; the Creator must be outside the creation; the painter must be outside the painting. StephenB
(Part 4 of 4) On the extent of the cosmos, Eric in the OP wrote: ________________________________________________ “Also a point worth considering. Again, that the universe had a cause does not mean that the universe is all that there is or that the cause has no attributes similar to the attributes of our universe. It is probably fair to say – definitionally so – that the cause of the universe exists outside the universe, but that does not speak directly to other attributes of that cause.” _____________________________________________ How huge the “All” in comparison to God? While in a near death state, anchoress, Julian of Norwich, England, c1373, at a time before the division into Catholic and Protestants, Jesus showed in the palm of her hand, that the size of everything created was no bigger than a hazel-nut. Julian, an unlettered woman at the time, became the first English women to write a book, and in English. She was a creation mystic. https://archive.org/details/revelationsofdiv00juliuoft Concerning human rest, she wrote: ______________________________________________________ “It needeth us to have knowing of the littleness of creatures and to hold as nought all-thing that is made, for to love and have God that is unmade. For this is the cause why we be not all at ease of heart and soul: that we seek here rest in those things that are so little, wherein is no rest, and know not our God that is All-might, All -wise, All-good. For He is the Very Rest.” [Revelations of Divine love, recorded by Julian, Anchoress at Norwich, A. D. 1373, Ed., Grace Warrack (London: Methuen, 1901), Ch. V, pp 10-11.] _____________________________________________________ Reflect, “to love and have God that is unmade.” Are we saying God waited 13.7 billion years, and before that for eternity in order to give us an uncreated particle of Himself? That is, the spirit, the higher part of the soul, which matter cannot create. (At least four venerated women in the Catholic Church have recorded given mystical writings of creation in six days. Of course speaking as a Catholic, we are not bound by any). However, BA77, for those collecting small sticks on the Sabbath, you appear to claim the Sabbath never ends because it is not covered by two limiting factors; evening and morning, therefore, six days is not six days. What a leap of another type of faith! Surely, a murderer and untrustworthy is made of your God and mine when moulding a divine law into theory. A truthful, just, accurate sentence only results from true law. If God did not create in six days, invalidated is His whole law-making system and the credibility of the whole of the Decalogue in doubt. What a destructive theology results from merging big bang theory to divine law! The man, God ordered to be stoned (Num 15:32-36) could only be justly condemned if the law was true. Nevertheless, how easy scripture is when we simply believe God verbatim. As in the beginning, so at the end; changed “in the twinkling of an eye” (1 Cor 15:51) we will be. Are we to say against such scripture, but ah; God needs billions of years to judge everyone and evolve them instantly into the likeness of His ascended body? Excuse me, but codswallop. You or I cannot even work a miracle at will, never mind prove or disprove a miracle, or let alone understand a cosmic miracle and its effects on data. We have never worked one miracle in six days in our life, and never will. In an intelligent pattern of words cut by God into the nature of Sinai stone, six means six. It is not even coded. Now we convince ourselves, needed is a PhD and additional hypotheses to work out six does not mean six. God help us! If we guide people that big bang cosmology equates to scriptural “truth,” by quoting Titus 1:2, then we cannot object when theistic evolutionism dress Darwinism in Titus 1:2. Surely, we have a God-given faith, original and in His own handwriting. These are my final comments on the matter to a God/cosmos encompassing OP. However, BA77, you may want to make further comments. No doubt, agreeing to disagree on the matter. All the best. mw mw
(Part 3 of 4) Of course, in the first instance, the Bible is not a book of science. However, it is a book of truth. It is a book of super science, of recorded super scientific miracles. It is a book of God’s word verbatim Personal, in the Flesh Personal and through people chosen for the job. ______________________________________________________ A point is, “Scripture teaches that the creation is cursed (Gen. 3:17—19, Rom. 8:20—22), but Scripture itself is ‘God-breathed’ (2 Tim. 3:15—17). So how can a cursed creation interpreted by a fallible methodology of sinful humans determine how we interpret the perfect, unfallen Word of God?” _______________________________________________________ An imaginary question BA77; if you or I were at Sinai, just after God had given the Ten Commandments, along with Hugh Ross, and the Pontifical Academy of Sciences; then on the Sabbath we all started collecting intellectual sticks (arguments, theories and thoughts), that God did not create as He said and wrote. What next? Surely, there would have been divine justice for rebellion, idolatry, profaning the sabbath, and for delivering a slap in the face to both God and Moses? Or should we have had first a trip up Sinai? Or at least asked Moses to go up on our behalf, but even that would go against us for obvious reasons. Still, who; me a rebel, beguiled – never; ref Jeremiah 7:1-11, ‘We are safe!’ Yet, God provided two food miracles a day for six days a week, but none on the seventh. And, two miraculous pillars to guide the Israelites; one pillar of fire by night and one pillar of cloud by day. Each dividing evening and morning. Clearly, He can count to six. A mass of miracles occurring continuously for forty years. How easy for infinite power to create the moon to guide by night and the sun by day. Give me faith as boundless as eternal time space of the Timeless “I am.” mw
(Part 2 of 4) You mention Hugh Ross, and in relation to long age creationism. Here are few references to balance some beliefs and opinions of Hugh Ross: http://creation.com/the-dubious-apologetics-of-hugh-ross http://creation.com/expose-of-the-genesis-question-by-hugh-ross http://creation.com/the-heavens-declare-a-different-story You quote: _____________________________________________________ Why I Reject A Young Earth View: A Biblical Defense of an Old Earth – Jonathan M. – 2011 Excerpt: “If, therefore, it may be considered legitimate to take the seventh day as representative of a much longer period of time, then whence the mandate for supposing a commitment to interpreting the other six days as representative of 24-hour periods?” _____________________________________________________ Should that not read, ‘Why I Reject a Divine Law.’ What a leap of faith BA77. The point is, neither OEC or YEC, but that first, God has not given us corrupt divine laws; if six really does not mean six, how may we rely on anything God says, long or short? If we do not believe heavenly things, how can we believe earthly things that Jesus says, as He said (Jn 3:12). The Christian ‘mandate,’ “believe every word out of the mouth of God” (Matt 4:4). Jesus/God was the Word speaking to Satan. However, the seventh day. “The Sabbath was made for man, not man for the sabbath.” (Mark 2:27). As for, “Our Father who art in heaven...” Meaning, at rest in heaven. Therefore, on earth, to enter into His rest may be a spiritual aim on earth, but eternal rest is of and in God. Sound is the closure of creation in six days as God asked us to be remember the event every seven days from Sinai, that He created in six days. The sabbath every seventh day, is closed by an act of God at Sinai in law, and in truth at the creation. Almost as if needed was a perpetual divine memorial commemorating such a majestic miracle. Are you saying, God is asking us to remember a fix, that He actually did not create in six days, that His holy sabbath was never a day, therefore never truly Holy? That would surely be blasphemy, wrapped in the beguiling big bang theory, and with some string left over from Darwinism. Besides, “And to whom did he swear that they would not enter his rest, if not to those who were disobedient?” (Heb 3:18) Does the seventh day continue as you try to shoehorn seven days into billions of years? Surely, based is the first sabbath on unclear elastication, why not remember the sabbath once every billion years or so? How can it be a sin to miss the sabbath if God never truly had a first sabbath? It would be God who is the sinner for stretching the truth beyond all recognition. However, the scholarly Dr Jonathan Sarfati responds; “Ross claims:” ____________________________________________________ ‘Each of the six creation days closes with the refrain: “There was evening, and there was morning,” then the day’s number. The statement suggests that each day had a start time … and an end time. However, the refrain is not attached to the seventh day. Its closure is missing. ‘… its absence from the account of the seventh day can be taken as a meaningful hint: the day has not ended.’ From this, Ross has concluded that the other creation days could be long ages. However, the systematic theologian, Dr Douglas Kelly, responded to the same argument from Ross in his book Creation and Change as follows: ‘To say the least, this places a great deal of theological weight on a very narrow and thin exegetical bridge! Is it not more concordant with the patent sense of the context of Genesis 2 (and Exodus 20) to infer that because the Sabbath differed in quality (though not—from anything we can learn out of the text itself—in quantity), a slightly different concluding formula was appended to indicate a qualitative difference (six days involved work; one day involved rest)? The formula employed to show the termination of that first sabbath: “And on the seventh day God ended His work which He had made; and He rested on the seventh day from all His work which He had made” (Genesis 2:2) seems just as definite as that of “and the evening and the morning were the first day”.’ (15) Ross also argues that Hebrews 4:1–11 teaches ‘that the seventh creation day began after the creation of Adam and Eve, continues through the present, and extends into the future.’ However, again Ross repeats an argument rebutted by Van Bebber and Taylor. (16) Hebrews never says that the seventh day of creation is continuing to the present; it merely says that God’s rest is continuing. If someone says on Monday that he rested on Saturday and is still resting, it in no way implies that Saturday lasted until Monday. (17) Kulikovsky carefully analyses the grammar of Hebrews 4 and concludes: ‘The “rest” of Hebrews 4 clearly refers to the Kingdom of God … Nowhere in the text is it equated with the seventh day of creation, nor is there any grammatical or contextual data suggesting any such equation.’ (18) http://creation.com/expose-of-the-genesis-question-by-hugh-ross ____________________________________________________ mw
(Part 1 of 4): - “Truth,” (Titus 2:1), the Big Bang and BA77 #169: “mw, I disagree with you that your interpretation of the word ‘day’, i.e. “Yom”, in Genesis is correct.” _____________________________________________________ Nevertheless, BA77, you have somewhat equated scriptural “truth,” (Titus 2:1) with the big bang theory, or placed “truth” in that area, #149. Possibly, therefore, you should have written: -------------------------------------------- “Father the Almighty I disagree with You that your interpretation of the word ‘day’, i.e. ‘Yom’, in Genesis is correct, which you have Personally written in ‘solid’ stone at Sinai, the only scripture ever written by the Hand of God, and the holy of holy scripture, which You sealed as an eternal commandment, set in an everlasting divine truth and law, which your Son died keeping to save us.” ------------------------------------------- It seems that the only place where six does not mean six is when God said six means six in His own handwriting through Moses. Are you saying, what God has written at Sinai; recorded and witnessed by Moses, and which law Jesus fulfilled (Matt 5:17-19) and who spoke to Moses at the Transfiguration, that we now know better than divine law? The big bang theory is a better explanation than what God wrote as divine law? Or, do you explain/teach the breaking of a commandment, that six does not mean six when Jesus teaches divine law is unalterable (Matt 5:18), down to the dot? Will the real “truth” and “six” in scripture stand up, please! No doubt, if we climb the heavenly Mount Sinai we will find the Commandments: The Ark of the Covenant in heaven (Rev 11:19), in which the divine law, Lasered by the finger of God, was once carried with utmost respect on pain of death. From heaven, Jesus spoke to St John of the importance in spiritual warfare of keeping the Commandments: ______________________________________________________ “Then the dragon was angry with the woman, and went off to make war on the rest of her children, those who keep the commandments of God and hold the testimony of Jesus.” (Rev 12:17) ______________________________________________________ Do you not think, BA77, that if God was not true and accurate in speech, Satan could be kept in Hell? Satan, second only to God in dialectics. That, on inaccurate judgement and divine law, Satan can be justly jailed, or we deserve heaven? If the Word is unclear; if six means any number we can think of to suit the big bang theory; rendered unjust would be divine laws, unclear befogging sign posts. There can only be one truth. There can only be one God. If you say QM is almost totally accurate, how accurate is God when He states a truth in divine law, and by being truthful, expects and commands us to reflect truth also being that is one of the Ten Commandments? Divine justice is equal to divine truth absolute. The accuracy of the word of God is total. The Word is perfectly accurate, that is why it is the basis of all judgment and truth. If we effectively justify our self for disfiguring and basically aborting a clear divine law for an unclear and questionable theory, all divine law will eventually lose credibility. If we try to shoehorn the big bang into the mouth of God, then we as professing Christians, may as well shred scripture and feed it every atheist and agnostic going. However, God must be accurate at judging down to the last farthing (Lk 12:59). If six does not really mean six, perhaps the last farthing does not really mean the last farthing. Perhaps there is no hell after all: all hail the big bang theory. Let’s see: against the big bang, hardly any scoffers. Against Darwinism, increasingly a number of scoffers. Then there a few that still believe that God created as He said in six days. Today, that is really being scoffed at (James 4:4). Ah well. mw
jdk, Thanks for the reply. I think I agree with all your responses/criticisms. A few somewhat scattershot replies to you and WLC: I'm also somewhat open to the concept of one interval/event, although like you (I gather) I find it just as natural to speak of isolated instants of time. It's not clear to me why one must necessarily be logically prior to the other. The notion that "one’s ability to specify certain points, like the halfway point along a certain distance, does not imply that such points actually exist independently of our specification of them" (from the WLC quote) also strikes me as odd. I guess he is saying that if I specify a line segment (in Euclidean geometry, for example), that doesn't also implicitly specify its midpoint, along with an infinite number of other points obtained by repeated bisection? It appears to me to be an obvious dodge to avoid "actual" infinities. Going back to your #177:
So I think WLC’s objection that if we tried to traverse all the points by sub-dividing and sub-dividing, which would create only a potential infinity, doesn’t actually fit the situation of just moving through space and time.
In my view, this is really the crux of the matter. Suppose we are talking about the motion of a particle P from point A to point B (which I assume WLC will grant "exist" to the extent required to do physics). It seems that WLC denies that the corresponding set of events "P is at point x_i" exist until someone goes back and retraces the trajectory of P; that is, these sets of events require the participation of a mind of some sort. To be fair to WLC and Sinclair, I'll have to get around to reading more about their positions. They have a nice 100-page overview of the Kalam argument in the Blackwell Companion to Natural Theology with some actual physics that addresses many of the issues that come up in these threads. In fact, something just caught my eye as I was skimming through it:
It is not the concept of causality which is incompatible with an infinite series of past events. Rather the incompatibility, as we have seen, is between the notion of an actually infinite number of things and the series of past events. The fact that causality has nothing to do with it may be seen by reflecting on the fact that the philosophical arguments for the beginning of the universe would work even if the events were all spontaneous, causally unconnected events.
daveS
StephenB @178,
[a] We have already established that the first cause must be causeless. That is obvious. Even our adversaries agree with that. [b] From [a] we can conclude that the first cause is simple, that is, it is not a composite whole. Put another way, it cannot consist of a part/whole relationship. [c] The reason for [b] is that for any material thing, which is a composite whole, the whole depends on the parts for its existence, and the parts must exist before the whole exists.
I agree. The first cause is explanatory fundamental. If parts explain the first cause, then the first cause is not the first cause. The first cause must be one and indivisible and cannot be a composite material whole.
[d] No physical thing can exist eternally because its parts would have to exist before the whole exists.
If by “physical thing” you mean “physical composite whole”, then I agree.
The physical whole, you will recall, depends on its constituent parts.
Indeed, the physical composite whole, depends for its existence on its constituents parts. The question left is: can there be a non-composite physical thing without parts.
Yet the first cause cannot depend on anything.
The first cause cannot depend for its existence on internal parts, because it must be simple. However, there are two options left for dependence: it can depend on itself or on something external to itself. At the bottom of post #97 I argue that there cannot be something external to the first cause. From this I conclude that the first cause must be outside space, time and laws. I would appreciate your comment.
[e] Thus, the first cause must be immaterial. That leaves either an abstract idea or a personal agent. Since abstract ideas cannot cause anything to happen, the first cause of everything must be a personal agent.
To this, I would like to add that a material first cause cannot cause anything to happen, because the first cause exists outside space, time and (natural) laws. Origenes
To dave: I read most of the paper by Puryear that you linked to. It was interesting, and I get what he is saying. The position I am describing is one of the Priority of the Parts, although I have never read about the distinction that he makes between that and the Priority of the Whole. I'm open, however, to the idea that there is just one interval of time and just one event, and that the subdivisions we make are just our anthro-centric way of conceptualizing and abstracting reality, not an accurate reflection of reality itself. I sort of refer to this idea when I say that the change of the entire state of the universe from one moment to the "next" (continuous?) moment is an event, although we experience sub-events because of the limitations of our perceptions. When discussing some of these ideas in the infinity threads a few months ago, I mentioned a "God's" eye view: to God (especially perhaps the more mystical God of Eastern religions), the entire state of the universe at any moment is a unified whole which proceeds any work on our part to subdivide into sub-events. This is analogous to WLC argument that an interval of time is a whole piece, and it is only in our minds that it can be divided into smaller pieces, leaving us with always a finite number of sub-intervals of time, not the infinitely dense number if one advocated for the Priority of the Parts. Very interesting stuff. jdk
I just got back home and re-read my post: it's supposed to say "certain sets of non-linear differential equations", not "different" equations. jdk
Further thoughts, a little hasty, but I've been thinking about these things for a month or so. As I mentioned in a reply to johhnyb recently, I've just read Chaos by James Glieck, and have read parts Fractals and Chaos by Peitgen et al, which has actual math and other details. The key idea here is recursive functions. For instance, the Mandelbrot set is formed by taking each point in the complex plane, applying a function to it, then applying the same function to the result, and so on indefinitely. The "boundary" of the set exhibits infinitely dense complexity: between any two points there is a whole world of complexity and pattern similar to, but not identical to, other areas of the plane, irrespective of scale. A discrete example is Conway's Game of Life. The starting generation g0 is a pattern of discrete points (squares in the real x-y plane), and there are a simple set of rules that decide whether "live" cells die and/or new one are born as you pass to the next generation g1. Again, the rules are applied recursively, so the original pattern goes through a life cycle that cannot be predicted at all from the original pattern: you just have to step through the generations to see what happens. In both these cases, it took computers to actually explore these situations, because you have to have a fast way of doing all the calculations Chaos theory began with the study of weather, and branched into many other fields, especially turbulence. (This history is what Gleick's book is about.) Chaos theorists discovered that if you recursively explore certain sets of non-linear different equations (which can be beginning models of real-world phenomena), you find regions where extremely small variations in initial conditions lead to vastly different historical paths as the system plays itself out. In an important sense, these systems lead to, in johnnyb's word, describable but not predictable systems. Now, to apply this to the topic at hand. Consider the state of the universe at any moment, frozen in time, so to speak. Now consider also that there are a certain set of rules that govern how various parts of the universe relate to each other (the four fundamental forces, etc.) If we apply those rules, the next moment of the universe is different, and constitutes the beginning point of the next application of the rules. So the action of the universe as it moves through time can be thought of as a vast recursive function. Therefore, it can be reasonably assumed that at various scales it displays chaotic behavior in the mathematical sense. Since quantum fluctuations are probabilistic and can add minute changes to the world, chaotic sensitivity to initial conditions means that ultimately the flow of the universe is not predicable nor determined. (And beyond that, just the general chaotic behavior that results from non-linear recursitivity creates a non-determined universe.) Two references to other posts: Denise just posted a news item on a conference on Time in Cosmology, and Lee Smolin said,
What is real is “the process by which future events are generated out of present events,” he said at the conference.
Yes to that. Also, in johhnyb's opening post at https://uncommondescent.com/intelligent-design/describable-but-not-predictable-modeling-non-naturalistic-causation/ johnny says we should not assume that reality must conform to popular notions of causation. That is exactly true. Traditional physics models the world with smooth, non-chaotic functions such as trig waves, exponential functions, etc, where you can plug time t into a function and get a predictable result for some time in the future. However, chaos theory shows us that if we think of the recursive nature of causation as the world proceeds moment-by-moment, causation is a much more complicated subject that can lead to chaotic results that can't be solved analytically for future moments in time, nor can be predicted with certainty in some cases. jdk
Eric, thanks for your response. You write,
I’m just not convinced that the force of pure logic leads us to that conclusion. Much less to a conclusion that the cause of the universe was the First Cause, or to the still further conclusion about the identity of the First Cause.
Well then, let me convince you, because it is surely the case that the First cause of the universe (or the ultimate cause of the cause) cannot be material. [a] We have already established that the first cause must be causeless. That is obvious. Even our adversaries agree with that. [b] From [a] we can conclude that the first cause is simple, that is, it is not a composite whole. Put another way, it cannot consist of a part/whole relationship. [c] The reason for [b] is that for any material thing, which is a composite whole, the whole depends on the parts for its existence, and the parts must exist before the whole exists. [d] No physical thing can exist eternally because its parts would have to exist before the whole exists. The physical whole, you will recall, depends on its constituent parts. Yet the first cause cannot depend on anything. [e] Thus, the first cause must be immaterial. That leaves either an abstract idea or a personal agent. Since abstract ideas cannot cause anything to happen, the first cause of everything must be a personal agent. StephenB
Hi Dave - nice to hear from you again. The paper looks interesting, although I'll not read it now. My quick reply is this: First, we probably can't know whether time and space are truly continuous or not, but if assume they are and model them with the real number line, then as one moves from point or time A to point of time B, one traverse an actual infinity of points or moments. So I think WLC's objection that if we tried to traverse all the points by sub-dividing and sub-dividing, which would create only a potential infinity, doesn't actually fit the situation of just moving through space and time. The important distinction here is that points and moments are dimensionless, and infinitely dense, not finite (no matter how small). jdk
jdk/aleta, This question of the number of events in an interval of time is quite thought-provoking. I'm in agreement with you on the matter. WLC's objection to an actual infinite number of such events is weak, IMO. He claims that while we can specify a potentially infinite number of such events, they might not actually exist independently of our specification. FWIW, here's some of what he has to say:
Sometimes it is said that we can find concrete counterexamples to the claim that an actually infinite number of things cannot exist, so that Premise (2.11) must be false. For example, Walter Sinnott-Armstrong asserts that the continuity of space and time entails the existence of an actually infinite number of points and instants (Craig and Sinnott-Armstrong 2003, p. 43). This familiar objection gratuitously assumes that space and time are composed of real points and instants, which has never been proven. Mathematically, the objection can be met by distinguishing a potential infinite from an actual infinite. While one can continue indefinitely to divide conceptually any distance, the series of subintervals thereby generated is merely potentially infinite, in that infinity serves as a limit that one endlessly approaches but never reaches. This is the thoroughgoing Aristotelian position on the infinite; only the potential infinite exists. This position does not imply that minimal time atoms, or chronons, exist. Rather time, like space, is infinitely divisible in the sense that division can proceed indefinitely, but time is never actually infinitely divided, neither does one arrive at an instantaneous point. If one thinks of a geometrical line as logically prior to any points which one may care to specify on it rather than as a construction built up out of points (itself a paradoxical notion), then one’s ability to specify certain points, like the halfway point along a certain distance, does not imply that such points actually exist independently of our specification of them.
(WLC & James D. Sinclair, The Blackwell Companion to Natural Theology, pp 112--113) BTW, Stephen Puryear has a critique here. daveS
Eric writes,
I’m not sure I know what you mean by physical events,
My point from #163. If each state of the universe and each moment of time is an event, and if time flows continually, then there are an infinite number of events in any interval of time, just as there are an infinite number of points on the real number line in any interval. If "event" does not mean the same as "state at a moment of time", I wonder how one would go about clearly defining what an "event" is. And otherwise good post by Eric. jdk
Another approach is to consider the nature and origin of wavefunctions, which make the existence of "material reality" dependent on observation and measurement by humans and their tools. Before there was observation and measurement, reality was simply probabilities. One would assume some Entity was required to supply some level of observation and measurement to collapse some proportion of the wavefunctions to material reality. That humans share this ability with the Entity seems remarkable. -Q Querius
StephenB @119: I apologize for the late response, as I have been completely out of pocket for a little over a week.
I am wondering if [Eric] agrees with me that an infinite number of physical events is logically impossible. If so, it seems that we have an unassailable proof for God’s existence, given the fact that a finite universe logically requires an immaterial, personal, self-existent first cause.
I'm not sure I know what you mean by physical events, as opposed to something physical existing for infinity. One could presumably have the latter without the former. In either case, let's assume that you are right and that an infinite number of physical events is an impossibility. I don't think that necessarily leads us to an unassailable proof for God or against the materialist position. For one thing, the materialist is not basing his position on an infinite number of physical events, so the argument would not directly refute his position. On the other hand, if we are focused on the infinite regress issue, then the question still boils down to whether the First Cause can be something physical. The materialist will continue to claim that it can be. Nor do I think observations about the finite nature of the universe are helpful in that regard. After all, as I have outlined in detail above, most materialists certainly hold to the Big Bang, to a beginning to the universe, to a finite state. Furthermore, when we carefully distinguish between the cause of the universe and the First Cause, the finite nature of our universe becomes really neither here nor there. The upshot is that, ultimately, I think we have to fall back to an analysis that is more evidentiarily practical, though less watertight logically. Namely, is there good reason to think that a purely physical First Cause could produce the universe as we know it, or does the universe exhibit indicia of an intelligent First Cause? At the end of the day, we are making a design inference about the origin of the universe. I happen to think the design inference is reasonable and that it is better than the materialistic alternative by a long shot. I'm just not convinced that the force of pure logic leads us to that conclusion. Much less to a conclusion that the cause of the universe was the First Cause, or to the still further conclusion about the identity of the First Cause. Eric Anderson
Job 26:10 He marks out the horizon on the face of the waters for a boundary between light and darkness. Job 38:4-11 “Where were you when I laid the foundations of the earth? Tell me if you have understanding. Who determined its measurements? Surely you know! Or who stretched a line upon it? To what were its foundations fastened? Or who laid its cornerstone, When the morning stars sang together, and all the sons of God shouted for joy? Or who shut in the sea with doors, when it burst forth and issued from the womb; When I made the clouds its garment, and thick darkness its swaddling band; When I fixed my limit for it, and set bars and doors; When I said, ‘This far you may come but no farther, and here your proud waves must stop!"
As well as the universe having a transcendent beginning, thus confirming Genesis 1:1, the following recent discovery of a 'Dark Age' for the early universe uncannily matches up with the Bible passage in Job 38:4-11.
For the first 400,000 years of our universe’s expansion, the universe was a seething maelstrom of energy and sub-atomic particles. This maelstrom was so hot, that sub-atomic particles trying to form into atoms would have been blasted apart instantly, and so dense, light could not travel more than a short distance before being absorbed. If you could somehow live long enough to look around in such conditions, you would see nothing but brilliant white light in all directions. When the cosmos was about 400,000 years old, it had cooled to about the temperature of the surface of the sun. The last light from the "Big Bang" shone forth at that time. This "light" is still detectable today as the Cosmic Background Radiation. This 400,000 year old “baby” universe entered into a period of darkness. When the dark age of the universe began, the cosmos was a formless sea of particles. By the time the dark age ended, a couple of hundred million years later, the universe lit up again by the light of some of the galaxies and stars that had been formed during this dark era. It was during the dark age of the universe that the heavier chemical elements necessary for life, carbon, oxygen, nitrogen and most of the rest, were first forged, by nuclear fusion inside the stars, out of the universe’s primordial hydrogen and helium. It was also during this dark period of the universe the great structures of the modern universe were first forged. Super-clusters, of thousands of galaxies stretching across millions of light years, had their foundations laid in the dark age of the universe. During this time the infamous “missing dark matter”, was exerting more gravity in some areas than in other areas; drawing in hydrogen and helium gas, causing the formation of mega-stars. These mega-stars were massive, weighing in at 20 to more than 100 times the mass of the sun. The crushing pressure at their cores made them burn through their fuel in only a million years. It was here, in these short lived mega-stars under these crushing pressures, the chemical elements necessary for life were first forged out of the hydrogen and helium. The reason astronomers can’t see the light from these first mega-stars, during this dark era of the universe’s early history, is because the mega-stars were shrouded in thick clouds of hydrogen and helium gas. These thick clouds prevented the mega-stars from spreading their light through the cosmos as they forged the elements necessary for future life to exist on earth. After about 200 million years, the end of the dark age came to the cosmos. The universe was finally expansive enough to allow the dispersion of the thick hydrogen and helium “clouds”. With the continued expansion of the universe, the light, of normal stars and dwarf galaxies, was finally able to shine through the thick clouds of hydrogen and helium gas, bringing the dark age to a close. (How The Stars Were Born - Michael D. Lemonick) http://www.time.com/time/magazine/article/0,9171,1376229-2,00.html History of the Universe - Timeline Graph Image http://www.der-kosmos.de/pics/CMB_Timeline300_gr.jpg
bornagain77
A period of darkness and light, as initially defined in Genesis marks, the beginning of time. One cycle is defined as "a day." So without having another measuring device, how long is this day? Subdividing the "day" won't help. -Q Querius
corrected links:
,,, 'And if you're curious about how Genesis 1, in particular, fairs. Hey, we look at the Days in Genesis as being long time periods, which is what they must be if you read the Bible consistently, and the Bible scores 4 for 4 in Initial Conditions and 10 for 10 on the Creation Events' Hugh Ross - Latest Scientific Evidence for God's Existence – video 56:14 minute mark https://youtu.be/d4EaWPIlNYY?t=3374 Hugh Ross - Testing RTB's (Old Earth) Creation Model against Young Earth Creation and Naturalism – 2015 video – 40:00 minute mark https://youtu.be/iIohXcdxrNM?t=2398 Seven Days That Divide The World (John Lennox) – video https://www.youtube.com/watch?v=0FmO2XKMe6g
bornagain77
mw, I disagree with you that your interpretation of the word 'day', i.e. "Yom", in Genesis is correct.
,,, 'And if you're curious about how Genesis 1, in particular, fairs. Hey, we look at the Days in Genesis as being long time periods, which is what they must be if you read the Bible consistently, and the Bible scores 4 for 4 in Initial Conditions and 10 for 10 on the Creation Events' Hugh Ross Science and Creation - Dr Michael Strauss - video (at approx 17:00 minute mark the old earth interpretation of Genesis 1 is discussed) https://youtu.be/EZJozX3sbE0?t=1017 Dr. Hugh Ross appeared on Fox News with Lauren Green to discuss Science and the Book of Genesis. (Long Days vs. 24 hour Days) http://video.foxnews.com/v/3633724402001/does-science-support-the-book-of-genesis/#sp=show- Hugh Ross - Testing RTB's (Old Earth) Creation Model tested against Young Earth Creation and Naturalism - 2014 video https://www.youtube.com/watch?v=jlYzYMDpTwY Seven Days That Divide The World (John Lennox) – video https://www.youtube.com/watch?v=6Y-AGFfKZFM Why I Reject A Young Earth View: A Biblical Defense of an Old Earth - Jonathan M. - 2011 Excerpt: If, therefore, it may be considered legitimate to take the seventh day as representative of a much longer period of time, then whence the mandate for supposing a commitment to interpreting the other six days as representative of 24-hour periods? Fourth, there is the multiple-usage of the word “day” in Genesis 1. Let’s take a look at the manner in which the word “day” is used in the Genesis 1 (up to 2:4) narrative alone: 1. Genesis 1:5a: “God called the light Day, and the darkness he called Night.” Here, “day” is contrasted with “night”: Thus, a 24-hour day is not in view, but rather “day” in the sense of “daytime” (i.e. 12 hours). 2. Genesis 1:5b: “And there was evening and there was morning — the first day.” Here, the word does indeed mean a 24-hour day. 3. Genesis 2:3: “By the seventh day God had finished the work he had been doing; so on the seventh day he rested from all his work. 3 Then God blessed the seventh day and made it holy, because on it he rested from all the work of creating that he had done.” To this, I have already alluded — the key point here is the absence of “evening” and “morning”, which denotes all of the previous six days. 4. The correct rendering of the Hebrew with respect to Genesis 2:4 is “This is the account of the heavens and the earth in the day they were created, when the LORD God made the earth and the heavens.” http://crossexamined.org/why-i-reject-a-young-earth-view-a-biblical-defense-of-an-old-earth/
bornagain77
Part 2 of 2: “Truth” and Big Bang codswallop? We are always in danger of being puffed up by unprovable godless consensus knowledge. Time and time again, God gives sound advice throughout J-C scripture. Do not add or subtract to scripture, (Deut 4:2), (1:3), (4:14), and, “Do not add to his words, or else he will rebuke you, and you will be found a liar.” (Prov 30:6) Heavens above, we basically added two new books to prove disbelief in such scripture: Big Bang Theory and Darwinism! Still, who cares that we may get the plagues for adding and subtracting or even our name removed from the tree of life (Revelation 22:18-19). The charges in such scripture are serious, given as good guidance and for our benefit. Of course God is divine mercy. Of course, we may be at different distances from the kingdom of heaven. Some going even in the wrong direction. God said, “You must diligently observe everything that I command you; do not add to it or take anything from it” (Deut 12:32). The reason God is so adamant on this is because “The sum of your word is truth; and every one of your righteous ordinances endures for ever. (Psalms 119:160) Jesus clearly teaches a valid lesson when faced with the beguiler, destroyer, liar, murderer, thief; all from the beginning, and prince of this world. Jesus/God, answered Satan, ‘It is written, “One does not live by bread alone, but by every word that comes from the mouth of God”’ (Matt 4:4). Not, we live by every word from the mouth of Darwin, Hubble and every big banger going. Jesus said “scripture cannot be annulled.” (Jn 10:35) “Annulled”? We have destroyed scriptural validity through belief in the Big Bang and Darwinism. New idols hold sway over us! Jesus used the word “hypocrites” (Matt 15:3-9) for those justifying themselves that they need not keep a commandment to the letter or tittle. The Big Bang Theory is now made a justifiable reason to disbelieve divine law. In this case, we are dangerously led to believe that we are no longer under a divine law which commands us to believe God created in six days, as He wrote in stone, and said at Sinai Concerning divine law, Jesus, as “the truth,” said: _____________________________________________________ ‘Do not think that I have come to abolish the law or the prophets; I have come not to abolish but to fulfil. For truly I tell you, until heaven and earth pass away, not one letter, not one stroke of a letter, will pass from the law until all is accomplished. Therefore, whoever breaks one of the least of these commandments, and teaches others to do the same, will be called least in the kingdom of heaven; but whoever does them and teaches them will be called great in the kingdom of heaven. (Matt 5:17-19) ______________________________________________________ Jesus spoke in obedience to the Father (Jn 12:49-50), keeping all divine law, therefore six means six, otherwise, He did not keep the law verbatim: our salvation is unreliable, as the Word becomes unreliable if we believe in consensus science. Divine law is central to the teaching of “truth” and salvation. “But when the fullness of time had come, God sent his Son, born of a woman, born under the law, in order to redeem those who were under the law, so that we might receive adoption as children.” (Gal 4:4-5). How could Jesus redeem a flawed divine law, as the big bang theory makes divine law totally corrupt! Equally bad; made a murderer and liar by Christians would be a God of the big bang theory. Yes, a murderer no less, as He instigated under divine law, the death penalty for anyone found working on the sabbath. God/Jesus, personally ordered the stoning of a man for collecting sticks on the Sabbath (Num 15:32-36). Therefore, as the Big Bang Theory covers 13.7 billion years, with no Sabbaths, the God of Sinai would be totally out of order, the man would have been under a trumped up charge! Theistic Big Bang theology makes total nonsense out of divine law. However, God changes not (Mal 3:6). Still, when Jesus returns, He cannot be a born-again Big Banger and a Darwin “Origin” Thumper. Nevertheless, thank you BA77 for all your excellent references against Darwinism. In this case: - more faith, less science. mw
Part 1 of 2: “Truth” and Big Bang codswallop? Thanks BA77 for your comments, sorry for a slow response. However, to recap, this conversation started when you made a scriptural Judaeo-Christian statement, # 149, and in relation big bang cosmology. _____________________________________________________ You quoted, “in the hope of eternal life, which God, who does not lie, promised before the beginning of time,” (Titus 1:2). # 160 my reply: “Therefore, in relation to big bang cosmology, the Word from the beginning is untrue, and which He wrote and spoke at Sinai: that on His honour, He created in six days. In other words, His Word on the Creation is codswallop and gobbledegook; worse, He asked us to remember the same every seven days. Praise the Lord and pass the self-destructive ammunition.” ____________________________________________________ My point: if the Big Bang Theory is true, Divine Law is untrue. You kindly replied # 161, _____________________________________________________ “First off, the Big Bang is far more supportive of Theism than you seem to believe.” _____________________________________________________ However, as big bang evolutionism is not even remotely supportive of what God clearly said in a divine law. As you scripturally invoke God as truth, your point is surely meaningless. In relation to the big bang theory, you have based your reply on consensus science, I will base my reply on Judaeo-Christian scripture alone. I respectfully suggest you infer, in an oblique way, the big bang theory is superior to the Word embedded in a divine law at Sinai? “What is truth?” as Pontius Pilate asked, facing Jesus, divine truth (Jn 18:38). Jesus said of the Father, “Sanctify them in the truth; your word is truth.” (Jn 17:17). Jesus said “I am” (Jn 8:58), “the truth” (Jn 14:4). Hence the same God who at Sinai give the Ten commandments, stated He created in six days. We do well to digest the following perpetual Holy statement: ______________________________________________________ “The LORD said to Moses: ‘You yourself are to speak to the Israelites: ‘You shall keep my sabbaths, for this is a sign between me and you throughout your generations, given in order that you may know that I, the LORD, sanctify you. You shall keep the sabbath, because it is holy for you; everyone who profanes it shall be put to death; whoever does any work on it shall be cut off from among the people. For six days shall work be done, but the seventh day is a sabbath of solemn rest, holy to the LORD; whoever does any work on the sabbath day shall be put to death. Therefore the Israelites shall keep the sabbath, observing the sabbath throughout their generations, as a perpetual covenant. It is a sign for ever between me and the people of Israel that in six days the LORD made heaven and earth, and on the seventh day he rested, and was refreshed.’ When God finished speaking with Moses on Mount Sinai, he gave him the two tablets of the covenant, tablets of stone, written with the finger of God.” (Exod 31:12-18) _____________________________________________________ The testimony of God about the Creation was given at Sinai, a sound historic event in Judaeo-Christian history. With respect, BA77, you simply revert back to some default position of belief in long ages (and readily accepted by atheism), by dismantling in Genesis what a “day” means; when qualified is the meaning of “day” by a beginning and end of a day. However, my stance begins at Sinai, not Genesis, but on divine law given at Sinai, in the Genesis Sabbath Commandment in the Ten Commandments, all of which are unalterable, and cannot be modified or tampered with, unless by divine command. Of Moses, Yahweh said: ______________________________________________________ “and he is entrusted with all my house. With him I speak face to face—clearly, not in riddles; and he beholds the form of the LORD. Why then were you not afraid to speak against my servant Moses?’ And the anger of the LORD was kindled against them, and he departed. (Num 12:49) ______________________________________________________ Therefore, there can be no riddles in divine laws. God could easily have said creation took Him ages and ages and ages. What then the sabbath? Further, He could have said He created all from one, but He did neither. Important are the following. There are datum’s given by God/Jesus and Paul, on how to stay true to inspired scripture (2 Tim 3:16); come hell or high water, so to speak. Pauls states: _____________________________________________________ “I have applied all this to Apollos and myself for your benefit, brothers and sisters, so that you may learn through us the meaning of the saying, ‘Nothing beyond what is written’, so that none of you will be puffed up in favour of one against another.” (1 Cor 4:6) _____________________________________________________ Wise Paul, who somehow transcended physical space-time to be taken up into the eternal time and space of heaven (2 Cor 12:3). mw
P.S. again. Edit isn't working for me it Chrome. The sentence above "I don’t think I have “defined” time, other than to ask whether if lows continually" should read "I don’t think I have “defined” time, other than to ask whether it flows continuously." jdk
Hmmm. I've been thinking about this sentence of yours, Stephen:
Your definition of time cannot be linked to causality.
I don't think I have "defined" time, other than to ask whether if lows continually, in the mathematical sense, or if it moves in very small discrete jumps, only when some quantum action happens. What you mean by my "definition of time"? Also, what you you mean by "linked to causality?" Causal chains happen in time: as time flows, however it does, states of the universe, in whole or in part, at one moment cause states to exist at further moments. It seems to me that time and causality are irrevocably linked, and I don't see what I have said that would imply otherwise. So can you explain what you think I have said about time that would not link it to causality? jdk
Editing doesn't seem to be working. My last paragraph above should read, "I think these questions are pertinent to your statement to me that “your definition of time is not consistent with real infinity. You are still promoting potential infinity. Your definition of time cannot be linked to causality.” jdk
On the one hand, I know my point has nothing to do with the first cause issue, so it is a distraction from that issue. However, you have also said a number of times that "infinity cannot be instantiated in physical reality," and my response does relate to that point. Some questions for you: 1. Do you believe that time flows continuously? For instance, between 1:00 pm today and 1:01 pm today, did time flow continuously? 2. What exactly constitutes an "event" in physical reality? Is every moment of time a separate event, or do events take a finite amount of time? And if the latter, how long? How do you measure how long an event takes? 3. Is every moment the cause of the next moment? That is, to tie the two questions above together, does causality flow continuously, or does causality take a finite amount of time to occur? And if so, how long, and how do you measure the amount of time to it takes A to cause B? I think these questions are pertinent to you statement to me that "our definition of time is not consistent with real infinity. You are still promoting potential infinity. Your definition of time cannot be linked to causality." jdk
jdk
Good – I think we’ve got these points cleared up between us.
Almost, but not quite. Your definition of time is not consistent with real infinity; it implies potential infinity. Real infinity cannot be instantiated in nature. And, as you seem to realize, it cannot be linked to causality, so there was no real reason to introduce the idea in the first place. StephenB
mw, First off, the Big Bang is far more supportive of Theism than you seem to believe. Indeed, if anyone should be utterly opposed to the implications of the Big Bang it should be the Atheist himself not the Theist. Mainly, there actually was a beginning to all time-space energy-matter!
The best data we have [concerning the Big Bang] are exactly what I would have predicted, had I nothing to go on but the five books of Moses, the Psalms, the bible as a whole. Dr. Arno Penzias, Nobel Laureate in Physics - co-discoverer of the Cosmic Background Radiation - as stated to the New York Times on March 12, 1978 “Certainly there was something that set it all off,,, I can’t think of a better theory of the origin of the universe to match Genesis” Robert Wilson – Nobel laureate – co-discover Cosmic Background Radiation “There is no doubt that a parallel exists between the big bang as an event and the Christian notion of creation from nothing.” George Smoot – Nobel laureate in 2006 for his work on COBE "Now we see how the astronomical evidence supports the biblical view of the origin of the world. The details differ, but the essential elements in the astronomical and biblical accounts of Genesis are the same: the chain of events leading to man commenced suddenly and sharply at a definite moment in time, in a flash of light and energy." Robert Jastrow – Founder of NASA’s Goddard Institute – Pg.15 ‘God and the Astronomers’ Big Bang Theory - An Overview of the main evidence Excerpt: Steven Hawking, George Ellis, and Roger Penrose turned their attention to the Theory of Relativity and its implications regarding our notions of time. In 1968 and 1970, they published papers in which they extended Einstein's Theory of General Relativity to include measurements of time and space.1, 2 According to their calculations, time and space had a finite beginning that corresponded to the origin of matter and energy."3 Steven W. Hawking, George F.R. Ellis, "The Cosmic Black-Body Radiation and the Existence of Singularities in our Universe," Astrophysical Journal, 152, (1968) pp. 25-36. Steven W. Hawking, Roger Penrose, "The Singularities of Gravitational Collapse and Cosmology," Proceedings of the Royal Society of London, series A, 314 (1970) pp. 529-548. http://www.big-bang-theory.com/
Secondly, the term 'day' in Genesis does not convey a strictly 24 hour period. In fact, the term 'day' in Genesis, in a short space, conveys varying lengths of time.
Why I Reject A Young Earth View: A Biblical Defense of an Old Earth - Jonathan M. - 2011 Excerpt: Fourth, there is the multiple-usage of the word “day” in Genesis 1. Let’s take a look at the manner in which the word “day” is used in the Genesis 1 (up to 2:4) narrative alone: 1.  Genesis 1:5a: “God called the light Day, and the darkness he called Night.” Here, “day” is contrasted with “night”: Thus, a 24-hour day is not in view, but rather “day” in the sense of “daytime” (i.e. 12 hours). 2. Genesis 1:5b: “And there was evening and there was morning — the first day.” Here, the word does indeed mean a 24-hour day. 3. Genesis 2:3: “By the seventh day God had finished the work he had been doing; so on the seventh day he rested from all his work. 3 Then God blessed the seventh day and made it holy, because on it he rested from all the work of creating that he had done.” To this, I have already alluded — the key point here is the absence of “evening” and “morning”, which denotes all of the previous six days. 4. The correct rendering of the Hebrew with respect to Genesis 2:4 is  “This is the account of the heavens and the earth in the day they were created, when the LORD God made the earth and the heavens.” http://crossexamined.org/why-i-reject-a-young-earth-view-a-biblical-defense-of-an-old-earth/
bornagain77
With respect BA77, for all the hard work that you contribute to UD, I still could not resist a reply to BA77 # 149: - "in the hope of eternal life, which God, who does not lie, promised before the beginning of time," (Titus 1:2). __________________________________________________ Therefore, in relation to big bang cosmology, the Word from the beginning is untrue, and which He wrote and spoke at Sinai: that on His honour, He created in six days. In other words, His Word on the Creation is codswallop and gobbledegook; worse, He asked us to remember the same every seven days. Praise the Lord and pass the self-destructive ammunition.  mw
jdk @ 142 - Good points all. I agree with your observations, but as kf suggested, "Caution would be advisable." One of my chemistry professors used to paraphrase H.L.Mencken like this: "For every problem, there is a solution. Neat. Plausible. And wrong." -Q Querius
Good - I think we've got these points cleared up between us. jdk
Thanks for the additional clarification. StephenB
Thanks for the review: those were not subjects I was addressing. I was addressing the common confusion that people often think that infinity in respect to countable discrete objects is the only kind of infinity there is, which is not true. If time is continuous, then there are an infinite number of moments every second (not a potential infinity, but an actual infinity). And if every state of the universe at every moment is an event, then there are an infinite number of events. But I understand that this is not the way you are using the notion of infinity, and that my remarks don't actually bear on the issues you are discussing. You are thinking of events as being discrete, and taking some finite amount of time. Therefore, as I said in my original post, I agree that our universe doesn't contain an infinite number of finite entities. jdk
jdk
I don’t think you understand my point, though, so I’ll say no more. I’ve already said that the point I’m making doesn’t relate to the questions you are asking about infinite regress or an infinite past, etc., as those are assuming that events are countable and take discrete amounts of time, and it is that assumption that I am questioning.
Don't worry about repeating yourself if you feel the need. I understand your point, but I don't think it is relevant. Perhaps it would help if I reviewed the context of the original discussion. The issue on the table is whether or not a cosmos, consisting of time/space/matter, could be the first cause (and the eternal causeless cause) of our universe, which also consists of time, space, matter. sean says yes; I say no. I am arguing that the cause of time, space, matter must be a timeless, spaceless, immaterial being for the obvious reason: Time space and matter cannot cause time, space, and matter to come into existence. So we are discussing causation, causal chains, and role of an actual (not potential) infinity. StephenB
Thanks Eric for your references, # 114, and for sharing you have “some personal intellectual unease about the Big Bang and recognize that there are some important open issues.” I took some time to go through the postings and offshoots. Particularly of interest to me were those between BA77 and tjguy. mw
Actually, the point I am making relates to the other one of Zeno's paradoxes, about the arrow in motion, not the one you mention. The modern mathematical idea of the continuum of real numbers is how that paradox has been resolved in traditional concepts of space and time. I don't think you understand my point, though, so I'll say no more. I've already said that the point I'm making doesn't relate to the questions you are asking about infinite regress or an infinite past, etc., as those are assuming that events are countable and take discrete amounts of time, and it is that assumption that I am questioning. jdk
jdk
Here’s the mathematical situation: between any two points on the number line, no matter how close, be it 0 and 1, or 5 and 5.00000000001, or whatever, there are an infinite number of other numbers. The interval starts and stops, but contains an infinite number of points. That is what I mean by an infinitely dense continuum.
What you are describing is the equivalent of Zeno's paradox, in which case one half the distance to a location is covered with each successive move. The goal is never reached. In this case, one is assuming an infinite number of possible (potential) points on a continuum. This is potential infinity. It is easy to establish potential infinity in nature. Simply assume a number of events or particles. Now, double that number. Do it again. You can go on forever. That is not actual infinity. An actual number of successive events from the infinite past until now is not logically possible. Actual infinity cannot be instantiated in physical reality. StephenB
Hi Stephen. I'm not sure that you understand what I am saying about the difference between infinity in respect to discrete events and to a continuum of infinitely dense events. What I am saying has nothing to do with causal chains going back into time, or infinite regresses, or counting the number of discrete events. What I am saying is that it is a matter of both empirical evidence and philosophical assumption as to whether we treat an "event" as discrete or as a momentary state of a continuous flow. Here's the mathematical situation: between any two points on the number line, no matter how close, be it 0 and 1, or 5 and 5.00000000001, or whatever, there are an infinite number of other numbers. The interval starts and stops, but contains an infinite number of points. That is what I mean by an infinitely dense continuum. To apply this to reality, the question is what do we mean by "event", and what happens has you go from one event to the next. If we define an event to be the state of the universe, or some subset of the universe, at a moment in time, and we assume that time flows continuously, then there is no "next" event, just as there is no next point on the number line: between event A at time t1 and event B at time t2 there are an infinite number of other events. That time flows continuously in this manner has been the logical and intuitive assumption for hundreds of years, until recently when empirical evidence has pointed to a smallest interval of time, Planck's time, that may be a lower limit for the amount of time in which anything can happen. However, as I explained in my post to Querius, this doesn't mean that time runs discretely (as opposed to continuously): it just brings up the possibility that time and space might not be continuous. (P.S. Yes, I accept big bang cosmology and the scientific consensus that time/space/matter is finite and began to exist? As I am explaining, the points I am making are about what happens in time and space in this universe, not anything about what may have happened "before" or "outside" of this universe.) jdk
Quotes of note:
“No, I regard consciousness as fundamental. I regard matter as derivative from consciousness. We cannot get behind consciousness. Everything that we talk about, everything that we regard as existing, postulates consciousness.” Max Planck (1858–1947), the originator of quantum theory, The Observer, London, January 25, 1931 “Consciousness cannot be accounted for in physical terms. For consciousness is absolutely fundamental. It cannot be accounted for in terms of anything else.” Schroedinger, Erwin. 1984. “General Scientific and Popular Papers,” in Collected Papers, Vol. 4. Vienna: Austrian Academy of Sciences. Friedr. Vieweg & Sohn, Braunschweig/Wiesbaden. p. 334. "In any philosophy of reality that is not ultimately self-defeating or internally contradictory, mind – unlabeled as anything else, matter or spiritual – must be primary. What is “matter” and what is “conceptual” and what is “spiritual” can only be organized from mind. Mind controls what is perceived, how it is perceived, and how those percepts are labeled and organized. Mind must be postulated as the unobserved observer, the uncaused cause simply to avoid a self-negating, self-conflicting worldview. It is the necessary postulate of all necessary postulates, because nothing else can come first. To say anything else comes first requires mind to consider and argue that case and then believe it to be true, demonstrating that without mind, you could not believe that mind is not primary in the first place." - William J. Murray
bornagain77
Origenes: Self-awareness cannot be analyzed in a “before” and “after”. Sean S: Sure it can
Perhaps Sean would like to submit his 'you fall asleep and wake up' answer for the enigma of "The Now of the mind" in a peer-reviewed philosophical journal somewhere? Of related interest to the undermining of the space-time of General (and special) Relativity as a 'complete' description of reality, Einstein was once asked by a philosopher:
"Can physics demonstrate the existence of 'the now' in order to make the notion of 'now' into a scientifically valid term?"
Einstein's answer was categorical, he said:
"The experience of 'the now' cannot be turned into an object of physical measurement, it can never be a part of physics."
Quote was taken from the last few minutes of this following video.
Stanley L. Jaki: "The Mind and Its Now" https://vimeo.com/10588094
And here a bit more detail:
The Mind and Its Now - May 22, 2008 - By Stanley L. Jaki Excerpt: ,,, Rudolf Carnap, and the only one among them who was bothered with the mind's experience of its now. His concern for this is noteworthy because he went about it in the wrong way. He thought that physics was the only sound way to know and to know anything. It was therefore only logical on his part that he should approach, we are around 1935, Albert Einstein, the greatest physicist of the day, with the question whether it was possible to turn the experience of the now into a scientific knowledge. Such knowledge must of course be verified with measurement. We do not have the exact record of Carnap's conversation with Einstein whom he went to visit in Princeton, at eighteen hours by train at that time from Chicago. But from Einstein's reply which Carnap jotted down later, it is safe to assume that Carnap reasoned with him as outlined above. Einstein's answer was categorical: The experience of the now cannot be turned into an object of physical measurement. It can never be part of physics. http://metanexus.net/essay/mind-and-its-now
The meaning of the question of 'the Now' can also be read in fuller context in the article:
The Mind and Its Now - Stanley L. Jaki, May 2008 Excerpts: There can be no active mind without its sensing its existence in the moment called now.,,, Three quarters of a century ago Charles Sherrington, the greatest modern student of the brain, spoke memorably on the mind's baffling independence of the brain. The mind lives in a self-continued now or rather in the now continued in the self. This life involves the entire brain, some parts of which overlap, others do not. ,,,There is no physical parallel to the mind's ability to extend from its position in the momentary present to its past moments, or in its ability to imagine its future. The mind remains identical with itself while it lives through its momentary nows. ,,, the now is immensely richer an experience than any marvelous set of numbers, even if science could give an account of the set of numbers, in terms of energy levels. The now is not a number. It is rather a word, the most decisive of all words. It is through experiencing that word that the mind comes alive and registers all existence around and well beyond. ,,, All our moments, all our nows, flow into a personal continuum, of which the supreme form is the NOW which is uncreated, because it simply IS. per metanex
Prior to his encounter with Carnap, Einstein had another encounter with another famous philosopher over 'the now of the mind'. In fact, that particular encounter over 'the now of the mind' was one of the primary reasons that Einstein never received a Nobel prize for relativity:
Einstein, Bergson, and the Experiment that Failed: Intellectual Cooperation at the League of Nations! - Jimena Canales page 1177 Excerpt: Bergson temporarily had the last word during their meeting at Société française de philosophie. His intervention negatively affected Einstein’s Nobel Prize, which was given “for his services to theoretical physics, and especially for his discovery of the law of the photoelectric effect” and not for relativity. The reasons behind this decision, as stated in the prize’s presentation speech, were related to Bergson’s intervention: “Most discussion [of Einstein’s work] centers on his Theory of Relativity. This pertains to epistemology and has therefore been the subject of lively debate in philosophical circles. It will be no secret that the famous philosopher Bergson in Paris has challenged this theory, while other philosophers have acclaimed it wholeheartedly.”51 For a moment, their debate dragged matters of time out of the solid terrain of “matters of fact” and into the shaky ground of “matters of concern.”52 https://dash.harvard.edu/bitstream/handle/1/3210598/canales-Einstein,%20Bergson%20and%20the%20Experiment%20that%20Failed%282%29.pdf?sequence=2 Einstein vs Bergson, science vs philosophy and the meaning of time - Wednesday 24 June 2015 Excerpt: The meeting of April 6 was supposed to be a cordial affair, though it ended up being anything but. ‘I have to say that day exploded and it was referenced over and over again in the 20th century,’ says Canales. ‘The key sentence was something that Einstein said: “The time of the philosophers did not exist.”’ It’s hard to know whether Bergson was expecting such a sharp jab. In just one sentence, Bergson’s notion of duration—a major part of his thesis on time—was dealt a mortal blow. As Canales reads it, the line was carefully crafted for maximum impact. ‘What he meant was that philosophers frequently based their stories on a psychological approach and [new] physical knowledge showed that these philosophical approaches were nothing more than errors of the mind.’ The night would only get worse. ‘This was extremely scandalous,’ says Canales. ‘Einstein had been invited by philosophers to speak at their society, and you had this physicist say very clearly that their time did not exist.’ Bergson was outraged, but the philosopher did not take it lying down. A few months later Einstein was awarded the Nobel Prize for the discovery of the law of photoelectric effect, an area of science that Canales noted, ‘hardly jolted the public’s imagination’. In truth, Einstein coveted recognition for his work on relativity. Bergson inflicted some return humiliation of his own. By casting doubt on Einstein’s theoretical trajectory, Bergson dissuaded the committee from awarding the prize for relativity. In 1922, the jury was still out on the correct interpretation of time. So began a dispute that festered for years and played into the larger rift between physics and philosophy, science and the humanities. Bergson was fond of saying that time was the experience of waiting for a lump of sugar to dissolve in a glass of water. It was a declaration that one could not talk about time without reference to human consciousness and human perception. Einstein would say that time is what clocks measure. Bergson would no doubt ask why we build clocks in the first place. ‘He argued that if we didn’t have a prior sense of time we wouldn’t have been led to build clocks and we wouldn’t even use them ... unless we wanted to go places and to events that mattered,’ says Canales. ‘You can see that their points of view were very different.’ In a theoretical nutshell this expressed perfectly the division between lived time and spacetime: subjective experience versus objective reality.,,, Just when Einstein thought he had it worked out, along came the discovery of quantum theory and with it the possibility of a Bergsonian universe of indeterminacy and change. God did, it seems, play dice with the universe, contra to Einstein’s famous aphorism. Some supporters went as far as to say that Bergson’s earlier work anticipated the quantum revolution of Niels Bohr and Werner Heisenberg by four decades or more. Canales quotes the literary critic Andre Rousseaux, writing at the time of Bergson’s death. ‘The Bergson revolution will be doubled by a scientific revolution that, on its own, would have demanded the philosophical revolution that Bergson led, even if he had not done it.’ Was Bergson right after all? Time will tell. http://www.abc.net.au/radionational/programs/philosopherszone/science-vs-philosophy-and-the-meaning-of-time/6539568
Moreover, the statement to Carnap on the train that, 'the now' cannot be turned into an object of physical measurement’, was an interesting statement for Einstein to make to the philosopher since 'the now of the mind' has, from many recent experiments in quantum mechanics, undermined the space-time of Einstein's General Relativity as to being the absolute frame of reference for reality.
LIVING IN A QUANTUM WORLD - Vlatko Vedral - 2011 Excerpt: Thus, the fact that quantum mechanics applies on all scales forces us to confront the theory’s deepest mysteries. We cannot simply write them off as mere details that matter only on the very smallest scales. For instance, space and time are two of the most fundamental classical concepts, but according to quantum mechanics they are secondary. The entanglements are primary. They interconnect quantum systems without reference to space and time. If there were a dividing line between the quantum and the classical worlds, we could use the space and time of the classical world to provide a framework for describing quantum processes. But without such a dividing line—and, indeed, with­out a truly classical world—we lose this framework. We must explain space and time (4D space-time) as somehow emerging from fundamentally spaceless and timeless physics. http://phy.ntnu.edu.tw/~chchang/Notes10b/0611038.pdf Einstein vs. "The Now" of Philosophers and Quantum Mechanics – video https://www.facebook.com/philip.cunningham.73/videos/vb.100000088262100/1129789497033982/?type=2&theater A Short Survey Of Quantum Mechanics and Consciousness Excerpt: Putting all the lines of evidence together the argument for God from consciousness can now be framed like this: 1. Consciousness either preceded all of material reality or is a ‘epi-phenomena’ of material reality. 2. If consciousness is a ‘epi-phenomena’ of material reality then consciousness will be found to have no special position within material reality. Whereas conversely, if consciousness precedes material reality then consciousness will be found to have a special position within material reality. 3. Consciousness is found to have a special, even central, position within material reality. 4. Therefore, consciousness is found to precede material reality. Four intersecting lines of experimental evidence from quantum mechanics that shows that consciousness precedes material reality (Wigner’s Quantum Symmetries, Wheeler’s Delayed Choice, Leggett’s Inequalities, Quantum Zeno effect) https://docs.google.com/document/d/1uLcJUgLm1vwFyjwcbwuYP0bK6k8mXy-of990HudzduI/edit New Mind-blowing Experiment Confirms That Reality Doesn’t Exist If You Are Not Looking at It - June 3, 2015 Excerpt: The results of the Australian scientists’ experiment, which were published in the journal Nature Physics, show that this choice is determined by the way the object is measured, which is in accordance with what quantum theory predicts. “It proves that measurement is everything. At the quantum level, reality does not exist if you are not looking at it,” said lead researcher Dr. Andrew Truscott in a press release.,,, “The atoms did not travel from A to B. It was only when they were measured at the end of the journey that their wave-like or particle-like behavior was brought into existence,” he said. Thus, this experiment adds to the validity of the quantum theory and provides new evidence to the idea that reality doesn’t exist without an observer. http://themindunleashed.org/2015/06/new-mind-blowing-experiment-confirms-that-reality-doesnt-exist-if-you-are-not-looking-at-it.html Lecture 11: Decoherence and Hidden Variables – Scott Aaronson – MIT associate Professor Excerpt: “Look, we all have fun ridiculing the creationists who think the world sprang into existence on October 23, 4004 BC at 9AM (presumably Babylonian time), with the fossils already in the ground, light from distant stars heading toward us, etc. But if we accept the usual picture of quantum mechanics, then in a certain sense the situation is far worse: the world (as you experience it) might as well not have existed 10^-43 seconds ago!”
i.e. 'the Now', as philosophers term it, and contrary to what Einstein thought possible for experimental physics, and according to advances in quantum mechanics, takes precedence over past events in time. Moreover, due to advances in quantum mechanics, it would now be much more appropriate to phrase Einstein's answer to the philosopher in this way:
"It is impossible for the experience of 'the now of the mind' to ever be divorced from physical measurement, it will always be a part of physics."
Of supplemental note, despite the undermining of relativity by quantum mechanics, Relativity actually has a lot more in common with 'the now of the mind' than one would, at first glance, expect it to have:
“For those of us who believe in physics. the distinction between past, present, and future is only an illusion, however tenacious this illusion may be.” – Albert Einstein – March 1955 – in the letter to comfort the family of a dear friend who had passed away. (of note: Einstein passed away the next month, in April of that same year) Einstein: A Biography, pg. 402 "The laws of relativity have changed timeless existence from a theological claim to a physical reality. Light, you see, is outside of time, a fact of nature proven in thousands of experiments at hundreds of universities. I don’t pretend to know how tomorrow can exist simultaneously with today and yesterday. But at the speed of light they actually and rigorously do. Time does not pass." Dr. Richard Swenson - More Than Meets The Eye, Chpt. 11 Special and General Relativity compared to Heavenly and Hellish Near Death Experiences – video (reworked May 2016) https://www.facebook.com/philip.cunningham.73/videos/1193118270701104/
Verse:
Titus 1:2 in the hope of eternal life, which God, who does not lie, promised before the beginning of time,
bornagain77
Sean S:
Origenes: Self-awareness cannot be analyzed in a “before” and “after”.
Sure it can. Every time you fall asleep and wake up, or drift off and return. Anyone who’s ever blacked-out, or gone under general anesthesia has experienced the stopping and restarting of consciousness.
Non-responsive. If on a Tuesday afternoon scientists show that some quantum effect precedes its cause in time, it makes little sense to say that it still can be analyzed in a “ before” and “after”, because the afternoon during which the experiment took place was preceded by a morning. Origenes
sean samis #134, referring to StephenB #129, who said, “An omnipotent Creator could (and did) create time itself.” Sean then said: __________________________________________________ Which (if true) means there was a time before “time was created” as well as a time after “time was created”. These mean that your hypothetical “omnipotent Creator” is subject to time. __________________________________________________ Not really Sean, it just means we do not understand how spirit/eternal time relates to our material space-time. God is Time Lord. “Our Father, who art in heaven . . .”. Are we to say God is subject to heaven? If we are honest, we surely must admit, we cannot imagine space ending. Nor do I profess to understand what spirit is. I believe that spirit gives life, not matter (Jn 6:63). Whereas, Sean believes nothing caused everything. Fine. Nor do I understand what exactly spirit is. But people have witnessed and documented that such a concept is real, powerful, and life-giving, as only life can come from life. True science, has never proved otherwise; it is hypothetical only in the imagination of consensus evolutionists minds in order to cast doubt or dismiss divine law as unreliable, therefore the God who wrote such is also unreliable, even though He gave His word how He created, witnessed historically in a show of power at Sinai. If divine law is true relative to the cosmos we live in; we should not bet our life, or place too much reliance on certain measurements, because we live in a cosmos that reflects the after effects of a miracle. That is, we live in a spiritual/material paradox, nevertheless, true to divine law in terms of Judaeo-Christianity. That is, "For in Him, we live and move and have our being." (Acts 17:28) mw
Folks, passing by. Ponder, movies, where states are discretely updated but our senses more or less integrate into an apparent continuum. We thus know we can perceive continuum where there is a succession of discrete states. A clue that something is up is a sampling artifact, aliasing; which is a mathematical-logical consequence of sampling. A fine grained discrete state cosmos where there is no "co-ordination" of events [as opposed to a sequence of updated snapshots] would indeed be well below Planck time limits. Who knows, it is sufficient that we can use discrete and continuum models. Where, continuum precisely does not entail discrete succession like steps along a ladder with jumps between. And as was discussed earlier this year, succession to endlessness is pivotal to countable transfinites. We can only ever go a finite extent with endlessness beyond. I argued, this imposes an endlessness spanning challenge on all infinitely successive in the past causal models, which poses such implications that am actually infinite past succession of discrete, cumulative, cause-effect bonded events to now does not seem reasonable. That we can pose a continuum in which between successive times we may always interpolate intermediate, valid times, etc [big "etc"], is a different matter. E.g. in the near neighbourhood of whatever zero one wishes, there is in effect a 1/t image of the full extent of the time line, which by simple addition [ think 1 s point with cloud of values c: 1 + c, c's taken from the 1/t neighbourhood of 0] entails that each distinct point has a similar inexhaustibly deep hyper-real neighbourhood of values. This is mathematical, it does not entail a physical world with an infinitely deep continuum of times that we may in principle discretise and then announce yes, infinite succession and real world infinite. No, we are here in a math-logic model world, not an observed actual demonstrated continuum. As to number of particles being actually infinite [as opposed to v v large but finite], we could never make such an observation, that is not science. And of course the big bang frame points to a finitely remote beginning of the only actually indisputably observed physical cosmos. Caution would be advisable. KF kairosfocus
It doesn't matter how you break it down. Infinity cannot be instantiated in physical reality. Whether it is a quadrillion events lasting one minute each or one event that "flows" for a quadrillion years, there is, and must be, an end to the causal chain going backwards in time. If you disagree, feel free to define "event" your own way and proceed from there: Show me how it is possible to use empirical evidence to instantiate infinity in that physical formulation. Show me how to get around the problem of an infinite regress of physical events. Show me how you would use empirical evidence to measure an infinite number/amount of particles/energy. Show me how something that is a measurable (the number of events) is also not measurable (infinity). (As a matter of curiosity, do you accept big bang cosmology and the scientific consensus that time/space/matter is finite and began to exist?) StephenB
If events are discrete, then I agree with you that there can't be an infinite number of them. However, in the real world what we decide to call an event is part of our model of reality, so the situation, as my post to Querius was meant to address, is not so simple. What exactly is an event: is it a clear, discrete thing, or an abstracted portion of a continuous flow of change in the world? The question about whether events flow continuously, so that they are not discrete, or "jump" from event to event as discrete, separate happenings, is a question that requires empirical evidence to decide. Whatever the case, once we decide, it is our model of reality that we are describing. By using language and mathematics (whether it be "cat" or "quantum particle") our knowledge is an abstraction from the world we experience. jdk
jdk
Hmmm. All of our knowledge about reality is a model: a mapping of abstract concepts with abstractions about reality. Are events, happening in time and space, discrete or continuous? Do we “really know”, or do we just have models that we test, and are functionally successful in describing what we experience, that assume one or the other. Most physics assumes continuity in some ways (complicated considerably by quantum mechanics).
Not all of our knowledge about reality is a model. I don’t need a model, for example, to know that a cat is not a dog, or that a human is on a higher intellectual plane than animals. There are many truths which cannot be arrived at through empirical methods. We are discussing one such truth. An infinite amount of physical objects, or an infinite regress of causes/physical events in the cosmos, is logically impossible.
So whether there are or aren’t an infinite number of events depends on which model of reality one adopts: does time flow continuously from moment to moment, so that events are an infinitely dense continuum, or are there fundamental discontinuities, so that actual events are discrete? I don’t think this a question that can be decided purely by logic, at either the physical or metaphysical level.
No model or theory could ever help us with this problem. You seem to sense this at some level. Again, reason has already provided the answer to this question: Infinity is not countable, but the number of physical particles and events in the universe could be counted if we had the technology and the wherewithal to do the counting. Thus, it is clear that infinite physical events cannot be instantiated in physical reality. Reason’s rules inform scientific evidence; scientific evidence does not inform reason’s rules. StephenB
Hi Querius. I understand that quantum theory, including planck's time as a limit, is consistent with time being discrete and space being discrete. However, it's more complicated than your example, because there are a huge number of quantum events going on in the universe, and not all simultaneously. So particle A may go from state a1 to state a2 with a jump in time of planck's length, but another particle B may be making it's jump from b1 to b2 at a different time, so instead of there being one period in which there is nothing happening, so to speak, as A and B change states, there are three such periods: a1 to b1, b1 to a2, and a2 to b2 (assuming B starts its jump after A). This continues for all the quantum particles in the universe, so the pixillation of events in the universe as a whole - the smallest amount of time in which nothing at all is happening, is vastly smaller than planck's time. Now, under this analysis, time and space are still not truly continuous in the mathematical sense. There are very very small moments when every particle in the universe is not doing anything: the universe is static at those moments, and there are gaps between events. If this is really how it is, then there are not an infinite number of events in our universe. In this case, the mathematical continuity we use is only a useful tool in describing the universe, because treating the pixellation of the universe as continuous is vastly more practical than working with discrete functions. And the question remains: in the very brief moments when nothing is happening, does time exist, or is time an artifact of activity? That is, do events happen in time, or does time happen because events take place? All very interesting to think about, in my opinion. jdk
Sean sez
Who is to say anything concrete “exists outside of space or time”?
LOL. Sean's in denial. Quantum mechanics demonstrates experimentally that there's nothing "concrete" in our own space and time. It's all fundamentally wavefunctions. Wavefunctions are mathematical probabilities. As such, they are without mass or energy until they are observed or measured. Daniel King sounds like a sycophant sock puppet stuffed with vacuous assertions. StephanB noted,
You just said that if God Created time, then time was present before God created it? Do you not understand the contradiction?
Apparently not. How about some humor? "How long did it take God to create time?" "Where did God stand when he created matter?" "Did God start to exist with the Big Bang?" The noise you just heard was Sean's head exploding. ;-) jdk opined,
Most physics assumes continuity in some ways (complicated considerably by quantum mechanics).
The assumption holds only at macro scales. Continuity is lost when observing quantum effects. For example, with quantum tunneling, an electron can suddenly materialize on the opposite side of an electrically impenetrable barrier. Practically speaking, this puts a limit on the miniaturization of microelectronics. Planck length is a subdivision limit within space. This is analogous to the pixel density on your monitor. No continuity to infinity there! -Q Querius
Hmmm. All of our knowledge about reality is a model: a mapping of abstract concepts with abstractions about reality. Are events, happening in time and space, discrete or continuous? Do we "really know", or do we just have models that we test, and are functionally successful in describing what we experience, that assume one or the other. Most physics assumes continuity in some ways (complicated considerably by quantum mechanics). So whether there are or aren't an infinite number of events depends on which model of reality one adopts: does time flow continuously from moment to moment, so that events are an infinitely dense continuum, or are there fundamental discontinuities, so that actual events are discrete? I don't think this a question that can be decided purely by logic, at either the physical or metaphysical level. jdk
jdk
Actually, as I mentioned in my response to johnnyb about natural causes, our mathematical model of reality, upon which the critical subject of derivatives is based, assumes continuity and thus an densely infinite number of moments of time and points in space. There isn’t an infinite number of discrete objects or events in the universe , but we assume an infinite number of events when we model the world as flowing continuously from one moment to the next.
Thank you for confirming my point. "There isn't an infinite number of discrete objects or events in the universe." Infinity cannot be instantiated in physical reality. How some people may model the universe is not relevant to the logical or metaphysical fact of what can or cannot be. StephenB
sean
I also cannot say in principle that Angels exist, much less have neither material nor dimension.
Angels are, by definition, pure spirits. Thus, they cannot have matter or dimension. Obviously, an immaterial being cannot be made of matter.
If we allow angels to be purely imaginary, they could be anything, and are irrelevant.
You said the anything that is not dimensional or material is nothing. That is a mistaken claim. A pure spirit, such as an angel, is a causal agent. A causal agent cannot be nothing.
If we treat angels as real, then we have no reason to say they are outside time, space, or matter.
If angels exist, then they came to exist outside of time since there was no such thing as time when they were Created.
First of all, the verb instantiated does not apply; the question is whether any infinity can exist in the physical world.
The word instantiated is often used in that context. In any case, infinity cannot exist in the physical world.
And, in the physical reality of our universe, an actual infinity might be impossible, but a more general claim about the cosmos(*) (as I used the term) cannot be shown absurd; it is certainly no more absurd than a “spaceless/timeless/immaterial” first cause.
There can be no infinite regress of physical events any more than there can be an infinite regress of physical causes. Indeed, it isn't even possible to have an infinite amount of particles in the cosmos. You can have potential infinity; you cannot have actual infinity. SB: Causation can be either chronologically or logically prior to its effect. Hence, a causative act need not occur in time
We’re not talking about only theoretical mental constructs where only logical causation is critical; for any physical causation, for any actual ACT to occur, chronological causation is essential, which means time is implicated.
A causal act need not occur in time/space/history. I have already provided the example of Angels. If God Created the universe outside of time, that, too, would be an causal act outside of time. In effect, you are saying that it is logically impossible for an eternal God to Create the universe exnihilo. I don't think you will get much support for that proposition. SB: An omnipotent Creator could (and did) create time itself.
Which (if true) means there was a time before “time was created” as well as a time after “time was created”. These mean that your hypothetical “omnipotent Creator” is subject to time.
You just said that if God Created time, then time was present before God created it? Do you not understand the contradiction? StephenB
They keep lobbing soft balls over the plate and sean samis keeps knocking them out of the park. Daniel King
Stephen writes,
Thus, we note the importance of linking the finite number of events that have occurred with the temporal duration of the physical cosmos. (i. e. infinitely cannot be instantiated in physical reality).
Actually, as I mentioned in my response to johnnyb about natural causes, our mathematical model of reality, upon which the critical subject of derivatives is based, assumes continuity and thus an densely infinite number of moments of time and points in space. There isn't an infinite number of discrete objects or events in the universe, but we assume an infinite number of events when we model the world as flowing continuously from one moment to the next. Just an FYI: jdk
Phinehas, re # 130; If your term is intended to promote a particular view-point, it is by definition “not neutral”. You can do better. sean s. sean samis
StephenB; re #129;
A pure spirit is not nothing. Even if you don’t believe in Angels, you cannot say, in principle, that Angels are nothing.
I also cannot say in principle that Angels exist, much less have neither material nor dimension. If we allow angels to be purely imaginary, they could be anything, and are irrelevant. If we treat angels as real, then we have no reason to say they are outside time, space, or matter.
The existence of an infinite set in mathematics does not, in any way, justify the absurd notion that infinity can be instantiated in physical reality.
First of all, the verb instantiated does not apply; the question is whether any infinity can exist in the physical world. And, in the physical reality of our universe, an actual infinity might be impossible, but a more general claim about the cosmos(*) (as I used the term) cannot be shown absurd; it is certainly no more absurd than a “spaceless/timeless/immaterial” first cause.
Causation can be either chronologically or logically prior to its effect. Hence, a causative act need not occur in time
We’re not talking about only theoretical mental constructs where only logical causation is critical; for any physical causation, for any actual ACT to occur, chronological causation is essential, which means time is implicated.
An omnipotent Creator could (and did) create time itself.
Which (if true) means there was a time before “time was created” as well as a time after “time was created”. These mean that your hypothetical “omnipotent Creator” is subject to time. sean s. (edited) sean samis
daveS, re #127
Certainly we can consider the man climbing the ladder, however in that scenario, we are no longer dealing with a beginningless process, and that changes everything (in the context of that thread, anyway).
My point is that changing beginningless to endless (or vice-versa) really does not change anything significant. Consider someone sitting down at a point in time and recollecting the prior moments, and then the moments prior to that, and the moments prior to that – ad infinitum. This person’s act turns a “completed infinity” (the past) into a “potential infinity” (the complete recollection of an infinite past). sean s. (edited) sean samis
Daniel King, re #126; Thank you, Daniel. sean s. sean samis
Origenes; Re # 124;
Consider ‘self-awareness’; a person is observer and observee simultaneously and it is impossible to make sense of it in the context of time and space.
Self-awareness is a property of our mind, which operates within our brains. Our brains (and our minds) are very much embedded in space and time.
Self-awareness cannot be analyzed in a “before” and “after”.
Sure it can. Every time you fall asleep and wake up, or drift off and return. Anyone who’s ever blacked-out, or gone under general anesthesia has experienced the stopping and restarting of consciousness.
Consciousness, morality, abstract ideas, emotions, thoughts and so forth arguably have no dimensions
Two comments: we’re not talking about “abstract ideas” here, but some First Cause which created our universe. And “arguably” means “that’s what some people say”, not “it’s a fact that...”. Who is to say anything concrete “exists outside of space or time”? sean s. sean samis
SS:
Perhaps. Do you have a better term, a neutral term to suggest? If not, we’ll have to stick with “natural selection”.
I'd be happy provide a better, neutral term: Nature. There is no "selection" in nature. That's just a teleological anthropomorphism that's been smuggled in. When you remove what doesn't belong there in the first place, the reality becomes clear. Phinehas
sean
Something that has neither material nor dimension is quite literally nothing.
Incorrect. A pure spirit is not nothing. Even if you don't believe in Angels, you cannot say, in principle, that Angels are nothing.
Infinite sets have been demonstrated to violate “finite” thinking; these sets have demonstrated properties that ordinary, valid, arithmetic thinking would consider absurd.
The existence of an infinite set in mathematics does not, in any way, justify the absurd notion that infinity can be instantiated in physical reality.
if the First Cause is actually outside of time, then this First Cause cannot act. Actions, even as small as forming a thought, are embedded in time: there’s the moment before the action begins, the moment the action begins, and the moment the action ends or is completed. Actions imply “before” and “after” (as well as various kinds of “during”); all these terms are expressions of time. Therefore, a “timeless” First Cause is INERT.
Incorrect. Causation can be either chronologically or logically prior to its effect. Hence, a causative act need not occur in time. An omnipotent Creator could (and did) create time itself. StephenB
Daniel King noted:
I’m taking a philosophy course, and we’ve been reading Aristotle, Augustine, Averroes, and Maimonides on the subject of your post.
Of course none of these gentlemen had any knowledge of quantum mechanics, entropy, or the relationships of space-time and mass-energy. In that sense, they are indeed much like Sean, although they came first. ;-) -Q Querius
sean s, I didn't notice this quote of mine until HeKS pointed it out:
daveS: I think this illustrates the issues with “running the tape backwards” so that the man appears to be ascending the ladder. I would recommend not even bringing that up. The scenario involves the man descending throughout an infinite past, until he reaches the ground at the present, say.
sean s: These are explanations but not a justification. If we count backwards (which is just as logical and permissible as counting forwards) then we see that potential and actual (or “completed”) infinities are both acceptable; that was one of the contributions of Georg Cantor. If we had a recording of a man ascending the infinite ladder (which you say is logically permissible) would it be illogical to play this recording in reverse? No. With the exception of time-irreversable phenomena (which climbing a ladder is not) time-reversal is logically permissible.
Just to clarify, in that thread, I was trying to get our opponents to make their arguments for the impossibility of the man completing the descent without switching to the thought experiment where the man is ascending. Certainly we can consider the man climbing the ladder, however in that scenario, we are no longer dealing with a beginningless process, and that changes everything (in the context of that thread, anyway). daveS
sean samis @ 122 I'm taking a philosophy course, and we've been reading Aristotle, Augustine, Averroes, and Maimonides on the subject of your post. You've summarized the skeptical argument more lucidly than I've seen before. Thanks Daniel King
Hi Sean, I just now finished the project I was working on after working about 100 hours this week. I start a new project tomorrow, though it shouldn't be nearly so intensive. I read through your post and I simply don't agree with your critiques. Part of the problem seems to be that you are simply assuming that past infinities do/can exist and are logically coherent and claiming I simply don't know how to look at them properly because they work fine as abstract mathematical constructs. Then you go on to attempt to quasi-justify them using precisely the same approaches that I had said were mistaken ... which are ways that incorrectly make completed infinities seem to be no different than potential infinities by looking at the completed infinities from the wrong side and in the wrong direction, and I don't agree with you that this is simply a problem of me using 'finite thinking'. Also, you a few times made this kind of comment:
If we were to count an infinite number of [whatever] then we could not do it in less than infinite time, but if the past IS infinite, then no such “traverse” is needed. The past did not traverse an infinite set of moments; the past is that infinite set of moments.
You seem to be suggesting that I said the past had to do the traversing. I didn't. At least not as far as I recall (If I did it would only have been as a shorthand). What I said repeatedly was what you quoted:
...we would still be left with the impossibility of traversing those infinities through any stepwise chain, whether we’re talking about seconds, causal events, or anything else.
BTW, I didn't remember saying this bit...
“I think this illustrates the issues with “running the tape backwards” so that the man appears to be ascending the ladder. I would recommend not even bringing that up. The scenario involves the man descending throughout an infinite past, until he reaches the ground at the present, say.”
... so I went back and checked. I was correct. That wasn't me. It was daveS who said that bit. Finally, "extensional" doesn't exactly mean finite, per se. It more specifically means 'extended in time and space', or in some cases things that actively extend/expand or are capable of extending/expanding. Our universe would fall under both senses. I'll respond more fully when I have some time and some energy. Take care, HeKS HeKS
sean samis @122,
sean samis: Consider: if the First Cause is actually outside of time, then this First Cause cannot act. Actions, even as small as forming a thought, are embedded in time: there’s the moment before the action begins, the moment the action begins, and the moment the action ends or is completed. Actions imply “before” and “after” (as well as various kinds of “during”); all these terms are expressions of time. Therefore, a “timeless” First Cause is INERT.
Time and space are contexts of material events, with some notable exceptions like quantum entanglement. However spiritual beings like us habitually perform actions which don’t fit a time space context. Consider ‘self-awareness’; a person is observer and observee simultaneously and it is impossible to make sense of it in the context of time and space. Self-awareness cannot be analyzed in a “before” and “after”. Self-awareness cannot be understood in the context of space — there is no person standing on the outside looking in. Self-awareness cannot arise from two distinct persons — an observer and an observee. Somehow it is one indivisible action. Consider having a thought and expressing it; it doesn’t fit a time context. Ideas are instantaneous and out of nowhere; we do not build them bottom-up, word for word. Again, notice the oneness: it cannot be broken down in: at time=1 “In”, at time=2 “the”, at time=3 “beginning”. Sure, occasionally we may look for a better word, but the idea and its coherence is already there, at once; it just needs to be expressed. Also, consider personal identity over time.
sean samis: Something that has neither material nor dimension is quite literally nothing. Therefore, an immaterial First Cause existing outside of space is NOTHINGNESS. Therefore, a timeless, immaterial First Cause existing outside of space is an INERT NOTHINGNESS.
Consciousness, morality, abstract ideas, emotions, thoughts and so forth arguably have no dimensions — are outside of space — but who is to say that they are “inert nothingness”? Origenes
Sean,
This idea has no merit.
All you've demonstrated is that you cannot imagine alternatives to our universe's space-time dimension, which is not particularly surprising. Most physicists and cosmologists will tell you that the universe had a beginning that involved the inflation of space-time (before which neither space nor time existed). By making your assertion, you also exclude the possibility that our universe had a cause, which means that it simply popped into existence for no reason at all, upon which time then began. Do you know what entropy is? You also have yet to explain John Stewart Bell’s rhetorical question that you quoted. What was he saying? -Q Querius
HeKS, A preliminary: going forward from this point in this comment, I will use the term “cosmos(*)” to refer to our universe, any other universe, and any multiverse/cosmic bubble/etc. which created our universe. Please bear this in mind as you read on. In my last comment to you, I promised a couple of things. The first was that I would watch the entire Rowe/Grayling debate because you wrote that,
the argument [about extensional reality] is laid out a bit more there.
Assuming this was in regard to my questions about claims that the cosmos(*) must be an extensional entity (and therefore needs an non-extensional entity to create it). Rabbi Rowe and why the cosmos(*) ”must be” extensional. Watched the entire debate and I can tell you Rabbi Rowe did not provide any better explanation for this claim than you or others have already. He did clarify that the term “extensional” means essentially the same thing as “finite”. That was helpful. But like all comments on this thread, R. Rowe simply assumes and asserted that a multiverse is an extensional (finite) entity without even a slight effort to justify the claim. Like most commentators, I agree that R. Rowe “won” this debate; Grayling not only said some foolish things, but missed some foolish things R. Rowe said, this item included. On this point however, R. Rowe did not deliver. He, and you, and the other commentators on this thread have utterly failed to justify the assumption that a multiverse/cosmic bubble/etc must be extensional (finite); and that assumption is not a given. That question remains unanswered. I reasonably believe that the cosmos(*) probably is actually infinite and eternal. Finite thinking about infinities HeKS, A second promise I made to you was to “read through the recent thread here (“Durston and Craig on an infinite temporal past . . .”) on the impossibility of infinite past time. If you’re looking for my specific take on it then it is explained at some length in my comments in that thread.” My promise was that; “given there are more than 1400 comments, I won’t even try to read them all. But I will try to read yours. There seems to be only about 30 of them.” At this moment, I’m about half-way through your comments, and certain patterns have emerged, to which I should reply. Please bear with me while I set-up my response. 1. Infinite sets have been demonstrated to violate “finite” thinking; these sets have demonstrated properties that ordinary, valid, arithmetic thinking would consider absurd. A classic example is that the set of Prime Numbers {Pr} has the same cardinality (number of elements) as the set of Natural Numbers {N}. Clearly, only some natural numbers are primes (and all primes are natural numbers), yet the total number of primes is the same as the total number of natural numbers; this can be demonstrated by showing that there is a one-to-one correspondence between the elements of N and Pr. There are many other examples of infinite sets which appear to violate “finite” thinking based on the obvious rules of arithmetic. This is an important point. The objections you raise on the other thread rely on “finite” thinking about infinite sets. Infinite sets are not things we regularly encounter, so the human mind is not inclined to think about them as it should. We are capable of thinking properly about infinities, but we have to consciously choose to do so. 2. “Finite” thinking is simply the wrong tool to use when thinking about infinites; if one insists on using a hammer, screws will seem incomprehensible and a really bad idea. But of course, they are not. The wrong tool leads to the wrong conclusion. On the other thread you expanded on a scenario which I think KF originated: the infinite ladder into the sky. In #1116 you wrote that,
On the other hand, if the past is beginningless then we immediately find ourselves with an actual infinite number of past days, minutes, seconds or events that have been traversed, one at a time, to arrive at the present. In other words, we have walked into a field and found a man stepping off the last rung of an infinite ladder, finishing an infinite climb that he never started.
This is “finite” thinking about an infinite set. A “beginningless” infinite set has no beginning, so the concept of “starting” (the act of doing the first thing) simply does not apply; an angel climbing a “beginningless” infinite set of ladder rungs has simply always been climbing. Looking for a beginning is “finite” thinking; it seems natural because in our finite world things are expected to have beginnings. Infinities defy that expectation. 3. The past did not traverse an infinite set of moments; the past IS that infinite set of moments. You also make much out of the idea of “traversing” an infinite set of moments; you wrote,
In other words, even if we were to allow the possible existence of actual infinities in the real world (which I don’t), we would still be left with the impossibility of traversing those infinities through any stepwise chain, whether we’re talking about seconds, causal events, or anything else.
If we were to count an infinite number of [whatever] then we could not do it in less than infinite time, but if the past IS infinite, then no such “traverse” is needed. The past did not traverse an infinite set of moments; the past is that infinite set of moments. There are other examples of your use of “finite” thinking which cause you to stumble over infinite pasts; but all can be resolved by treating an infinity as an infinity. You also struggle with the idea of counting time backwards; writing that,
“The reason I’m addressing events going into the future is because if you conceptualize “ending a beginningless sequence” by moving backwards from the ending towards the direction of the beginningless, then you are changing the concept of “ending the beginningless” into the concept of “beginning the endless”, but the two concepts are not in any way comparable. For us in the present, the existence of an infinite past would be a case of “ending the beginningless”. Conceptualizing this a starting from the present and moving backwards in time along a negative timeline would be to reconceptualize it as “beginning the endless”, and thereby to reconceptualize an actually infinite past as being comparable to a potentially infinite future." ... "I think this illustrates the issues with “running the tape backwards” so that the man appears to be ascending the ladder. I would recommend not even bringing that up. The scenario involves the man descending throughout an infinite past, until he reaches the ground at the present, say."
These are explanations but not a justification. If we count backwards (which is just as logical and permissible as counting forwards) then we see that potential and actual (or “completed”) infinities are both acceptable; that was one of the contributions of Georg Cantor. If we had a recording of a man ascending the infinite ladder (which you say is logically permissible) would it be illogical to play this recording in reverse? No. With the exception of time-irreversable phenomena (which climbing a ladder is not) time-reversal is logically permissible. Inert Nothingness as the First Cause Problems creationists find with extensional realities and infinities are used to justify claims of the necessary existence of a “spaceless, timeless, and immaterial” First Cause; a role usually offered to a deity to play. This idea has no merit. Consider: if the First Cause is actually outside of time, then this First Cause cannot act. Actions, even as small as forming a thought, are embedded in time: there’s the moment before the action begins, the moment the action begins, and the moment the action ends or is completed. Actions imply “before” and “after” (as well as various kinds of “during”); all these terms are expressions of time. Therefore, a “timeless” First Cause is INERT. Consider: anything that takes up no space, and has no constituent parts or material is nothingness itself. Empty space is at least something: space. Material compressed into a dimensionless point is something: matter. Something that has neither material nor dimension is quite literally nothing. Therefore, an immaterial First Cause existing outside of space is NOTHINGNESS. Therefore, a timeless, immaterial First Cause existing outside of space is an INERT NOTHINGNESS. Are you sure you want to designate this Inert Nothingness as “God”? I will continue to read your other comments on infinites; there are other things I could comment on (such as the cosmos(*) being an uncountable infinity and what that means) but I’ll save that for another time. I hope you got some rest. I’m going to go and mow my lawn now, it’s gotten quite shaggy. sean s. sean samis
StephenB,
Under the circumstances, we don’t need to fall back on the mere probability of a cosmological big bang in order to argue for the existence of God
For probability to exist, one needs time. And time came into existence with the universe. It's been argued that there's another universe that includes time and chance, and "musta" had the ability to spawn new universes, namely ours. This is fantasy, not science.
It seems to me that if infinity cannot be instantiated in physical reality, then we do, indeed, have a slam-dunk argument for God as the first cause.
Yes. Our universe seems to have limits based on entropy and the Planck length, which is sort of a cosmological equivalent of a pixel. Logically, one would assume that the coordinates for any arbitrary point in space are irrational, however the Planck length terminates the number of digits--they're not infinite in digit length. -Q Querius
StephenB, Why exactly is an unlimited number of physical events logically impossible in the context of limited finite particles? I can understand that an unlimited number of different physical events is logically impossible, but how about an unlimited number of repetitive physical events? Origenes
Querius Yes, you have a point. Once the materialist opens the door to one absurdity, there is no reason to think he will stop at any absurdity. Also, I was hoping that Eric would comment. I am wondering if he agrees with me that an infinite number of physical events is logically impossible. If so, it seems that we have an unassailable proof for God's existence, given the fact that a finite universe logically requires an immaterial, personal, self-existent first cause. Under the circumstances, we don't need to fall back on the mere probability of a cosmological big bang in order to argue for the existence of God, which was Eric's main reason for saying that we don't have an air tight case. It seems to me that if infinity cannot be instantiated in physical reality, then we do, indeed, have a slam-dunk argument for God as the first cause. StephenB
StephenB @ 116, Good luck with that. When deterministic materialists demonstrate their willingness to ditch parsimony/Occam's Razor, Chaos theory, the limits of entropy, causality, and so on, I think we can be fairly certain that conjuring natural infinities will pose no problem for them as long as it rationalizes their ideology. -Q Querius
Harry @ 106, Yes, exactly! I was fortunate to have once taken a class in computer architecture from one of the guys who designed the Z-80. His final exam required us to complete a simple microprocessor that was incomplete at several levels of abstraction, which ranged from logic to microcode. And then to think how some people believe that life in a single cell, which is a billion, billion, billion, billion, billion, billion, billion, billion, billion, billion, billion, billion, billion or more times as complex as a microprocessor, could develop spontaneously is beyond all credibility! Yet they persevere. They have an unshakable faith in Time, Chance, and Necessity that somehow everything "musta" developed naturally, when even the universe could not have developed naturally since it could not have caused itself. And then they believe that their unsupported assertions and speculations somehow constitute irrefutable proof, as in "She's a witch!" When confronted with actual *experimental results* from the most intensely tested area (quantum mechanics) within the most fundamental of the sciences (physics), they prefer to cling to materialism despite scientific data to the contrary, believing that the non-existent natural laws of the non-existent universe somehow caused the universe to *pop* into existence from Nothing. Thanks for your great observations. -Q Querius
I would be interested in any well-thought out objections to the following proposition: actual infinity (as opposed to potential infinity) cannot be instantiated in physical reality [universe, multiverse, cosmos etc]. It would seem that an eternal universe is logically impossible in the context of limited finite particles and a limited number of physical events? StephenB
On a related note, as a caution against relying on the Big Bang as slam-dunk evidence for design, it may be worth highlighting a prior discussion. As referenced to mw above, in a prior discussion (relating to a different aspect of the Big Bang -- homogeneity of structure), I suggested design proponents be cautions about putting "all our marbles in the Big Bang basket." https://uncommondescent.com/intelligent-design/why-is-the-universe-not-homogeneous/#comment-443952 The key portion that relates to the present thread is this:
Ultimately, we have the following logical possibilities for the universe: 1. The universe always existed in some form. 1.a. The universe has developed to its current state through purely natural and material processes. 1.b. The universe has developed to its current state through processes that included, at least in part, intelligent intervention. 2. The universe has not always existed, but came into being at some point. 2.a. The universe came into being in a cataclysmic explosion along the lines of what is currently termed the Big Bang. 2.a.1. The universe then developed to its present state through purely natural and material processes (i.e., the naturalistic view of Big Bang cosmology). 2.a.2. The universe then developed to its present state through processes that included, at least in part, intelligent intervention. 2.b. The universe came into being through some process other than what is currently termed the Big Bang. 2.b.1. The universe then developed to its present state through purely natural and material processes. 2.b.2. The universe then developed to its present state through processes that included, at least in part, intelligent intervention.
Many individuals are currently arguing for 2.a.2. It may well be correct, but note that two other design-centric options are available: 1.b. and 2.b.2. The real question is whether the cosmos exhibit indicia of design (for example, the fine tuning of the constants or some other well-defined indicia). Those indicia are what would allow us to infer design, more so than whether there was a Big Bang, in and of itself. Eric Anderson
mw, thank you for your thoughtful comments. As to the Big Bang, you are correct that in this OP, for purposes of the discussion, I have assumed the Big Bang to be true. However, I have some personal intellectual unease about the Big Bang and recognize that there are some important open issues. For example, on other threads I have pointed out: https://uncommondescent.com/physics/the-big-bang-how-did-one-of-the-best-attested-theories-in-science-become-so-unpopular/ Look at my comments 16, 18, and 22. Also, https://uncommondescent.com/intelligent-design/should-christian-apologists-use-big-bang-as-evidence/ Comments 15 and 31. Finally, https://uncommondescent.com/intelligent-design/why-is-the-universe-not-homogeneous/ Comments 13, 19, 43, and (most of all) 44. Eric Anderson
HeKS;
I may be more irritable than normal. I worked 20.5 hours yesterday.
Good grief. Get some rest. This all can wait.
If you haven’t watched the Rowe / Grayling debate, I suggest you do so
I will make a point of that this weekend.
I also suggest that you read through the recent thread here on the impossibility of infinite past time. If you’re looking for my specific take on it then it is explained at some length in my comments in that thread.
Given there are more than 1400 comments, I won’t even try to read them all. But I will try to read yours. There seems to be only about 30 of them. Just looking at the OP, the objection to an “infinite past time” seems to rely on the idea of a “completed infinity of past events”: “an actually completed infinite past succession of finite causal steps” as the OP puts it. I think that the best response to this dilemma is to point out that all the problems with an “infinite past” for materialists are fully mirrored in an “infinite past” for deities. If a deity is capable of thought, contemplation, planning, etc, then each and every though is an event. If a deity is capable of action, each and every action is an event. The idea of an infinite past is actually unfathomable, as is nearly any absolute. The Problem of Evil is built on extrapolating from absolutes. (And no, I am not trying to inject the POE into this thread). I think we need to just say that when we talk about “infinite pasts” we are referring to something we really don’t understand. Invoking a deity does not make the incomprehensible suddenly comprehensible.
It should be noted that our ignorance about whether some other completely different combination of values might also allow for intelligent life if they were all adjusted in relation to each other does not remove the problem.
True, it does not remove the problem, but it does put some context around it which is valuable.
We can discern quite well what would happen if any one or a couple were slightly different than they actually are and the result is pretty uniformly a lack of any possibility for life.
What we cannot discern is whether these values actually are independent; Is it even possible for just one or a couple to be slightly different without altering the others. We don’t know.
We’re concerned with our local landscape of values and the vast majority of physicists accept that the fine-tuning issue is very real, which is why so many are starting to appeal to a multiverse.
I reject the term “fine tuning” but I acknowledge the problem is real. Though we are concerned with our local landscape, we cannot evaluate it thoroughly in isolation from a more general view of the landscape (borrowing your metaphor). Get some rest. This can wait. sean s. sean samis
Mung @45 Right you are science is in the business of observing, what causes what. Andre
Part 5 Theology, Summary It seems Sean has made his replies with reasonable points. His belief: evolutionary consensus science is equally as valid as faith-based systems and in relation to apparent fine tuning. Not quite. The J-C belief system is based on historic facts with faith. From Sinai, built on a humanly observable supernatural event coming together with human experience in order to impart divine knowledge. Neither the big bang scenario or the little cell scenario, have the slightest qualifications in those terms. The cosmic evolutionist tree and the tree of Darwin are imaginary constructs, grounded in godless knowledge, especially in J-C terms. If we do not have the full truth of the Word of an intelligent God to rely on, how can God judge us if His law says one thing then such theoretical science creates circumstances that totally corrupt divine law? However, Sean includes in his reply #68: _____________________________________________________ “I realize that it is widely used, but the term ‘fine tuning’ is conclusory; because it contains a verb form, it naturally implies a conclusion has been reached; that a particular design decision was made to ‘finely tune’ our universe. This should only be referred to as ‘apparent fine tuning’; the appearance of ‘tuning’ does not prove any fact of ‘tuning’. But of course, the ‘apparent’ part seems to be dropped.” _____________________________________________________ This seems to be an echo of Dawkins: deny design having the appearance of design is design. However, it may also appear, under such terms, that God has finely tuned the big bang theory, against what He clearly wrote, with an almighty big statement of truth. Thus, in that context, we may safely assume the figures of fine tuning are simply the left-overs from a miracle in six days. Still, in relation to biological evolutionary science, Sean's own word come to mind: ______________________________________________________ “So you all need to give science some time to do its work. Considering from where it started, evolutionary biology has made enormous strides.” (Sean, # 242: https://uncommondescent.com/philosophy/prominent-atheists-fundamentally-misunderstand-first-cause-arguments/#comment-612909) _______________________________________________________ In relation to the evolution of the cosmos, my reply has not changed: _______________________________________________________ “It seems that by such an admission, ‘give science some time to do its work,’ things are not going too well for consensus scientific evolution theory, and the work is lacking in clear evidence. Lacking a law. Lacking direct experimental reproducible unequivocal proof. Shall we “all” agree therefore; it is still at some type of faith stage in its major pillars of assertions.” _______________________________________________________ The echoes of Genesis remain, the beguilement of Satan still abounds, ‘yes, but six days really does not mean six.’ Eating that tree, for humans, has surely brought today intellectual crucifixion to “I am.” At Sinai, superior knowledge was given in stone and verbatim, as a light throughout time. It matters not that if people were relatively primitive, even if given to children. What matters is that divine law was given. Jesus confirmed we were generated as “gods,” children of the Most High (Ps 82:6-7), but we would die like men. Jesus reinforced that scripture by saying, “scripture cannot be broken.” (Jn 10:34-35) Note, the truth and accuracy in God’s statement; we would die, implying gods do not die, and, we would not die as animals, as we were not generated animal. Divine Justice and Mercy have operated regularly throughout the historical journey of the Judaeo-Christian faith. We should expect more of the same. In terms of the J-C movement, we will be judged by every word from the mouth of God: how could it be otherwise; not by the words of Darwin Hubble, Einstein or Guth. The Word of God said He ‘fine tuned' the cosmos in six days. Authenticated at Sinai in divine law. Who can prove the works of the Almighty! The same can be said for creation in six days. We have ‘only’ superior divine knowledge to rely on. “In the beginning was the word,” who said it was so, authenticated at Sinai, ratified in the divine flesh, and confirmed at the Transfiguration when speaking to Moses. Again face to face, clearly, as witnessed by Peter, James and John. “In the beginning....” (Gen 1:1) = “In the beginning....” (Jn 1:1). Both are equally valid scripture (2 Tim 3:16) inspired by God. As for Satan, a liar from the beginning, prince of this world, now lies about the beginning. Believe ourselves, our own knowledge, not the beginning of scripture, the alleged first book of the works of God. However, if we totally destroy the clear word of the Genesis Sabbath Commandment, which has been done, then ethically and morally, we should apply the same philosophy and principles to the rest of the Ten Commandments, and the two summary Commandments Jesus gave to crown in love the Ten, then fine tune with evolutionary thinking. However, Sean, #90: ____________________________________________________ “Fine tuning” has never been observed, it has only been inferred. Nobody we know of watched a designer “fine tune” any universe—much less ours; this is what an observation would require. _____________________________________________________ Let me see. We know of Jesus “I am” (Jn 8:58). And, “Through him all things were made” [in six days] (Jn 1:3), (Col 1:16) and (Exod 20:8-11). Ah, but that does not count because it is not right knowledge! Further, in human terms, it seems divine law and divine knowledge mutate. It would be more honest, scientific and accurate to say, humans mutate information, bearing in mind we have the same number as the Beast 666, in terms of spirit. I thank Eric for his wide-reaching OP, and Sean, a fair and reasonable interlocutor, for giving us an opportunity to consider the mysteries of the creation in various ways, and to try and make some sense out of seeing through a dark glass. These are my opinions and belief, given in the spirit of discussion. Still, many may be tempted to say, ‘how silly to’ “believe every word from out of the mouth of God” (ref Matt 4:4). mw
Part 4 The Big Bang Theory, an uncertainty By coincidence, at the time of writing, CMI posted: “Contrary to impressions given by the media, never before has there been so much evidence challenging this story. Such are the problems secular cosmologists have in trying to make their big bang models work, they end up making desperate appeals to the most unscientific notions imaginable. For example, in December 2014, top cosmologist Lee Smolin published a book suggesting that the only way to solve the big bang’s many scientific problems is to argue that the laws of physics were different in the past. (Smolin, L. & Unver, R.M., The Singular Universe and the Reality of Time: A Proposal in Natural Philosophy, Cambridge University Press, 2014.) So, when the facts don’t fit their creation story, secular scientists must turn to alternative ‘scientific laws’ drawn from their imaginations.” http://creation.com/atheists-creation-story Give are a few examples of big bang theory concerns: http://crev.info/2014/02/big-bang-answer/ http://www.icr.org/article/plasma-universe/ https://biblescienceforum.com/?s=the+big+bang http://creationtoday.org/the-big-bang-theory/ http://big-bang-theory.com/ How can a theoretical linear chaotic explosion, then suddenly change direction into rotational forces and produce supreme order in the universe? How did a star originate? What force compressed gaseous mass to form a star? Why do some planets and moons rotate in a different spin, to others? Consider the angular momentum of the sun relative to the planets in our solar system? There is a host of other major problems with the big bang theory. If some profess to stand on any one of the Ten Commandments, we cannot reject then the rest as not for today. Are we not in danger of bearing false witness against the Decalogue, do we not dishonour the word of God? Robbing Him of worship that should glorify Him each day by remembering what each miraculous day of creation represents, and with a remembered rest? Believing in the first miracle given by divine law must be worship. The creation event was confirmed at Sinai by God and ratified by Jesus as “I am” in the flesh. The God of Sinai walked the earth. We do well to remember what Jesus said face to face with the perfection of evil, and reflect on the manner how He dealt with Satan, that is, by challenging him in the rules of engagement: Jesus answered Satan, ‘It is written, “One does not live by bread alone, but by every word that comes from the mouth of God.”’ (Matt 4:4) If, “it is written,” is good enough for God, it should be good enough for those who profess to believe in that faith. However, if we claim one part of the general theory of evolution is wrong in relation to Darwinism, are we certain, therefore, that the same does not apply to the big bang theory? There is deemed irreducible complexity relative to biological systems, life units, life components and life forms; my hypothesis is, there may well be also cosmic irreducible complexity. Perhaps; delivering the immediate weight of the universe without an immediate sustaining force to keep everything falling through a bottomless space, and in need of immediate centripetal forces for rotational order and formations. No doubt, there may be better examples. mw
Part 3 Before the beginning It may be assumed, God planned the cosmos in eternal time and space, and then delivered all the material needed for the interrelated physical cosmos and spiritual cosmos, and at a specific instant in time; a delivery date appointed with the exact amount of goods needed, no more no less, exact: tuned to perfection. God cast into the vaults of heaven the creation. That is, into existing eternal space and time our space-time. It is believed God created matter from nothing, thus, nothing was second hand or recycled. Everything unique in all life forms. He could create any particle or particles of matter it any order He so wanted, and at whatever speed, however big or small. He did as He said. When we look up to a starry night and see that vast amount of energy and matter rotating in a boundless space, we are taking about power beyond our understanding. That if all such power came into existence in an instant, it could equally be fashioned in an instant, to create the sun and moon in His ordered fashion, and after the earth. Mature and fully functional. As with Adam Man. Mature at birth. My own opinion is that God created a unique impossible order relative to human science, to totally rebuke the evolutionist theories of today. God surely can create a planet in any order He wills and instantly. We are left with the effects, not of a scientific big bang, but of one big miracle, spread over six days for the sake of humans, not God! It is quite possible that God created a mature universe, but equally, it cannot be proved or disproved, only His Word is proof, passed down through the pages of time in an almost insignificant manner. Still, some people may want more proof. If Jesus/God “I am” is not proof enough, what would be? Nothing, not even if someone rose from the dead, and the word got around that such a thing really happened. mw
Part 2 In the beginning #22, BA77 quotes John 1:1-4, “In the beginning was the Word . . .” Eric, writes in the OP, “no-one has any real observational evidence as to the cause of the universe.” However, the exception is Jesus, Divine Man, a real man and God (“I am”). The First Cause, in J-C terms, provides true and valid observational evidence, indeed, written in the Ten Commandments as divine law, with all the might and weight of divine justice and divine truth. Jesus spoke twice of the beginning. “From the beginning of the creation, he made them male and female.” (Mk 10:6) And: Satan; “He was a murderer from the beginning and does not stand in the truth, because there is no truth in him. When he lies, he speaks according to his own nature, for he is a liar and the father of lies.” (Jn 8:44) The theology related to that teaching about Satan is for another time; otherwise, I will go off topic. Therefore, on the first teaching quote alone; if we add 13:69999 billion years to that, either Jesus is telling porkies (a blasphemy) or the big bang theory is not, cannot, and will not take into account the maturing effects of one miracle which is believed to have generated the cosmos in six days, that is according to divine law, and in plain speech, face to face with Moses. As an aside miracle: six jars of water, were turned into mature wine, instantly, and no expert could have told the difference it was an instant old! Does God deceive us therefore by any subjective appearance of age? No, He told us the truth, in stone, His Personal witness statement, that He created in six days, and to remember it every seven days. Two of the miraculously written stone tablets of the divine law were smashed in dismay and anger by Moses, seen by a developing nation and two miraculously more tablets were carried with utmost respect and honour in the Ark of the Covenant and carried in a prescribed manner on pain of death. We surely take a leap and a dose of naturalistic faith by believing God created by the big bang theory, when He stated clearly He did not. Still, we do not really know if we exist in a ready made, matured Cosmos, or have been abandoned by the same First Cause, nor know full well that we are truly sons and daughters of Adam and Eve, nor do we know truly we have fallen. The only way we know of such things is by direct divine law/divine revelation and by people documenting the humanly impossible. By divine information. Supernatural information. The big bang theory is not law. As anyone a spare space/cosmos to test the hypothesis, and who will supply the material? But we have calculations some may say. What we have with the naturalistic big bang theory, is reductionism to the ridiculous, whereby all space and time is contained in the cosmic cell, containing every particle in existence surviving from since who knows when theoretically contained in no space and heated up to hellish infinite temperatures. Therefore, as per evolutionist reductionism; reduce everything to the smallest, use your imagination, and everything works upwards devoid of God, the beginning of theoretical scientific naturalism. mw
A very clear and enjoyable post by Eric Anderson; well-reasoned, with pain-staking logic, and accompanied with thought provoking comments. ______________________________________________________ “However, I want to focus in this post on a specific aspect of the discussion, namely, some of the points raised by sean samis, starting @37 on that thread. In his comments, samis urges caution in drawing any conclusion from the Big Bang about deity’s existence or involvement.” ______________________________________________________ Of course, it may be said that we are arguing to some degree on Sean’s terms: naturalism. It seems to me, so far on this thread, no one has really objected to the big bang theory as a whole? The inference may be; in Judaeo-Christian terms, the Holy Trinity created the big bang with all its associated outcomes. Applying what God said to Isaiah (1:18), “Come now, let us argue it out, says the LORD. . .”. My comments are to be posted in five parts. These contributions are offered in the spirit of debate. Nothing more, nothing less. They are my beliefs, my opinions, offered in the spirit of discussion and debate. Part 1 The unseen God ______________________________________________________ Eric Anderson: “no-one has any real observational evidence as to the cause of the universe.” ______________________________________________________ # 22, BA77 quoted Job (38:19-20). God asks the big questions to Job, and which remain current for today: “Then the LORD answered Job out of the whirlwind . . . ‘Where were you when I laid the foundation of the earth? Tell me, if you have understanding. Who determined its measurements—surely you know!’” (Job 38:1–5) ‘Surely you know of big bang origins’, God is entitled to say. How true the following: “He has made everything suitable for its time; moreover, he has put a sense of past and future into their minds, yet they cannot find out what God has done from the beginning to the end.” (Eccl. 3:11) We cannot readily identify the work of God, in terms of a common occurrence, supernatural or otherwise: “Just as you do not know how the breath comes to the bones in the mother’s womb, so you do not know the work of God, who makes everything.” (Eccl. 11:5) And: “If the heavens above can be measured, and the foundations of the earth below can be explored, then I will reject all the offspring of Israel because of all they have done, says the LORD.” (Jer. 31:37) Indeed, God cannot be found by material calculations or physical instrumentation, therefore, it may be concluded God does not exist. In Judaeo-Christian (J-C) terms, God is spirit and we should worship Him in spirit and in truth (Jn 4:24), and remember the condition of humanity: “banished children of Eve.” Surely, it is God who chooses the method of how He may be experienced. If we could prove that God created by the big bang theory; or as written into divine law at Sinai, that God created in six days, we could prove God or His Word. Clearly, such would be delusions of grandeur. mw
Eric @103, Querius @105, Until one has some idea of how the computer's CPU and its instruction set work, has some notion of how the data moves to/from memory and between the CPU registers, and has done some programming in assembly language, and maybe written a high level language interpreter -- until then, computers might as well be magical and high level programming languages might as well be the special concoctions that make that magic happen. Naiveté about how computers really work engenders belief that there can really be such a thing as artificial intelligence and that computers actually think. Such is the naiveté that allows people to believe the assembly instructions for cellular machinery in the coding regions of DNA, and the cellular machinery that utilizes those assembly instructions, could have come about mindlessly and accidentally. harry
Eric, I completely agree. The typical approach as you describe is merely teaching motor skills and methods reminiscent of that produced by the industrial revolution or a B.F. Skinner. There's a profound amount of perspective that comes with a historical or principle approach---something that's applicable to computer science, software engineering, and the sciences, long after old "facts" and theories eventually die. The historical/principle approach is slower, but it provides a way for students to earn their education by thoroughly interacting with it. In my own experience, the courses I took that were organized in this way turned out to be my most valuable and enjoyable ones. -Q Querius
Sean, I'm not sure what you see funny in churning out vacuous, unsupported assertions, to which you now add random quotes. It's not science and you've demonstrated that you're following your ideology, not the data.
“Was the wave function waiting to jump for thousands of millions of years until a single-celled living creature appeared? Or did it have to wait a little longer for some highly qualified measurer – with a PhD?” -John Stewart Bell, 1981
John Stewart Bell's rhetorical question was provocative, if not ironic, von Neumann chains notwithstanding. By including this quote in your post, what are you saying that John Steward Bell was getting at? -Q Querius
Querius @92:
Programmers have told me that they’re essentially capturing a thought pattern of logical relationships. The rest is coding.
I like that. I have often wished that beginning computer courses would throw out the specific language and code for a while so that students can first understand what programming is really about. I've even toyed with an initial lesson curriculum that starts with a few stones and bowls on the table. If I had my wish I would start with that -- even spend several days on it. Then move into some fun history with Babbage and others, eventually hit binary, hexidecimal, do lots of "programming" with paper and pencil, and so on. There is a great deal of programming that can be learned -- real, foundational, exciting stuff -- without ever typing a single keystroke or without knowing a single command of higher-level language. Unfortunately, most beginning programming classes today just jump right in to some language and have the students learn to write "Hello World!" on the very first day. Gives them something tangible to grab onto right out of the gate -- makes them feel that they are making progress and doing some "real" programming. What I'd like to see is more fundamental, foundational, what-is-really-going-on-at-the-base-level kind of instruction. Eric Anderson
sean samis @87
I can see from HeKS’s comments here that, on the other thread I have annoyed him. That certainly was not intentional;
Fair enough. I may be more irritable than normal. I worked 20.5 hours yesterday. A few quick comments about your post because I don't have time to respond in any detail. If you haven't watched the Rowe / Grayling debate, I suggest you do so, as the argument is laid out a bit more there. I was not attempting in my OP to fully elucidate the argument but merely to comment on Coyne's inept response to it. I also suggest that you read through the recent thread here on the impossibility of infinite past time. If you're looking for my specific take on it then it is explained at some length in my comments in that thread. Finally, regarding the term "Fine-Tuning", I can't agree with your take on the issue. "Fine-Tuning" does not mean "design". If it did, the argument would be circular, but it isn't. This is why you'll see W.L. Craig address this very issue in virtually every debate. The meaning of "fine-tuning" in this case simply means that the laws and constants have specific values out of much larger possible ranges and the values they have are conducive to the existence of intelligent life while the vast swaths of other values would not be conducive to the existence of intelligent life [1]. The term "fine-tuning" is agnostic with respect to the actual cause of the fine-tuning. The term itself does not militate against a natural explanation. Take care, HeKS [1] It should be noted that our ignorance about whether some other completely different combination of values might also allow for intelligent life if they were all adjusted in relation to each other does not remove the problem. We can discern quite well what would happen if any one or a couple were slightly different than they actually are and the result is pretty uniformly a lack of any possibility for life. We're concerned with our local landscape of values and the vast majority of physicists accept that the fine-tuning issue is very real, which is why so many are starting to appeal to a multiverse. Those that have rejected the reality of the fine-tuning, such as Victor Stenger, have been taken to task for the fact that in claiming to have dispatched the problem they have only demonstrated that they never even understood the problem in the first place. HeKS
Querius; regarding #91;
Sean, Does it even occur to you that your entire argument in 89 consists of unsupported assertions? Go read your post again. Is there a single quote from a quantum physicist? Any links to papers? How about live interviews with qualified researchers? No?
I actually LOL’d when I read this. I really did. In response: Querius; does it even occur to you that your entire argument in 66 (to which my comment #89 is a reply) consists of unsupported assertions? Go read your post again. Is there a single quote from a quantum physicist? Any links to papers? How about live interviews with qualified researchers? No? If you don’t need those things, why would I? You want references? OK: "Of course the introduction of the observer must not be misunderstood to imply that some kind of subjective features are to be brought into the description of nature. The observer has, rather, only the function of registering decisions, i.e., processes in space and time, and it does not matter whether the observer is an apparatus or a human being; but the registration, i.e., the transition from the "possible" to the "actual," is absolutely necessary here and cannot be omitted from the interpretation of quantum theory." - Werner Heisenberg, Physics and Philosophy, p. 137 "Was the wave function waiting to jump for thousands of millions of years until a single-celled living creature appeared? Or did it have to wait a little longer for some highly qualified measurer - with a PhD?" -John Stewart Bell, 1981, Quantum Mechanics for Cosmologists. In C.J. Isham, R. Penrose and D.W. Sciama (eds.), Quantum Gravity 2: A second Oxford Symposium. Oxford: Clarendon Press, p.611. Jaeger, Gregg (September 2014). "What in the (quantum) world is macroscopic?". American Journal of Physics 82 (9): 896–905. Bibcode:2014AmJPh..82..896J. doi:10.1119/1.4878358. Bell, John (2004). Speakable and Unspeakable in Quantum Mechanics: Collected Papers on Quantum Philosophy. Cambridge University Press. p. 170. ISBN 9780521523387. Feynman, Richard (2015). The Feynman Lectures on Physics, Vol. III. Ch 3.2: Basic Books. ISBN 9780465040834. As a result of this, my conclusion is that you are not aware of the basic facts QM, nor do you have the slightest intention of following the data. From here on, you can do your own homework. sean s. sean samis
Phinehas; regarding #83
Me: “Fine Tuning” is a conclusory term and should not be used.
Could we say the same of “natural selection?”
Perhaps. Do you have a better term, a neutral term to suggest? If not, we’ll have to stick with “natural selection”. sean s. sean samis
We can conclude that God exists without appealing to science or any probabilistic arguments based on quantitative measurements. Let’s think it through. The cosmos cannot be eternal because it is impossible to instantiate infinity in physical reality. By its very nature, physical reality can be measured and its units can be counted. Infinity cannot be measured or counted. Only finite quantities can be measured and counted. Under the circumstances, there cannot be an infinite number of particles, or an infinite amount of energy, or an infinite number of physical events that have occurred in the physical cosmos. So, the cosmos (universe or multiverse) must be temporal. Even if infinity could be counted, an eternal universe would indicate that an infinite number of physical events had already happened at the earliest point of human history, which means that we could never have arrived at this point, which is absurd. Even if we ignore that impossible barrier and say we arrived anyway, each new physical event would be infinity + the cumulative total and ever-increasing number of successive events, which is equally absurd. Since the cosmos cannot be eternal, time, space, and matter came into being. Since time, space and matter cannot bring themselves into being, the first cause that produced them must be a timeless, spaceless, immaterial being. The first cause must also be a person. The Cosmos is contingent. It did not have to exist. Thus, the immaterial first cause of the cosmos must be a supernatural person with a free will because only such a person can freely decide to leave nothingness as it is or create a contingent physical cosmos out of nothing. This supernatural person we call God. StephenB
Again, the substance on the table: https://uncommondescent.com/atheism/fyi-ftr-luke-barnes-on-fine-tuning-and-the-case-of-the-fine-structure-constant/ kairosfocus
Eric Anderson,
EA: I’m curious, however, about your use of the word “fundamental,” and then applying that concept to parts. Surely if there can be one eternal, uncaused entity there could be more than one? There is no principle of logic that dictates there can only be one.
If the First Cause consists of distinct parts, which explain the First Cause, then the First Cause is not (explanatory) fundamental. In that scenario their would be parts which are fundamental to the First Cause. IOWs their would be parts which explain/cause the First Cause. Such a state of affairs would be incoherent. Therefore, the First Cause cannot consist of parts. Therefore, the First Cause must be one and indivisible. - - - - - Also, the First Cause is (explanatory) fundamental, so we cannot situate the First Cause within a context (e.g. matter, space, time and laws), which the First Cause does not explain. IOWs the First Cause cannot be encapsulated by anything which is not explained/caused by the First Cause. Therefore, the First Cause can operate outside time and space. Origenes
PS: John Leslie:
One striking thing about the fine tuning is that a force strength or a particle mass often appears to require accurate tuning for several reasons at once. Look at electromagnetism. Electromagnetism seems to require tuning for there to be any clear-cut distinction between matter and radiation; for stars to burn neither too fast nor too slowly for life’s requirements; for protons to be stable; for complex chemistry to be possible; for chemical changes not to be extremely sluggish; and for carbon synthesis inside stars (carbon being quite probably crucial to life). Universes all obeying the same fundamental laws could still differ in the strengths of their physical forces, as was explained earlier, and random variations in electromagnetism from universe to universe might then ensure that it took on any particular strength sooner or later. Yet how could they possibly account for the fact that the same one strength satisfied many potentially conflicting requirements, each of them a requirement for impressively accurate tuning? [Our Place in the Cosmos, 1998] and also: . . . the need for such explanations does not depend on any estimate of how many universes would be observer-permitting, out of the entire field of possible universes. Claiming that our universe is ‘fine tuned for observers’, we base our claim on how life’s evolution would apparently have been rendered utterly impossible by comparatively minor alterations in physical force strengths, elementary particle masses and so forth. There is no need for us to ask whether very great alterations in these affairs would have rendered it fully possible once more, let alone whether physical worlds conforming to very different laws could have been observer-permitting without being in any way fine tuned. Here it can be useful to think of a fly on a wall, surrounded by an empty region. A bullet hits the fly Two explanations suggest themselves. Perhaps many bullets are hitting the wall or perhaps a marksman fired the bullet. There is no need to ask whether distant areas of the wall, or other quite different walls, are covered with flies so that more or less any bullet striking there would have hit one. The important point is that the local area contains just the one fly.
It is not a good, empirically well supported explanation to suggest arrival at a deeply isolated operating point by chance, absent direct evidence of adequate search resources and opportunity. The observational evidence of a quasi-infinite multiverse is effectively nil. Where, the carpeted portion of the Leslie wall, on recent discussion, is, a Boltzmann brain popping up then vanishing by fluctuation. The deeply isolated island of complex organised information-rich function does not go away so easily. PPS: It seems we can identify a fallacy of dismissal by word-count. A substantial point takes reasonable space, and is not to be got rid of by clipping and pasting into a word-count app. Twitter-length headline/ comment/ textual soundbite/ snidely dismissive talking point is not generally a sound basis for responsible decision and action. (Especially, when it is little more than rhetorical appeal to prejudice and/or party-spirit driven polarisation.) PPPS: Nor is citation of a responsible or significant source on a point an empty argument. Acknowledging intellectual source is an intellectual virtue not a vice, and it is the facts and logic in play that need to be substantially addressed. kairosfocus
F/N: As fine tuning is being rhetorically dismissed here on one excuse or another, I point to a current FTR: https://uncommondescent.com/atheism/fyi-ftr-luke-barnes-on-fine-tuning-and-the-case-of-the-fine-structure-constant/ The issue is plainly on the table, and the rhetorical pattern in too many attempts to avoid or sideline it in the end tell us more about its strength and the unwelcome nature of what it points to than anything else. Where, actually, fine tuning of systems that sets them at operating points is a commonplace feature of designed entities, with millions of examples, starting with telecommunications equipment. As Barnes and Hoyle and others have highlighted, we see no super-physics that sets the frame of our physics where it is in parameter space, and it can be added that such a super-physics would in turn be fine tuned at the next level. So, there is a relevant empirical base, that points to design as reliable cause of fine tuned systems. There is a good logical-analytical reason to see that we are in a lone fly swatted situation rather than one amidst a carpet of flies one. That is very suggestive indeed and calls for a sensible inference to the best explanation. Where, design is the candidate to beat, absent ideologically imposed, lab coat clad a priori, self-refuting -- so, necessarily false -- evolutionary materialism. KF kairosfocus
Phinehas @ 83, Haha, nice one! No response naturally. ;-) -Q Querius
Eric, I thought harry's post in 59 makes some good points. I agree that we can make few if any inferences about the extra- or super- something that was the cause of our universe. Compared to our existence it must be extremely powerful. But whether it consists of the same kind of space-time, mass-energy, with the same physical laws--gravity, quantum mechanics, dark energy, and what not is actually doubtful when you consider that one of it's properties is to be able to spawn a universe such as ours. My conscious action can collapse wavefunctions into mass-energy, but I can't run out into the night shouting "let there be light" and have anything happen (well, beyond irritated neighbors turning on their lights). But according to people who actually know something about quantum mechanics, wavefunctions were indeed generated and collapsed to form our universe. The wavefunctions, as probability waves, could have found a comfortable home in any spreadsheet. Maybe they did. So turning to the Kalam, one could claim that ultimately there must be an uncaused cause, a prime cause. It does not good to argue that an UCC/PC is not acceptable on the ground of infinite regression, because here we are. Furthermore, there is no evidence that suggests that the UCC/PC is more than one step removed from our universe on account of parsimony. What is intriguing to think about is that since our consciousness, which seems to have the power of collapsing wavefunctions, does have a small amount of transcendent control, it might actually belong to the extra- super- existence outside our universe! Maybe our spirits do sit in heavenly places! -Q Querius
Dionisio, What I've observed about information is the parallels to our natural world. This includes its fractal and chaotic nature--it seems to grow in dendritic patterns of abstraction levels, and you cannot predict what will turn out to be critically important. Programmers have told me that they're essentially capturing a thought pattern of logical relationships. The rest is coding. That professor Vedral recognized the binary nature of the universe seems really significant to me, although I don't understand it. That entangled information seems to be able to change instantly regardless of locality reminds me of the changes that cascade through a spreadsheet. And then there's the dynamics of parametric metadata that throws a whole new level of information about information! As I said, I admit that I really don't understand information theory, but I sometimes wonder whether there isn't a lot more to it. For example, unprovenanced antiquities may not be any different physically, but in the absence of Context, they lose most of their value. Just some thoughts to add to yours. -Q Querius
Sean, Does it even occur to you that your entire argument in 89 consists of unsupported assertions? Go read your post again. Is there a single quote from a quantum physicist? Any links to papers? How about live interviews with qualified researchers? No? All we get is assertions that are logically equivalent to saying "She's a witch." My conclusion as a result is that you're not aware of the latest discoveries in QM, nor do you have the slightest intention of following the data. -Q Querius
KF @ 75;
SS, Before going further, I suggest you answer substantially to Sir Fred Hoyle: [650 word cite omitted]
As a rule, I don’t debate dead men. They cannot comment on my answers or reply to my questions. If you have questions, KF, ask.
PS: Your discomfort with the widely recognised observations on fine tuning of physics that supports cell based life is inadvertently revealing. BTW, sweet spots are a manifestation of fine tuning.
Fine tuning” has never been observed, it has only been inferred. Nobody we know of watched a designer “fine tune” any universe—much less ours; this is what an observation would require. Sweet spots could be a manifestation of design (a point I acknowledged) but they could also just be chance. We might be able to test the later supposition, we can’t test the former. sean s. sean samis
Querius @ 66;
... you’ll have no trouble following the data from quantum mechanics to the demonstrated conclusion that “natural” reality as it is commonly thought of doesn’t actually come into existence apart from observation and measurement. Right?
The “observer effect” in QM is a really poorly chosen term that leads to major confusion. It’s a common misconception that quantum mechanics requires the mind of a conscious observer to cause reality to come into existence. That is totally wrong; it does not and never has. Standard quantum mechanics does not care if an experimenter is watching their experiment, or if they leave the room and leave the “observing” to inanimate detectors and recording devices. It is the inanimate objects which amplify the microscopic events into macroscopic measurements and then record them. As long as this process is time-irreversible, the quantum event is recorded and the experimenter will eventually see the results. In the case of the LHC, results are recorded and not even reviewed for months. These inanimate devices are not magical. They are just inanimate objects carefully arranged so as to react to quantum events in ways that allow experimenters to know what happened. Out in the world, naturally arranged inanimate objects similarly react at the quantum level; reality does not need some person observing it to come into existence; quantum events resolve themselves all the time. I realize that for a believer, the observer effect is damned tempting, but you must resist it because it’s a false lead. sean s. sean samis
Origenes @ 40:
I’m no physicist so the following question may very well show my ignorance, but how can an “eternal universe” not be in a state of “heat death” — a state of no thermodynamic free energy? How can it be that in an eternal universe suns have not already died?
Suns have already died in our universe so I take this to ask how it could be that all suns have not died. Since no one is suggesting that our universe is actually eternal, I take this question to be about whatever multiverse/cosmic-bubble/etc. that might have created our universe; how could such a “universe” have avoided heat-death? We don’t know. However, we also don’t know that the concept of entropy even applies to such a multiverse/cosmic-bubble/etc. sean s. sean samis
I can see from HeKS’s comments here that, on the other thread I have annoyed him. That certainly was not intentional; if anything I try to avoid annoying people because it just gets in the way of productive discussion. A particular problem seems to be some disagreement about what exactly was the argument in HeKS’ OP. In #8 (on this thread) HeKS wrote;
I was commenting on the lack of validity in Sean Samis’ response to the issues addressed in my OP. ...
As HeKS wrote, there were multiple issues addressed in the OP. As for which is the most due a response, I will defer to what appears to be HeKS’ thought on the matter, from comment # 8:
As you say in your article:
In his comments, samis urges caution in drawing any conclusion from the Big Bang about deity’s existence or involvement.
Any attempt to counter the argument described in my OP by appealing to the possibility of something like a mulitverse or any other higher level spacetime or material state preceding the Big Bang is doomed to failure because the argument says nothing about the Big Bang in particular. The argument is that anything that is extended in time and/or space, which includes both time and space themselves, cannot have existed infinitely into the past. ... Emphasis added.
In the commentary that follows HeKS’ OP and up to comment 260, the letter string “extension” was used 39 times, but there was never an explanation of why extentional objects cannot also be eternal. I’ve not found words to the effect that: “all things that are extensional are finite and cannot ever transition from being finite to being infinite, which means that they cannot occupy an infinite amount of space and they cannot exist for an actually infinite amount of time” BECAUSE of [insert missing rationale here]. The claim about extensional reality is asserted repeatedly, but these claims are never explained or justified. I reread the comments on the other thread (and those here) and I cannot find anyone who actually explains it. The closest on HeKS’ thread was a comment by KF: (#164)
Seversky, the classical theistic view has long since been stated, the world was created from no material or quasi-material predecessor. Where, nothing, properly, denotes non-being. And, even through multiverse speculations, it remains so that no material atom based entity or extensional entity like that can but be contingent. Which is not self explanatory. Emphasis added.
“... not self-explanatory”. Indeed; it is profoundly obscure. In comment 8, HeKS continues further on,
Sean’s appeals to the idea that the First Cause must have just been some other “thing” made of “stuff”, just not stuff from our universe, utterly fails to respond to the argument. Emphasis added
As far as I can see, there’s no argument to respond to. There are just bare, unfounded claims; that is not an argument. I have responded to the claim, and I’d love to respond to the rationale, but it’s nowhere I can find it. And then:
But as I’ve said above, [scientific explanations] require some extensional reality existing infinitely into the past, which is simply not possible. And so the argument educates “science” by informing it that we are forced to conclude that there was some point at which there was either a) absolutely nothing at all or b) some non-extensional reality with the characteristics described in my OP, and logic forces us to conclude that it was (b).
The force of this claim is severely attenuated by the simple fact that the premise remains unfounded. Why is it true that “some extensional reality existing infinitely into the past ... is simply not possible”? So what is the explanation? Without one, all we have is a pile of assertions; a house built on sand. The closest HeKS has come to justifying this claim is in comment # 1 on this thread in which HeKS wrote about his OP;
“However, the argument presented by Rabbi Rowe was not the Kalam Cosmological Argument but the Argument from Extensionality. The latter works like the Kalam applied to any physical spacetime, like making the second premise of the Kalam (“the universe began to exist”) operate on a variable in place of the term “universe”, so that it becomes “X began to exist”. As such, the Argument from Extensionality is agnostic regarding whether or not our universe is the only physical spacetime and its conclusion regarding the need for a non-extensional First Cause holds with equal force regardless of whether or not our universe exists within any kind of higher level super-space... ” Emphasis added.
I am familiar with W.L. Craig’s formulation of the Kalam, it is severely defective; what I know of the Argument from Extensionality (Calosi and Tarozzi) does not really apply here or compel the conclusions HeKS insists on. Perhaps someone could explain how it works here? Perhaps this conversation would be more productive if HeKS stated his thesis clearly and justified his reasoning. Short of that, confusion and futile commentary are likely. Regarding,
I would love to engage Sean Samis on his arguments because I don’t think any of them hold up at all, but as I said in comment #1 of my OP, I simply don’t have the time to engage right now because of my work schedule. Maybe in a couple weeks I’ll have some time to revisit the issue. ... If only I had more time on my hands.
I understand that, I’m wedging my writing in-between things too. I look forward to being able to continue this discussion as time permits. Fortunately, the topics are not perishable. I will do my best to not annoy, but I will not simply accept unexplained assertions as facts or reasoning. And I expect to be held to the same standard. That’s only fair. sean s. sean samis
KF @53: Your point is well taken and I mean no disrespect to the principles passed down through the ages. I am trying to home in on a careful distinction between what can be deduced versus what can be inferred, between what is an absolute necessity versus what is a reasonable possibility, between what must be versus what we have rational warrant to think is. Eric Anderson
Origenes @81:
EA: We cannot say, it seems to this author, as a matter of logic and deduction that the First Cause is God. Origenes: I don’t agree. There are excellent arguments which show that the First Cause is very likely God.
Of course you agree. :) Notice the use of your word "likely." I have in my discussion, I hope, carefully drawn a distinction between (a) what can be deduced as a matter of pure logic, and (b) what can be reasonably inferred based on various bits of evidence. Bare logic and deduction do not get us to God as the First Cause. But, as I concluded in my essay, God might well be a reasonable inference based on clues and the evidence. Indeed, likely (to borrow your word) a better candidate than the various materialistic notions.
EA: We cannot even say that the universe was caused by the First Cause, rather than some intermediate cause. Origenes: I agree, but this doesn’t weaken the case for the supernatural nature of the First Cause.
Agreed. I don't think it speaks to it one way or another.
EA: Indeed, as a matter of dispassionate objective scientific inquiry and reasoning, we can say but very little about the First Cause. Origenes: You seem to think that arguments concerning the First Cause depend on whether it is the direct cause of the universe or not. If so, I don’t agree with that notion.
I'm not proposing that directly. Let me see if I can better explain. There are two things: First is the question of whether there is a First Cause. That does not depend on whether the First Cause is the direct cause of the universe. I agree with you there. One way or another we get back to a First Cause. Materialist and theist alike -- everyone has to go back to a First Cause. Second is the question of particular characteristics of the First Cause. Many -- no doubt not all, but many -- of the the arguments I've heard over the years about this or that characteristic of the First Cause depend on a negation of what our universe is like. Such arguments rest on an assumption that the characteristics of our universe cannot have existed prior to our universe. So the alleged characteristics of the First Cause then end up being just a list of negations of characteristics of our universe: our universe has space, mass, energy, etc., so, the thinking goes, the First Cause must not have any of those characteristics. Eric Anderson
Origenes @52: I'm not sure we are that far apart. In the part you quoted I was referring to the cause of universe, not the First Cause. I'm curious, however, about your use of the word "fundamental," and then applying that concept to parts. Surely if there can be one eternal, uncaused entity there could be more than one? There is no principle of logic that dictates there can only be one. Eric Anderson
“Fine Tuning” is a conclusory term and should not be used.
Could we say the same of "natural selection?" Phinehas
EA, it is not the best way to interject a potential infinity of intermediates without warrant. Perhaps, use world in the wider sense to mean the domain of reality that enfolds our observed cosmos. Then, ask, what is the root of the world. KF kairosfocus
Eric Anderson, I agree with all these points:
EA: (1) An inference, from observable facts, that the universe had a beginning. (2) A deduction that the universe had a cause. (3) A deduction that the cause was not within the universe itself (i.e., existed outside of the universe, both spatially and temporally). (4) An inference, from observable facts, that the universe has been finely tuned. (5) A deduction that the cause was capable of producing the universe and of finely tuning the constants. Most everyone is in agreement up to this point. One additional item that everyone should agree on is the following: (6) Ultimately, when traced back, there must be a First Cause – that which existed in and of itself, without a beginning. It is true that whether the universe was caused by the First Cause or by some intermediate cause is entirely open to question. However, at some point, we must regress to a First Cause.
The fact, that it is entirely open to question whether the universe was created by the First Cause or by some intermediate cause, makes discussing the First Cause an entirely independent matter from discussing the cause of the universe. In fact, when Aquinas contemplated the First Cause, he did not consider the Big Bang. But you don't treat the 'First Cause' and 'the cause of the universe' as separate issues. You mix things up:
EA: Identifying the First Cause, unfortunately, is a trickier matter. The Nature of the First Cause A number of proposals might be put forward, but let us focus on the two most common. One proposal on the table is that the First Cause was a purely naturalistic phenomenon: some unidentified, never-before-seen, essentially indescribable, powerful phenomenon, that coincidentally (through sheer luck or sheer repetition over time) managed to produce the finely-tuned universe in which we find ourselves.
What is being discussed here? The 'First Cause' or 'the cause of the universe'? With the arguments provided by Aquinas and others it's easy to show that a purely naturalistic phenomenon fails horribly as a First Cause, but it can indeed qualify as a cause (intermediate cause) of the universe.
EA: The inference that the universe had a beginning does not allow us to identify the First Cause.
I agree. It can be some intermediate cause.
EA: We cannot say, it seems to this author, as a matter of logic and deduction that the First Cause is God.
I don't agree. There are excellent arguments which show that the First Cause is very likely God.
EA: We cannot even say that the universe was caused by the First Cause, rather than some intermediate cause.
I agree, but this doesn't weaken the case for the supernatural nature of the First Cause.
EA: Indeed, as a matter of dispassionate objective scientific inquiry and reasoning, we can say but very little about the First Cause.
You seem to think that arguments concerning the First Cause depend on whether it is the direct cause of the universe or not. If so, I don't agree with that notion. Origenes
Querius @51:
Since space-time had its beginning with the big bang . . .
Ah, but that it precisely the assumption that makes the description of the cause circular. Our space-time may have begun with the Big Bang, but we have already claimed that something exists outside of our space-time.
Why would you say it’s circular? If space-time had a beginning, it had a primary cause. That primary cause could not possibly have existed within space-time, otherwise a non-existent space-time managed to cause itself to exist.
Yes, if our space-time had a beginning, it had a cause. But when you say the “primary cause could not possibly have existed within space-time,” you are making a critical unstated assumption: our universe is all that there is; nothing exists except in the context of our universe (which has certain characterstics, such as space-time). And yet at the very same time we are arguing that something does exist outside of our universe.
All this is easily imaginable and possible with virtually none of the complexity of our current understanding of physics, chemistry, and biology. It’s far more likely that existence outside our space-time is far more complex. Try imagining two or more time dimensions “orthogonal” to each other . . .
Exactly. Another dimension, another plane of existence . . . something. Some state of existence. I fully agree. And it is a state of existence about which we have essentially no observational information. All we know for sure is that it is some type of existence; that it is not nothing; that it has some kind of characteristics. So when we claim that it has none of the characteristics of our universe simply because it is not in our universe we are going a bridge too far and going beyond what can be deduced from the evidence. Eric Anderson
It is really not all that surprising that an atheist would insist fine-tuning is illusory instead of real since everything within the atheistic worldview, even the atheist himself, turns out to be illusory:
Why Evolutionary Theory Cannot Survive Itself – Nancy Pearcey – March 8, 2015 Excerpt: To make the dilemma even more puzzling, evolutionists tell us that natural selection has produced all sorts of false concepts in the human mind. Many evolutionary materialists maintain that free will is an illusion, consciousness is an illusion, even our sense of self is an illusion — and that all these false ideas were selected for their survival value. So how can we know whether the theory of evolution itself is one of those false ideas? The theory undercuts itself.,,, Of course, the atheist pursuing his research has no choice but to rely on rationality, just as everyone else does. The point is that he has no philosophical basis for doing so. Only those who affirm a rational Creator have a basis for trusting human rationality. http://www.evolutionnews.org/2015/03/why_evolutionar094171.html Atheistic Materialism - Where All of Reality Becomes an Illusion - video https://www.facebook.com/philip.cunningham.73/videos/1213432255336372/ Darwinian evolution, and atheism/naturalism in general, are built entirely upon a framework of illusions and fantasy https://docs.google.com/document/d/1Q94y-QgZZGF0Q7HdcE-qdFcVGErhWxsVKP7GOmpKD6o/edit
Humorously, it is precisely because of fine-tuning of the universe that the 'illusory' atheist, if he actually existed as a real person, would be forced to believe that pink unicorns are real, i.e. would be forced to believe that illusory entities are real.
Fine Tuning, Multiverse Pink Unicorns, and The Triune God – video https://www.facebook.com/philip.cunningham.73/videos/vb.100000088262100/1145151962164402/?type=2&theater Why Most Atheists Believe in Pink Unicorns - May 2014 Excerpt: Given an infinite amount of time, anything that is logically possible(11) will eventually happen. So, given an infinite number of universes being created in (presumably) an infinite amount of time, you are not only guaranteed to get your universe but every other possible universe. This means that every conceivable universe exists, from ones that consist of nothing but a giant black hole, to ones that are just like ours and where someone just like you is reading a blog post just like this, except it’s titled: “Why most atheists believe in blue unicorns.” By now I’m sure you know where I’m going with this, but I’ll say it anyway. Since we know that horses are possible, and that pink animals are possible, and that horned animals are possible, then there is no logical reason why pink unicorns are not possible entities. Ergo, if infinite universes exist, then pink unicorns must necessarily exist. For an atheist to appeal to multiverse theory to deny the need of a designer infers that he believes in that theory more than a theistically suggestive single universe. And to believe in the multiverse means that one is saddled with everything that goes with it, like pink unicorns. In fact, they not only believe in pink unicorns, but that someone just like them is riding on one at this very moment, and who believes that elephants, giraffes, and zebra are merely childish fairytales. Postscript While it may be amusing to imagine atheists riding pink unicorns, it should be noted that the belief in them does not logically invalidate atheism. There theoretically could be multiple universes and there theoretically could be pink unicorns. However, there is a more substantial problem for the atheist if he wants to believe in them and he wants to remain an atheist. Since, as I said, anything can happen in the realm of infinities, one of those possibilities is the production of a being of vast intelligence and power. Such a being would be as a god to those like us, and could perhaps breach the boundaries of the multiverse to, in fact, be a “god” to this universe. This being might even have the means to create its own universe and embody the very description of the God of Christianity (or any other religion that the atheist otherwise rejects). It seems the atheist, in affirming the multiverse in order to avoid the problem of fine-tuning, finds himself on the horns of a dilemma. The further irony is that somewhere, in the great wide world of infinities, the atheist’s doppelganger is going to war against an army of theists riding on the horns of a great pink beast known to his tribesman as “The Saddlehorn Dilemma.” https://pspruett.wordpress.com/2014/05/12/why-most-atheists-believe-in-pink-unicorns/
So basically, fine-tuning forces illusions (atheists) to believe that they are having illusions (pink unicorns)! :) Edgar Allen Poe would be very pleased:
Take this kiss upon the brow! And, in parting from you now, Thus much let me avow- You are not wrong, who deem That my days have been a dream; Yet if hope has flown away In a night, or in a day, In a vision, or in none, Is it therefore the less gone? All that we see or seem Is but a dream within a dream. I stand amid the roar Of a surf-tormented shore, And I hold within my hand Grains of the golden sand- How few! yet how they creep Through my fingers to the deep, While I weep- while I weep! O God! can I not grasp Them with a tighter clasp? O God! can I not save One from the pitiless wave? Is all that we see or seem But a dream within a dream? - Edgar Allan Poe
bornagain77
Headlined FTR: https://uncommondescent.com/atheism/fyi-ftr-luke-barnes-on-fine-tuning-and-the-case-of-the-fine-structure-constant/ --> notice, the key illustration on the fine structure constant and the strong nuclear force kairosfocus
F/N: Luke Barnes has a useful semi-pop summary:
http://www.thenewatlantis.com/publications/the-fine-tuning-of-natures-laws Today, our deepest understanding of the laws of nature is summarized in a set of equations. Using these equations, we can make very precise calculations of the most elementary physical phenomena, calculations that are confirmed by experimental evidence. But to make these predictions, we have to plug in some numbers that cannot themselves be calculated but are derived from measurements of some of the most basic features of the physical universe. These numbers specify such crucial quantities as the masses of fundamental particles and the strengths of their mutual interactions. After extensive experiments under all manner of conditions, physicists have found that these numbers appear not to change in different times and places, so they are called the fundamental constants of nature. These constants represent the edge of our knowledge. Richard Feynman called one of them — the fine-structure constant, which characterizes the amount of electromagnetic force between charged elementary particles like electrons — “one of the greatest damn mysteries of physics: a magic number that comes to us with no understanding by man.” An innovative, elegant physical theory that actually predicts the values of these constants would be among the greatest achievements of twenty-first-century physics. Many have tried and failed. The fine-structure constant, for example, is approximately equal to 1/137, a number that has inspired a lot of worthless numerology, even from some otherwise serious scientists. Most physicists have received unsolicited e-mails and manuscripts from over-excited hobbyists that proclaim, often in ALL CAPS and using high-school algebra, to have unlocked the mysteries of the universe by explaining the constants of nature. Since physicists have not discovered a deep underlying reason for why these constants are what they are, we might well ask the seemingly simple question: What if they were different? What would happen in a hypothetical universe in which the fundamental constants of nature had other values? There is nothing mathematically wrong with these hypothetical universes. But there is one thing that they almost always lack — life. Or, indeed, anything remotely resembling life. Or even the complexity upon which life relies to store information, gather nutrients, and reproduce. A universe that has just small tweaks in the fundamental constants might not have any of the chemical bonds that give us molecules, so say farewell to DNA, and also to rocks, water, and planets. Other tweaks could make the formation of stars or even atoms impossible. And with some values for the physical constants, the universe would have flickered out of existence in a fraction of a second. That the constants are all arranged in what is, mathematically speaking, the very improbable combination that makes our grand, complex, life-bearing universe possible is what physicists mean when they talk about the “fine-tuning” of the universe for life.
That's the issue in broad overview, from one angle. Barnes adds some details that we can ponder for a moment:
. . . we can calculate all the ways the universe could be disastrously ill-suited for life if the masses of these [fundamental] particles were different. For example, if the down quark’s mass were 2.6 x 10-26 grams or more, then adios, periodic table! There would be just one chemical element and no chemical compounds, in stark contrast to the approximately 60 million known chemical compounds in our universe. With even smaller adjustments to these masses, we can make universes in which the only stable element is hydrogen-like. Once again, kiss your chemistry textbook goodbye, as we would be left with one type of atom and one chemical reaction. If the up quark weighed 2.4 x 10-26 grams, things would be even worse — a universe of only neutrons, with no elements, no atoms, and no chemistry whatsoever. The universe we happen to have is so surprising under the Standard Model because the fundamental particles of which atoms are composed are, in the words of cosmologist Leonard Susskind, “absurdly light.” Compared to the range of possible masses that the particles described by the Standard Model could have, the range that avoids these kinds of complexity-obliterating disasters is extremely small. Imagine a huge chalkboard, with each point on the board representing a possible value for the up and down quark masses. If we wanted to color the parts of the board that support the chemistry that underpins life, and have our handiwork visible to the human eye, the chalkboard would have to be about ten light years (a hundred trillion kilometers) high. And that’s just for the masses of some of the fundamental particles. There are also the fundamental forces that account for the interactions between the particles. The strong nuclear force, for example, is the glue that holds protons and neutrons together in the nuclei of atoms. If, in a hypothetical universe, it is too weak, then nuclei are not stable and the periodic table disappears again. If it is too strong, then the intense heat of the early universe could convert all hydrogen into helium — meaning that there could be no water, and that 99.97 percent of the 24 million carbon compounds we have discovered would be impossible, too. And, as the chart to the right shows, the forces, like the masses, must be in the right balance. If the electromagnetic force, which is responsible for the attraction and repulsion of charged particles, is too strong or too weak compared to the strong nuclear force, anything from stars to chemical compounds would be impossible. Stars are particularly finicky when it comes to fundamental constants. If the masses of the fundamental particles are not extremely small, then stars burn out very quickly. Stars in our universe also have the remarkable ability to produce both carbon and oxygen, two of the most important elements to biology. But, a change of just a few percent in the up and down quarks’ masses, or in the forces that hold atoms together, is enough to upset this ability — stars would make either carbon or oxygen, but not both.
And more. This is not an illusion, it is a significant point. And, Robin Collins aptly used the concept of a bread factory. A super-physics that forces these constants to a just-right range would be like how a factory has to be in a Goldilocks just-right zone to bake good loaves. In short, pushing up the fine tuning one level does not get rid of it. And a bread factory bakes a LOT of loaves, so if we have it randomly tuned, if the just right zone is not broad and easily found on a well behaved fitness slope, then things begin to get tricky: we should not be at this sort of deeply isolated operating point. That is, John Leslie was right, locally isolated operating points are like the lone fly on the patch of wall swatted by a bullet. Matters not that other stretches may be carpeted and any bullet would hit a fly. In this zone, there's the one fly. Crack-splat. Tack-driver rifles and marksmen capable of using that capability don't come along just so. Just ask Olympics champs about their training regimes and rifles. Then, take a look at how the AR-15 family has evolved by design to take up features of such rifles. (Then ask how such a rifle would work at 1,000 m.) KF kairosfocus
Origines, There are simply not enough white dwarfs -- cooling off old stars -- around. In addition, the H-R diagrams for clusters show main sequence branching to the giants bands. Such branching often points to a H-rich ball physics model age of 6 - 8 BY. Notoriously, cosmological expansion points to perhaps 14 BY. As for thermodynamics, its message is simple. Molecules etc behave stochastically; indeed that is tied to what temperature is an index of. As a direct result in an isolated system there is a strong overall trend to degrade concentrations of energy. There is little or no actual empirical observation of anything more than the observed cosmos, all else is speculation. So, the key point is to ignore the lab coats and point out that putting speculation in mathematical terms does not change its character. It is still metaphysics, not physics. On such, we then have the metaphysical right to insist that there are no privileged worldviews, all must sit to the table of comparative difficulties. All must answer to the facts across the board (including those of us sitting here and reasonably arguing while being prompted by conscience). All must answer to coherence. All must answer to balanced explanatory power. With that on the table, evolutionary materialist scientism does not fare well. It cannot ground responsible, rational freedom sufficient to be able to argue like we must and do. KF kairosfocus
SS, Before going further, I suggest you answer substantially to Sir Fred Hoyle:
>>[Sir Fred Hoyle, In a talk at Caltech c 1981 (nb. this longstanding UD post):] From 1953 onward, Willy Fowler and I have always been intrigued by the remarkable relation of the 7.65 MeV energy level in the nucleus of 12 C to the 7.12 MeV level in 16 O. If you wanted to produce carbon and oxygen in roughly equal quantities by stellar nucleosynthesis, these are the two levels you would have to fix, and your fixing would have to be just where these levels are actually found to be. Another put-up job? . . . I am inclined to think so. A common sense interpretation of the facts suggests that a super intellect has "monkeyed" with the physics as well as the chemistry and biology, and there are no blind forces worth speaking about in nature. [F. Hoyle, Annual Review of Astronomy and Astrophysics, 20 (1982): 16.]>> . . . also, in the same talk at Caltech: >>The big problem in biology, as I see it, is to understand the origin of the information carried by the explicit structures of biomolecules. The issue isn't so much the rather crude fact that a protein consists of a chain of amino acids linked together in a certain way, but that the explicit ordering of the amino acids endows the chain with remarkable properties, which other orderings wouldn't give. The case of the enzymes is well known . . . If amino acids were linked at random, there would be a vast number of arrange-ments that would be useless in serving the pur-poses of a living cell. When you consider that a typical enzyme has a chain of perhaps 200 links and that there are 20 possibilities for each link,it's easy to see that the number of useless arrangements is enormous, more than the number of atoms in all the galaxies visible in the largest telescopes. [ --> 20^200 = 1.6 * 10^260] This is for one enzyme, and there are upwards of 2000 of them, mainly serving very different purposes. So how did the situation get to where we find it to be? This is, as I see it, the biological problem - the information problem . . . . I was constantly plagued by the thought that the number of ways in which even a single enzyme could be wrongly constructed was greater than the number of all the atoms in the universe. So try as I would, I couldn't convince myself that even the whole universe would be sufficient to find life by random processes - by what are called the blind forces of nature . . . . By far the simplest way to arrive at the correct sequences of amino acids in the enzymes would be by thought, not by random processes . . . . Now imagine yourself as a superintellect working through possibilities in polymer chemistry. Would you not be astonished that polymers based on the carbon atom turned out in your calculations to have the remarkable properties of the enzymes and other biomolecules? Would you not be bowled over in surprise to find that a living cell was a feasible construct? Would you not say to yourself, in whatever language supercalculating intellects use: Some supercalculating intellect must have designed the properties of the carbon atom, otherwise the chance of my finding such an atom through the blind forces of nature would be utterly minuscule. Of course you would, and if you were a sensible superintellect you would conclude that the carbon atom is a fix. >> . . . and again: >> I do not believe that any physicist who examined the evidence could fail to draw the inference that the laws of nuclear physics have been deliberately designed with regard to the [--> nuclear synthesis] consequences they produce within stars. ["The Universe: Past and Present Reflections." Engineering and Science, November, 1981. pp. 8–12]>>
KF PS: Your discomfort with the widely recognised observations on fine tuning of physics that supports cell based life is inadvertently revealing. BTW, sweet spots are a manifestation of fine tuning. kairosfocus
Querius Thank you for the very insightful comments on the referred interview. You pointed to a few interesting details I had not noticed when I read that article. At this moment I don't understand much of QM, but will heed your advice to look into it deeper. My poor understanding about complex specified information (CSI), which sometimes I additionally qualify as "procedural prescriptive purpose-oriented" CS(PPPO)I, is mostly based on empirical observation and analysis of the process of developing software for engineering design systems. My boss, a brilliant engineer, directed the project I worked on. The functionality of the software existed in his mind before he explained to us how it should look on the screen and work and before he wrote the technical specifications for the programmers to write the code for the compiler to translate our code to a lower level code that ultimately was converted into the lowest level code that the computer operated on to produce the resulting system my boss had so clearly visualized earlier. The morphogen gradient formation and interpretation issues are a relatively 'simple' example for illustrating the same concept of CS(PPPO)I. There are several references to related papers in the thread "Mystery at the heart of life". This has been one of the cases where biologists have stumbled while trying to describe their observations. Actually, professor L.M. himself failed to answer a simple question related to this very issue. Apparently that made him write that he would not discuss further with me because I don't ask honest questions. News wrote that probably what professor L.M. meant was that since his was a losing position he better quit. :) If you want more details on this, I'll gladly provide them to you. Dionisio
Eric @69, WRT to my question "how can it be that in an eternal universe suns have not already died?" I have found this article by W.L.Craig. It's a rather technical piece, but the take-home message seems to be that the second law of thermodynamics is indeed unstoppable. W.L.Craig:
As we have seen, the application of relativity theory to cosmology has altered the shape of the eschatological scenario predicted on the basis of the Second Law, but it has not materially affected the fundamental dilemma. Thus, the same pointed question raised by classical physics persists: why, if the universe has existed forever, is it not now in a cold, dark, dilute, and lifeless state? In contrast to their nineteenth century forbears, contemporary physicists have come to question the implicit assumption that the universe is eternal in the past. P. C. W. Davies reports, Today, few cosmologists doubt that the universe, at least as we know it, did have an origin at a finite moment in the past. The alternative - that the universe has always existed in one form or another—runs into a rather basic paradox. The sun and stars cannot keep burning forever: sooner or later they will run out of fuel and die. The same is true of all irreversible physical processes; the stock of energy available in the universe to drive them is finite, and cannot last for eternity. This is an example of the so-called second law of thermodynamics, which, applied to the entire cosmos, predicts that it is stuck on a one-way slide of degeneration and decay towards a final state of maximum entropy, or disorder. As this final state has not yet been reached, it follows that the universe cannot have existed for an infinite time.11 Davies concludes, "The universe can't have existed forever. We know there must have been an absolute beginning a finite time ago."12
You mention,
Eric Anderson: It seems the idea would be some kind of expanding/collapsing cycle — not a currently held view, to be sure, but theoretically possible.
an idea that is being addressed in the article:
In the 1960s and '70s some cosmologists tried to escape this conclusion by adopting an oscillating model of the universe which never began to exist nor ever reaches a final state of equilibrium .... ... Not only was such a theory extraordinarily speculative, but the prospects of such a model were severely dimmed in 1970 by Roger Penrose and Stephen Hawking's formulation of the Singularity Theorems which bear their names.14 The theorems disclosed that under very generalized conditions an initial cosmological singularity (or beginning point) is inevitable, even for inhomogeneous and non-isotropic universes. ... But wholly apart from these difficulties, the thermodynamic properties of this model turned out to imply the very problem that its proponents sought to avoid. For entropy is conserved from cycle to cycle in such a model, which has the effect of generating larger and longer oscillations with each successive cycle .... As one scientific team explains, "The effect of entropy production will be to enlarge the cosmic scale, from cycle to cycle. . . . Thus, looking back in time, each cycle generated less entropy, had a smaller cycle time, and had a smaller cycle expansion factor then [sic] the cycle that followed it." ... In fact, astronomer Joseph Silk estimates on the basis of current entropy levels that the universe cannot have gone through more than 100 previous oscillations. ...
Other speculative models have been proposed, such as the 'Chaotic Inflationary Model'. Craig concludes:
Since such speculative conjectures fail to elude the problem, we seem left with the conclusion that the universe is not past eternal. The Big Bang represents the absolute beginning of the universe, just as it does in the Standard Big Bang model; and the low entropy condition was simply an initial condition. Indeed, thermodynamics may provide good reasons for affirming the reality of the singular origin of space-time postulated by the Standard Model.
Origenes
Eric, Mung noted that
If things with[in] the universe can have no cause then science is a myth.
I don't think Mung's statement was too broad. Once to eliminate the need for causality in nature and in science, you cease to be able to make reliable inferences. "Yes, the apple dropped from the tree, but was it due to gravity or an anomaly?" You could never be sure. -Q Querius
Dionisio, First of all, thank you for posting the link to the interview of Dr. Vedral by Aleks Krotoski, who asked brilliant, insightful questions. Dr. Vedral's self deprecation was not false humility, but his recognition of the dichotomy between QM and various explanations of its significance. QM has been demonstrated as precisely as 10 parts per billion. It is considered to be the most thoroughly challenged and experimentally validated of all the sciences. However, the explanations of its significance is wildly controversial. Dr. Vedral did explain that information is expressed in terms of material existence, but he did not go on to describe that the collapse of probabilistic wavefunctions through observation and measurement is the cause for this material expression. I noticed that Dr. Vedral stumbled over the false notion that God must be subject to time before time even came into existence, which as a result, led Dr. Vedral to think that it would lead to an infinite regression of gods creating gods. Since we live temporally it's very difficult for us to imagine existence without time (or space). A related question might be, "If God is infinite, how can He fit into the universe?" The answer is the same. Dr. Vedral rejected the notion of a mechanistic, predictable universe (that has no necessity or room for God), and he noted that the origin of the laws of physics is currently a mystery. Also, I didn't think Dr. Vedral adequately addressed Ms. Krotoski's question about how our physical, social, and economic existence traces itself back to information. Both Dr. Vedral and Ms. Krotoski seemed very nervous about bringing up the subject of God. :-) If you're not already familiar with quantum entanglement, quantum erasure, and the zeno effect, you might want to look into them, realizing that everything in our macro existence ultimately traces back to quantum interactions. I freely admit that I don't understand the theory of information and its classification. Dr. Vedral pointed out that the information is much simpler than we make out. It also intrigued me that he touched on the binary nature of information, which, by the way, is a primary theme in the Genesis account of creation. For example, the separation of light from darkness generated information. Yes, I do think Dr. Vedral was struggling with the metaphysical implications of QM. As you heard, he admitted that he was a simpleton with regards to the significance of QM. -Q Querius
Mung @45:
Anything that is caused requires a cause. It follows that there must be a first cause. Whether the universe had a beginning or not is irrelevant. [For determining whether there must be a first cause.] If it has a cause for its existence, there must be a first cause. Perhaps the universe is eternal. So what. It still either requires a cause of its existence or it does not.
Agreed. Up to this point.
If the universe can be uncaused, then by what principle must everything in the universe have a cause?
No principle. The need for causation is based by our observation and analysis. If we observe (or infer on good evidence) that something needed a cause, then so be it. We conclude that thing required a cause.
If things with[in] the universe can have no cause then science is a myth.
That is too broad a statement, referencing, as it does, everything in the universe. Furthermore, science -- understood as an objective, evidence-driven, search for truth -- could certainly help lead us to conclude that something is eternal, if that is in fact the case. Whether something has a cause or not doesn't prevent science from observing that fact and pointing us in the right direction. Now if what you mean is slightly more modest, namely that we generally carry out our scientific activity on the assumption that everything we see in the physical world around us has a cause, then, yes, I agree. The assumption that things have a cause is a highly useful and time-worn principle of doing science in the real world. Not necessarily a purely material cause, mind you -- the whole principle of intelligent design being an argument against that materialistic version of "science." But some kind of cause. Eric Anderson
Origenes @40: Great question. It seems the idea would be some kind of expanding/collapsing cycle -- not a currently held view, to be sure, but theoretically possible. Your point is well taken, at least regarding the universe as we experience it. That experience might not translate well to whatever the cause of the universe is, but it is still an observation that should give us some pause. Eric Anderson
Querius @63 #67 addendum You quoted Oxford professor Vlatko Vedral (originally from Belgrade, Serbia). Professor Vedral's idea of information seems kind of materialistically defined in terms of collection of atoms or something like that? At least that was my perception from his interview by British "The Guardian" over 6 years ago: https://www.theguardian.com/science/2010/mar/07/vlatko-vedral-interview-aleks-krotoski Did I get it right? Did I miss or misinterpreted something in that article? Are software programs just a bunch of tiny electronic impulses wildly dancing around? There's an abysmal difference between information and complex specified information which contains procedural prescriptive purpose-driven meaning. Oxford math professor John Lennox said that nonsense remains nonsense even if said by famous scientists. Dionisio
Querius @63 It seems like Vlatko Vedral's ideas on information are inaccurate. Yes, I agree that before matter, energy, time, space there was information. But he seems to have a reductionist oversimplified concept of what we would call complex specified information. Dionisio
Sean,
History alone would give a reasonable person more confidence in the “natural” proposal than the “God proposal”. Those are reasons why “natural” explanations are preferable. They don’t provide much comfort except for the satisfaction that comes from pursuing the truth wherever it leads.
Which is why you'll have no trouble following the data from quantum mechanics to the demonstrated conclusion that "natural" reality as it is commonly thought of doesn't actually come into existence apart from observation and measurement. Right? -Q Querius
Since I am named in the OP, I felt an extended reply was warranted. I have not yet even tried to look at the 60+ comments. Fine Tuning” is a conclusory term and should not be used. I realize that it is widely used, but the term “fine tuning” is conclusory; because it contains a verb form, it naturally implies a conclusion has been reached; that a particular design decision was made to “finely tune” our universe. This should only be referred to as “apparent fine tuning”; the appearance of “tuning” does not prove any fact of “tuning”. But of course, the “apparent” part seems to be dropped. I prefer to state this question as: “how did our universe end up in a sweet spot for life?” The meaning of sweet spot being “an optimum point or combination of qualities or factors”. This precisely describes the problem. The phrase “sweet spot” does NOT imply the lack of a deity: we might be in the sweet spot because that’s where the deity put us. Or it could have been a chance event; we just landed there. Sweet spot is a neutral term. It is a very strong inference from physics that even the most minor variation in some of the universal constants and physical attributes of our inverse would preclude life in our universe. That these constants and attributes happen, in combination, to enable life is an example of a sweet spot. What is completely missing is any evidence that this sweet spot was created FOR us, or that we were CHOSEN to be in it. That we are in the sweet spot is a given, why we are in the sweet spot is a complete mystery. The phrase “sweet spot” also reveals an idea that “fine tuning” conceals: there may well be many sweet spots. We landed in a sweet spot for our kind of life, there may be other sweet spots marginally better or worse that we might have landed in. There may be sweet spots for other, completely different forms of life. We just don’t know. I am sure the writer of the OP would prefer to use the phrase “fine tuning”; it is just too useful in its’ implication to ignore. For example, the OP said
4. An inference, from observable facts, that the universe has been finely tuned.
Actually, not so. Using the label “fine tuning” strongly implies that these values were selected; implying the desired result; and assuming that our universe was created by a unique process (whatever that may have been). But there’s zero evidence of that, what the evidence shows is that our universe is in a sweet spot; there’s no evidence showing how we got there. Fine Tuning” is unfounded.Fine tuning” assumes three things: 1. that our conclusion is correct: if our universe was not in this sweet spot, then our universe would be sterile; 2. that whatever caused our universe had the freedom to create a universe with literally any random value or combination of these constants and attributes; and 3. that whatever caused our universe acted only once or at most only a small number of times. None of those assumptions is a given. The first one seems necessary and “natural” but is provisional. Maybe our understanding of physics will be significantly altered and we’ll come to doubt this conclusion. It could be wrong in any event and many kinds of life-supporting universes are possible. There may be many sweet spots; a result we might not discover for centuries. The second one also seems necessary and “natural”, but it is equally uncertain. The process that resulted in our universe is completely unknown; we cannot say with any certainty what constraints apply to it. The third one is neither necessary nor “natural”. It implicitly asserts uniqueness to our universe as the product of a unique, one-off event. This is not a reasonable thing to assume. Whatever created our universe may have done so many times; maybe a vast number of times. “Natural” processes are rarely characterized by such one-off events. An equally valid interpretation of the problem is that whatever process created our universe is a recurrent process (as “natural” processes tend to be) and that our universe is only one of many universes created by it. The basis of the “fine tuning” argument is the claim that our universe’s specific attributes are improbable; but implicit in that claim is the untenable assumption that we know enough about how our universe was created to determine the “probable” outcomes. We simply do not have that information. If the creating event happens often, then even if the results of these frequent events are actually random in nature, the likelihood of our particular universe resulting by chance alone becomes more probable. The more universes created, the more likely one or more will end up in sweet spots for some form of life. Likewise, we don’t know if sweet spots are rare. The processes that create universes and determine their attributes could operate under constraints that force most universes into some sweet spot or other. Again, we don’t know. The OP appears to endorse “hyperskepticism”.
We can, indeed we must, approach claims of a multiverse or cosmic bubbles or some other universe-generating natural phenomenon with extreme skepticism.
This appears to be a call to “hyperskepticism” which is regularly (and rightly) criticized on this site. Multiverses; cosmic bubbles, or other universe-generating natural phenomenon need to be treated with due skepticism, but not extreme skepticism. Likewise with deistic explanations. The OP is flawed by omissions of logical conclusions that run contrary to the writer’s apparent preferences. For example:
... what can be said is that: (a) no-one has any real observational evidence as to the cause of the universe; and (b) it is possible that the cause of the universe was the First Cause. In addition, we might add that (c) it is possible that the First Cause had a plan, a purpose, an intent, a desire, a design – attributes similar to what we see ourselves possessing as rational, intelligent, individual, creative beings.
This is incomplete. To be correct, it must include: (d) it is also possible that the First Cause had no attributes similar to what we see ourselves possessing as rational, intelligent, individual, creative beings; it may have been a truly mindless thing. Later on you wrote that,
The inference that the universe had a beginning does not allow us to identify the First Cause. We cannot say, it seems to this author, as a matter of logic and deduction that the First Cause is God. We cannot even say that the universe was caused by the First Cause, rather than some intermediate cause. Indeed, as a matter of dispassionate objective scientific inquiry and reasoning, we can say but very little about the First Cause. In that sense, the claim that the First Cause is God must be viewed with some caution. But it must not be viewed with derision. Rather, it should be seriously viewed as a live possibility, very much worthy of consideration.
That first paragraph is worthy of it’s own response (below). But on the whole what you wrote is true enough, but incomplete. To be complete, you must also add that the claim of a “natural” First Cause must be viewed with no more caution than your “God proposal” (and vice-versa). Neither should be viewed with derision. The “God proposal” is not on an equal footing with the “natural” proposal. Under the heading “The Nature of the First Cause” you first describe the non-deistic explanation. I have little to object to in that except that this explanation never arrives at a “finely-tuned universe”; it merely resulted in the universe we have being in a sweet spot for life. Then you describe the “God proposal” and quickly note that “materialist” objections “only serves to put the God proposal on at least the same footing” as the “naturalistic” proposal. This is not correct. The “naturalistic” proposal proposes phenomena that are at least hypothetically testable; the “God proposal” does not. It may be that all efforts to test the “naturalistic” proposal fail or come up indecisive; at which point the two proposals might be on the same footing. But until that day, the two proposals are decidedly not on the same footing. Although your comments on the many persons who have claimed personal encounters with their deities “(often at great risk to their reputation and physical safety)” is superficially compelling, it remains a fact that the vast majority of risk to these claimants is from other people claiming different beliefs about the vast number of deities that have been reported. These claims do amount to “evidence” of deities, but the evidence also points to a vast number of deities apparently at war with each other and with each other’s followers. The simple chaos of competing and conflicting religious claims robs all claims of any credibility. The chaos of religious claims and conflict bears greater resemblance to a mass-delusion than to any believable conclusion. I know that this conclusion makes many believers angry, but they have no call to be angry at me; I’m just the messenger. If there is a deity, this mass-confusion is their work, not mine. At the end of the day, no claim about any of the upwards-of-four-thousand-possible-deities is believable without first having a favorable prejudice (i.e.: Faith) or a direct, face-to-face encounter with a deity.
The concept of God at least has the benefit of positing a First Cause with the ability to make the purpose real, to fine tune for a purpose, to have a plan and a goal and an intended outcome;
In other words, the concept of God has the benefit if explaining things that may actually be illusions. As benefits go, that’s one’s illusory.
Indeed, when compared against the materialistic claim, the proposal that the First Cause is God is eminently reasonable – being more consonant with the evidence, with our experience, and with the reasonable inferences that can be drawn from scientific inquiry.
The “God proposal” is only more consonant with some evidence, experiences, and inferences, and only when those things are viewed with the favorable prejudice that is Religious Faith. Seen neutrally, the “God proposal” at best is no better and is utterly unverifiable. When viewed against the totality of evidence, experiences, and reason, the “God proposal” does not look reasonable. It could be true, but that truth is irretrievably buried.
While recognizing a significant lack of direct observational evidence on either side of the debate, the objective observer must at least consider the existence of God as a live possibility and, when weighed against the alternative, as the more rational and supportable possibility.
This is internally contradictory. If there were enough evidence to make one proposal or the other “the more rational and supportable” then references to a “significant lack of direct observational evidence” would be mischaracterization. The only real difference between the two proposals is that the natural proposal is simpler (gods are the ultimate complication), and the natural proposal more rational and supportable. The natural proposal is the more rational and supportable because it’s the only one that could ever be falsified. This is not a minor point. You appear to treat these comparisons as static; but as more evidence is produced, one proposal or the other is likely to prevail; and only the “natural” proposal can be tested. Trust but verify. Conclusions
So what are we left with? The inference that the universe had a beginning does not allow us to identify the First Cause. We cannot say, it seems to this author, as a matter of logic and deduction that the First Cause is God. We cannot even say that the universe was caused by the First Cause, rather than some intermediate cause. Indeed, as a matter of dispassionate objective scientific inquiry and reasoning, we can say but very little about the First Cause.
I actually gasped when I read this. Wow. That pretty much moots the entire OP in which you try valiantly to make God your first cause (without seeming to). Kudos for your subtlety. But you stumbled here. When the totality of the evidence and human experience is considered (including our experiences with mass delusions, self-serving claims, and the embracing of comforting illusions) we cannot conclude that the “God proposal” is wrong but we can reasonably conclude it deserves a good deal of skepticism. The extraordinary claims of creation in some kind multiverse/bubble-cosmos deserve a good deal of skepticism too. It might not even be acceptable except for the realization that such “natural” explanations are inherently simpler, and they posit effects that should be observable and verifiable. As for the current lack of evidence for the “natural” proposal, that objection needs to be put in context: the “God proposal” has been around for millennia and remains, at best on the same footing” as the “natural” proposal the “natural” proposal has been around only for a few years. History alone would give a reasonable person more confidence in the “natural” proposal than the “God proposal”. Those are reasons why “natural” explanations are preferable. They don’t provide much comfort except for the satisfaction that comes from pursuing the truth wherever it leads. sean s. sean samis
Eric:
2. Yes, any cause needs to be prior to and outside of whatever it caused.
If you mean "prior to" in a temporal sense then this is simply mistaken. That a cause must be outside of that which it causes is very likely also not correct. At least I can't appeal to any irony in the title of the OP about other people not understand the First Cause argument. :) Mung
A multitude of experiments in quantum mechanics have demonstrated that the fundamental nature of existence is mathematical information in the form of probability distributions. Each probability distribution is termed a wavefunction, psi. “Solid matter” or electromagnetic waves do not exist physically apart from the collapse of wavefunctions, which occurs when they are observed or measured. Thus, consciousness seems to be more fundamental than matter. QM is involved in all light, chemical bonds, electron movement, and atomic behavior. Niels Bohr, who was awarded the 1922 Nobel Prize in physics for his application of quantum theory to atomic and molecular structure, expressed it this way:
Everything we call real is made up of things that cannot be regarded as real. If quantum mechanics hasn’t profoundly shocked you, you haven’t understood it yet.
This concept is now widely accepted by physicists. For example, Vlatko Vedral is a Professor of Physics at the University of Oxford, and CQT (Centre for Quantum Technologies) at the National University of Singapore, and a Fellow of Wolfson College. As a recognized leader in the field of quantum mechanics, here’s how he expresses it:
The most fundamental definition of reality is not matter or energy, but information–--and it is the processing of information that lies at the root of all physical, biological, economic, and social phenomena.
The key here is the realization that when we simply observe light, electrons, even small molecules or viruses in the double-slit experiment, it determines whether you get a particulate or a wave pattern. This scientifically falsifies • Materialism – All that exists is matter and energy and the rearrangements of it. (extreme realism) • Realism – A physical reality exists independent of observation. • Naïve Realism — Reality exists independent of observation, just that our perceptions are just a representation of something actually there. (Falsified by QM experiments in 2011, 2012) And it leaves us with only two other options: • Idealism – Reality is a mental construct, and doesn't exist independent of observation. • Solipsism – The extreme skeptical version of idealism, which claims that only your mind exists and anything outside of it is an illusion. Take your pick! :o
What we perceive as reality now depends on our earlier decision what to measure, which is a very, very deep message about the nature of reality and our part in the whole universe. We are not just passive observers. - Anton Zeilinger
-Q Querius
Hi Eric,
However, I am not convinced that the eternal existence of physical reality is logically impossible.
Which of the following are you saying you're not convinced about? A) that the past-infinite temporal existence of physical reality is logically impossible? B) that the eternal non-temporal existence of physical reality is logically impossible? Take care, HeKS HeKS
HeKS @34:
That said, where the fact that materialistic scientists and philosophers thought the physical universal existed as a brute eternal fact becomes relevant is in pointing out that it is hypocritical of them to have been willing to accept the infinite temporal existence of the physical universe (which is logically impossible) without demanding any external cause for it but then demanding an external cause for God, being unwilling to accept his eternal non-temporal existence (which is not logically impossible) as a brute fact.
I agree with your assessment of the hypocritical nature of the position, and materialists should be challenged on that point. This is why I said that, at most, the materialist complaints about God simply serve to place God on the same footing as the materialist position. And that is even ignoring the other inferential evidence that might weigh in favor of the God hypothesis. However, I am not convinced that the eternal existence of physical reality is logically impossible. So far, it seems to me that any attempt at a logical deduction in this regard runs up against its own set of problems by attempting to define the nature of the existence of the First Cause as the absence of that which was caused. Eric Anderson
Eric, I feel privileged to engage in such a high level analysis. We have everything to gain and nothing to lose by putting it all out there. You write,
If the universe were eternal, then its existence could simply be, as with the First Cause. One could even argue that it is the First Cause. The reason why a causal explanation is needed for the universe is because it had a beginning.
Yes, I agree in context. (***To be sure, an eternal universe cannot be the ultimate first cause because as Aristotle pointed out, it requires a “prime mover” as a logically prior [not chronologically prior] cause. Put another way, even an eternal universe requires an active first mover. Nothing can move unless moved or designed to move by an outside agent). Still, I get your broader point. We cannot simply depend on science to reassure us that time/space/matter probably came into existence at the big bang. If we are going to argue in terms of what it is or is not logically possible, we need something like this: infinity cannot be instantiated in reality (or matter). We can have potential infinity, but not actual infinity. Under the circumstances, we cannot have an infinite number of physical events. Thus, the universe cannot be eternal. .
I like where you are going, but how would you reconcile your last comment about ideas with the “Word” concept Querius and BA77 referred to above? Does information count as an “idea”?
I would say that “information” is more than an idea, number, or mere abstraction. Unlike the former, the latter elements cannot bring something about. (We know that an abstract number cannot be the product of time, space, matter, but that is another subject altogether). With respect to “Word,” I would need to have it defined in context. A string of letters cannot produce an effect, but the eternal Logos or the "Word made flesh" certainly can. Meanwhile, I would hold to my main point (with which you seem to agree): the beginning of the universe can be explained only by an immaterial first cause. It seems to me that the only thing we need to close the loop, logically, is to show that the universe cannot be eternal. Thus, we note the importance of linking the finite number of events that have occurred with the temporal duration of the physical cosmos. (i. e. infinitely cannot be instantiated in physical reality). [The scientific argument speaks for itself: The universe probably began to exist], followed by the philosophical reflection [if it did, it was the product of an immaterial cause]. Clearly, time, space, and matter cannot produce time, space, and matter. StephenB
Erik Anderson @30:
Logically, some form of existence must be in play for the cause of the universe. What is the nature of that existence? We might be able to say that the cause existed outside of our current observable parameters of space and time and matter that exist in our universe. That could be accurate.
harry @32:
Whatever it is, the nature of that existence isn’t natural according to our understanding of the natural. It obviously transcends the natural. Therefore, it is entirely correct to say that it is supernatural. To then say that that supernatural reality might not be the uncaused first cause, sounds a lot to me like one takes seriously Dawkins’ silly question, “Then who created God?”
Erik Anderson @57:
What is obvious is that it is outside of and prior to our universe. Whether it is “natural” or not, is a separate question and we must not conflate the two.
That which is outside of space and time, and does not consist of matter and energy -- that which transcends the natural and brought it into being -- is supernatural. It is beyond the ability of science to observe (science's observations are restricted to the material universe), so there is no way for us to find out if it is a "natural" reality that is not God, or if it was caused by another "natural" reality that is still not God. Whatever its true nature, it is still supernatural from our perspective. It is pure, pointless, unverifiable speculation to suggest that there may be layers and layers of "natural" realities, each of which was preceded and caused by a supernatural reality from its perspective, before we arrive at the uncreated Supreme Being Whose nature is to be. What are you after, Erik? Where are you going with this speculation? harry
StephenB @33:
Eric, I like this discussion and it is evident that you don’t mind tugging away at these ideas.
It is definitely an interesting topic and I appreciate people being willing to examine it. I realize it goes to the heart of some strongly-held beliefs, so a willingness to examine it thoroughly and carefully is a good sign of objectivity. Just a few thoughts regarding the points you made:
Logically, this is possible, but this hypothetic static universe also consists of time/space/ matter and needs explaining for all the same reasons.
It is not just that it is static during a particular point in time that is the issue, but the eternal nature of it. I apologize if I didn't make that clear enough. If the universe were eternal, then its existence could simply be, as with the First Cause. One could even argue that it is the First Cause. The reason why a causal explanation is needed for the universe is because it had a beginning.
If it is part of a larger multiverse, (or a product of the multiverse) then the multiverse network becomes the ultimate universe and we are back to where we started. It hardly matters whether we attach the prefix “uni “or “multi” to the ultimate cosmic time/space/continuum. The question remains the same: Does the ultimate time/space/continuum of impersonal forces (that once didn’t exist and now does exist) need a first cause and what must that first cause be like.
Agreed.
If the cosmic time/space/matter continuum once didn’t exist and now does exist, there are only two possible candidates for a first cause: It must be either be an immaterial cause or a material cause. (We can rule out abstract principles because ideas and numbers cannot act as causes).
I like where you are going, but how would you reconcile your last comment about ideas with the "Word" concept Querius and BA77 referred to above? Does information count as an "idea"?
It would seem that only a timeless, spaceless, immaterial cause (being) can bring time, space, and matter into existence.
Two things: 1. Again, if time, space, and matter came into existence at some point, then, yes, they need to be explained and cannot be the explanation for themselves. If they always existed, then no such explanation is required. (BTW, I hope I have made it clear that I am not making this claim; I am simply using it to help us analyze the cause of the universe, as contrasted with the First Cause.) 2. Yes, any cause needs to be prior to and outside of whatever it caused. But (a) the First Cause need not be the cause of our universe (as you rightly pointed out, the First Cause could be the cause of the cosmic bubble, or the multiverse, or some other intermediate cause); and (b) we need to be careful about defining the First Cause as the negation of the parameters we see in our universe. Eric Anderson
harry @32:
Whatever it is, the nature of that existence isn’t natural according to our understanding of the natural. It obviously transcends the natural.
What is obvious is that it is outside of and prior to our universe. Whether it is "natural" or not, is a separate question and we must not conflate the two. Eric Anderson
BA77 @21: Thanks for the good comments and quotes about the nature of mind and information. Reminds me also of the "Word" concept Dembski (and you and Querius above) and others have hinted at many times. Certainly something worth considering in this area, and one more reason why we could infer that the First Cause might not be a purely naturalistic phenomenon. Eric Anderson
Eric Anderson as to this comment you made at 49:
After all, materialists can just claim that meaning, consciousness, morality — when they are willing to acknowledge the existence of such things — somehow inexplicably arose as a byproduct of evolution or as some kind of emergent property.
IMHO Eric, you are giving the atheistic/naturalistic worldview far too much credit as to being even remotely possible and/or reasonable. The atheist's worldview, in regards to explaining the origin of anything within the universe and even explaining the origin of the universe itself, is simply insane. For example, in what I consider to be a shining example of poetic justice, in their claim that God is not really real but is merely an illusion, the naturalist also ends up claiming that he himself is not really real but is merely an illusion. Francis Crick stated,,,
“that “You”, your joys and your sorrows, your memories and your ambitions, your sense of personal identity and free will, are in fact no more than the behaviour of a vast assembly of nerve cells and their associated molecules. As Lewis Carroll’s Alice might have phrased: “You’re nothing but a pack of neurons.” This hypothesis is so alien to the ideas of most people today that it can truly be called astonishing.” Francis Crick – “The Astonishing Hypothesis” 1994 “We have so much confidence in our materialist assumptions (which are assumptions, not facts) that something like free will is denied in principle. Maybe it doesn’t exist, but I don’t really know that. Either way, it doesn’t matter because if free will and consciousness are just an illusion, they are the most seamless illusions ever created. Film maker James Cameron wishes he had special effects that good.” Matthew D. Lieberman – neuroscientist – materialist – UCLA professor The Confidence of Jerry Coyne – Ross Douthat – January 6, 2014 Excerpt: But then halfway through this peroration, we have as an aside the confession (by Coyne) that yes, okay, it’s quite possible given materialist premises that “our sense of self is a neuronal illusion.” At which point the entire edifice suddenly looks terribly wobbly — because who, exactly, is doing all of this forging and shaping and purpose-creating if Jerry Coyne, as I understand him (and I assume he understands himself) quite possibly does not actually exist at all? The theme of his argument is the crucial importance of human agency under eliminative materialism, but if under materialist premises the actual agent is quite possibly a fiction, then who exactly is this I who “reads” and “learns” and “teaches,” and why in the universe’s name should my illusory self believe Coyne’s bold proclamation that his illusory self’s purposes are somehow “real” and worthy of devotion and pursuit? (Let alone that they’re morally significant: But more on that below.) Prometheus cannot be at once unbound and unreal; the human will cannot be simultaneously triumphant and imaginary. Per NY Times Atheistic Materialism – Does Richard Dawkins Exist? – video 37:51 minute mark Quote: “You can spout a philosophy that says scientific materialism, but there aren’t any scientific materialists to pronounce it.,,, That’s why I think they find it kind of embarrassing to talk that way. Nobody wants to stand up there and say, “You know, I’m not really here”. https://www.youtube.com/watch?v=rVCnzq2yTCg&t=37m51s
Moreover, this illusory nature inherent to naturalism gets worse for the atheist. Much worse! In the atheistic worldview, not only do people become illusions, but the perceptions that these supposedly 'illusory people' are having of reality become illusory to! In the following video and article, Donald Hoffman has, through numerous computer simulations of population genetics, proved that if Darwinian evolution were actually true then all of out perceptions of reality would be illusory.
Donald Hoffman: Do we see reality as it is? – Video – 9:59 minute mark Quote: “,,,evolution is a mathematically precise theory. We can use the equations of evolution to check this out. We can have various organisms in artificial worlds compete and see which survive and which thrive, which sensory systems or more fit. A key notion in those equations is fitness.,,, fitness does depend on reality as it is, yes.,,, Fitness is not the same thing as reality as it is, and it is fitness, and not reality as it is, that figures centrally in the equations of evolution. So, in my lab, we have run hundreds of thousands of evolutionary game simulations with lots of different randomly chosen worlds and organisms that compete for resources in those worlds. Some of the organisms see all of the reality. Others see just part of the reality. And some see none of the reality. Only fitness. Who wins? Well I hate to break it to you but perception of reality goes extinct. In almost every simulation, organisms that see none of reality, but are just tuned to fitness, drive to extinction that perceive reality as it is. So the bottom line is, evolution does not favor veridical, or accurate perceptions. Those (accurate) perceptions of reality go extinct. Now this is a bit stunning. How can it be that not seeing the world accurately gives us a survival advantage?” https://youtu.be/oYp5XuGYqqY?t=601 The Evolutionary Argument Against Reality - April 2016 The cognitive scientist Donald Hoffman uses evolutionary game theory to show that our perceptions of an independent reality must be illusions. Excerpt: “The classic argument is that those of our ancestors who saw more accurately had a competitive advantage over those who saw less accurately and thus were more likely to pass on their genes that coded for those more accurate perceptions, so after thousands of generations we can be quite confident that we’re the offspring of those who saw accurately, and so we see accurately. That sounds very plausible. But I think it is utterly false. It misunderstands the fundamental fact about evolution, which is that it’s about fitness functions — mathematical functions that describe how well a given strategy achieves the goals of survival and reproduction. The mathematical physicist Chetan Prakash proved a theorem that I devised that says: According to evolution by natural selection, an organism that sees reality as it is will never be more fit than an organism of equal complexity that sees none of reality but is just tuned to fitness. Never.” https://www.quantamagazine.org/20160421-the-evolutionary-argument-against-reality/
Thus, given atheistic premises, people become illusions whose observations of reality are illusory. And exactly why in blue blazes should anyone, much less an ID proponent, trust anything that illusions having illusions say about reality or science? Especially since reliable ‘observation’ of reality is a necessary cornerstone of the scientific method itself,,,
Steps of the Scientific Method Observation/Research Hypothesis Prediction Experimentation Conclusion http://www.sciencemadesimple.com/scientific_method.html
Moreover, completely contrary to materialistic premises, conscious observation, far from being illusory, is experimentally found to be far more integral to reality, i.e. far more reliable, than materialism had ever predicted. In the following experiment, it was found that reality doesn’t exist without an observer.
New Mind-blowing Experiment Confirms That Reality Doesn’t Exist If You Are Not Looking at It – June 3, 2015 Excerpt: The results of the Australian scientists’ experiment, which were published in the journal Nature Physics, show that this choice is determined by the way the object is measured, which is in accordance with what quantum theory predicts. “It proves that measurement is everything. At the quantum level, reality does not exist if you are not looking at it,” said lead researcher Dr. Andrew Truscott in a press release.,,, “The atoms did not travel from A to B. It was only when they were measured at the end of the journey that their wave-like or particle-like behavior was brought into existence,” he said. Thus, this experiment adds to the validity of the quantum theory and provides new evidence to the idea that reality doesn’t exist without an observer. http://themindunleashed.org/2015/06/new-mind-blowing-experiment-confirms-that-reality-doesnt-exist-if-you-are-not-looking-at-it.html
Apparently science itself could care less that atheists are forced to believe, because of population genetics, that their observations or reality are illusory! Finally, this unconstrained ‘illusory’ nature inherent to naturalism/materialism becomes even more acute when atheists try to explain the origin and sustaining of the universe, i.e. try to explain the origin, fine-tuning, and quantum wave collapse of the universe. That is to say, every time an atheist postulates a random infinity to try to get around the glaringly obvious Theistic implications of the Big Bang, fine-tuning, and the quantum wave collapse, of the universe, then the math surrounding that random infinity tells us that everything that is remotely possible has a 100% chance of existing somewhere in that random infinity of possibilities that the atheist had postulated. Even an infinite number of Richard Dawkins riding on an infinite number of pink unicorns becomes assured. Since that absurdity is epistemologically self-defeating, then the atheistic worldview is falsified as a coherent theory of knowledge. Scientific knowledge or otherwise.
Atheistic Materialism - Where All of Reality Becomes an Illusion - video https://www.facebook.com/philip.cunningham.73/videos/1213432255336372/
bornagain77
RH7, just to post a comment, you were forced to implicitly rely on distinct identity. This instantly partitions the world in a broad relevant sense: W = {A|~A}, that is: once there is a distinct identity of being or possible being for any entity A, the world is implicitly partitioned. Instantly, this dichotomy brings with it the triune first principles of right reason: LOI, LNC, LEM. These are not optional, as to think, communicate, reason or use rhetoric to persuade you are forced to rely on distinct identity. Nor are they provable, every proof must rely on distinct identity. They must be humbly faced as self-evident, inescapable, necessary truth, the premise on which we may think. And, such do not cut off at the borders of our observed world, they are part of the inherent framework of any possible world; world in the wide sense of domain of actual or potential reality. Try to think or speak of a possible world without relying on distinct identity. Impossible. Such are frameworking principles of reality, reflected in our world of thought. And if you are now considering the Kantian ugly gulch between the inner life of thoughts and the outer one of things in themselves, ponder a point by F H Bradley. He who would consider that he cannot know anything about external reality, implies a claim to know that much about external reality. His thought becomes self-referentially incoherent. Self evident, generally applicable first principles of reason are of utterly general utility. Again, we see how our age has led us into a vast morass of confused and self-referentially incoherent schemes and snares of thought. Let us follow sound guides and escape before we sink into the quicksands of self-falsification. KF kairosfocus
EA, the legal corpus of thousands of years of experiences, rules of thumb and principles goes over a very broad range of matters and subjects:
-- eye-witnesses, -- circumstantial facts and objects, -- scientifically grounded natural law and reliable processes, -- arithmetic, statistics, higher math underneath, -- logic, induction vs deduction, ex falso quodlibet, -- the issue of logical and dynamical coherence, -- questions of factual adequacy -- quality of record, chain of custody, repository and credibility of documents -- issues of inferring to moral certainty across competing explanations -- characteristics of truthful vs false witnesses and advocates -- and more.
There are hard-bought principles, rules of thumb and lessons of history, often paid for with pain, tears and blood. Not to mention, lessons on the results of eras of marches of folly. Such as our own is all too rapidly becoming. Therefore, it is quite reasonable to see such as a valuable repository in its own right, and to draw upon this corpus. Such is my context above. KF kairosfocus
Eric Anderson @46, Thank you for clarifying.
EA #30: We might be tempted to go one step further and argue that the cause had to exist outside of space and time and matter as well. Yet – and here we must pay very close attention because it is critical to understand the chain of thinking – such a claim rests upon and is conditioned upon the proposition that these very parameters (space, time and matter) exist only in our universe. But that is only because we have defined them as such.
My argument (#15) attempts to show that there is nothing external to the First Cause. The following is incoherent: “At the very root of reality there is the First Cause, which is encapsulated by …. “ IOWs the First Cause — not “the cause” — exists ‘outside’ of anything else. Here “anything” obviously includes time, space, matter and laws. I could have written more clearly. Essentially, I’m arguing that the First Cause, because it is fundamental, cannot consist of parts and cannot be encapsulated by something external. Such states of affairs would be incoherent. Origenes
Eric, There are a lot of places to go astray here.
Indeed, already in positing a First Cause we are positing that something existed prior to the universe. And that something existed, by definition, outside of the universe. I presume you agree up to this point.
Not exactly. Since space-time had its beginning with the big bang, using the word "before" rather than extra- or super- makes no sense. It's hard to imagine existence without time, but maybe you can start by thinking in terms of an instantaneous finite state machine. Or think of everything that's happened between two times, T0 and T1, spread out as a static space-time solid. Nothing is changing anymore between T0 and T1. It's all captured, including time, like a 4D photo on your extra-universal desk.
Next, we could go on to claim that this something existed outside of “space and time,” because space and time only exist in the context of the universe. But that is essentially a circular argument.
Why would you say it's circular? If space-time had a beginning, it had a primary cause. That primary cause could not possibly have existed within space-time, otherwise a non-existent space-time managed to cause itself to exist. Another point to consider is that many quantum physicists believe that there's a better than even chance (~60%) that our existence is within a simulation. A simulation has a cause, a purpose, and an evaluation. Or maybe it's all more like a dream, which has a beginning with an external first cause, internal virtual space-time dimensions, information, activity, and an external termination. All this is easily imaginable and possible with virtually none of the complexity of our current understanding of physics, chemistry, and biology. It's far more likely that existence outside our space-time is far more complex. Try imagining two or more time dimensions "orthogonal" to each other . . . -Q Querius
Eric Anderson @48 Can the Christian terms "general revelation" and "special revelation" be associated somehow with the questions @17? Dionisio
john_a_designer @19: Thanks for your comments. At the end:
Of course, the naturalist could still argue that maybe there is no ultimate explanation. Bertrand Russell conceded as much in his 1948 BBC radio debate with Fr. Fredrick Copleston about the existence of God. However, even if that could be proven as true (which it can’t) it nevertheless has dire consequences, which we have argued about on other threads, when we begin to consider meaning and morals. (Briefly, if God is not the ultimate explanation for our existence then there is no kind of ultimate explanation in the area of meaning and morals.)
The question of ultimate meaning, morality, truth/error, right/wrong, meaning/nonsense, is indeed an interesting and important issue that relates. I don't know that it is the knock-out blow we might want it to be. After all, materialists can just claim that meaning, consciousness, morality -- when they are willing to acknowledge the existence of such things -- somehow inexplicably arose as a byproduct of evolution or as some kind of emergent property. Nevertheless, you are quite right that the materialist claim is a terribly weak position, and is one that the theist can, and should, put pressure on to significant effect. Eric Anderson
Dionosio @17:
What would qualify as a substantive answer about the cause itself? What could help us to get to such an answer?
I suggest that there is rather little that can be said as a matter of either observation or deduction regarding the cause of the universe or regarding specific attributes of even the First Cause. That does not mean there is nothing that can be said. As I detailed, there are some scraps of evidence and some reasonable inferences that can be drawn. It seems to me the more reasonable conclusion is that an intelligent agent caused the universe and perhaps an intelligent agent is the First Cause. But such a conclusion needs to be cognizant of the limited ground it rests upon and needs to be cautious about not engaging in either circular reasoning or conflation of entities. Eric Anderson
KF @16: Good points and excellent quotes. Thanks. There is certainly ample evidence on which to infer design, and no design proponent need apologize for doing so. Although it is an inference (not a deduction), it is a stronger case than the essentially naked claim of some materialist creative "force" that allegedly produced the universe. ----- KF @18: I wonder if the evidentiary approach taken in the legal system is broadly consistent with what we would expect to see in science or in our lives practically? That said, it is an interesting quote worth considering. And your point about the old "extraordinary claim . . ." line of argumentation is well taken. More useful as a rhetorical tool to avoid engaging in serious debate than as a helpful tool for getting to the truth. However, I find it is sometimes more useful to simply turn such rhetoric back on the questioner. After all, what is more extraordinary than the claims of the materialist creation story? Yet the accompanying evidence is notable precisely for its embarrassing paucity. Eric Anderson
Origenes @31: I am using "First Cause" in the normal sense, the ultimate First Cause. What I am saying (among other things) is that the cause of the universe need not necessarily be the First Cause. Also, @30 (and previously) I am highlighting that the attributes often attributed to either cause are essentially circularly defined (taken as the negative of certain attributes in the universe), such that some people make broad claims about the nature or identity of the cause, when those claims might not be quite as well supported as they might hope. Eric Anderson
Anything that is caused requires a cause. It follows that there must be a first cause. Whether the universe had a beginning or not is irrelevant. If it has a cause for its existence, there must be a first cause. Perhaps the universe is eternal. So what. It still either requires a cause of its existence or it does not. If the universe can be uncaused, then by what principle must everything in the universe have a cause? If things with the universe can have no cause then science is a myth. Mung
rhampton7:
I really don’t understand the persistent need to justify an extra-universal first cause using logic that derives from within the universe.
At the risk of being repetitious I am going to repeat myself. LNC, LEM and LOI are not rules of logic. They are not derived from some system of logic. You cannot reason to them as a conclusion from some premises. Go ahead and try. Mung
Got to love rhampton7 who can imagine a realm that can both exist and not exist at the same time and in the same respect. Take the realm of God. Certainly that is a realm we know nothing of. Perhaps God both exists and does not exist. Yeah, that's probably the case. Mung
Rhampton7
I really don’t understand the persistent need to justify an extra-universal first cause using logic that derives from within the universe.
The point is to show that we don’t need to exercise our faith in order to know that God exists. We can figure it out by thinking clearly. Faith should be reserved for revealed truths that we cannot know any other way. Faith and reason are supposed to work together; neither should be expected to suffice for the other.
To borrow a phrase, it’s not even wrong. Truly, there is no way to reason what exists outside of space and time, nor by what logic, if any, applies.
We not only can, but we must reason about what exists outside of space and time. Reasons rules do not dissolve or become irrelevant at the edge of the universe. The laws of identity, causality, and non-contradiction hold in all possible universes. It not, they would be useless in the unknown and undiscovered territory of our own universe.
I doubt no one will ever equal Aquinas in his breadth of thought on the matter, but the whole edifice is built upon the assumption that our logic can perceive God and the Heavens. As beautiful as his work is, though, that is speculation at best. It is and always will be the greatest mystery that requires personal faith.
You are promoting Fideism, which is both a religious and an intellectual heresy. "(Latin fides, faith) A philosophical term meaning a system of philosophy or an attitude of mind, which, denying the power of unaided human reason to reach certitude, affirms that the fundamental act of human knowledge consists in an act of faith, and the supreme criterion of certitude is authority. As against these views, it must be noted that authority, even the authority of God, cannot be the supreme criterion of certitude, and an act of faith cannot be the primary form of knowledge. This authority, indeed, in order to be a motive of assent, must be previously acknowledged as being certainly valid; before we believe in a proposition as revealed by God, we must first know with certitude that God exists, that He reveals such and such a proposition, and that His teaching is worthy of assent, all of which questions can and must be ultimately decided only by an act of intellectual assent based on objective evidence. Thus, fideism not only denies intellectual knowledge, but logically ruins faith itself."
For example, the impossibility of infinite regress or eternal existence derives from what we know to be true, given our experience in time and space. But it would be a great mistake to assume that the same rules must apply outside of it. Please, do not mistake this admission as weakness or retreat, for it takes great courage and fortitude to believe, to have faith.
Religion should be allowed to illuminate our reason only if has first passed the test of reason. Christianity has passed the test of reason. Islam, Buddhism, and Hinduism have not. The worst advice we can give anyone is to say “just believe.” It is on the basis of that reckless doctrine that men mindlessly fly airplanes into buildings.
That such Divine faith is necessary, follows from the fact of Divine revelation. For revelation means that the Supreme Truth has spoken to man and revealed to him truths which are not in themselves evident to the human mind.
If you mean Divine faith in the truth of Christianity, I agree. If you mean Divine faith in anything, I disagree. Fideism is death for an individual’s temporal and eternal destiny.
We must, then, either reject revelation altogether, or accept it by faith; that is, we must submit our intellect to truths which we cannot understand, but which come to us on Divine authority.”
Again, if you mean we must give intellectual assent to legitimate authority, I agree; If you mean we must give intellectual assent to any authority at all, I disagree. Question, how do you discern one from the other? Answer—reason. I have good reasons for accepting the authority of Jesus Christ (and His Church). Muslims do not have good reasons for accepting the authority of Muhammed.
At best, the first cause rationalization (unaccompanied by revelation) is a deist exercise in argumentation and a stumbling block to forming true faith.
The first cause argument is a boost for forming the true faith. As we read in Romans 1:20: “For ever since the world was created, people have seen the earth and sky. Through everything God made, they can clearly see his invisible qualities--his eternal power and divine nature. So they have no excuse for not knowing God.” That is a form of the first cause argument. Notice that they have no excuse for rejecting God because his handiwork provides evidence for his existence. In this passage, they are condemned not for refusing to believe, which is a gift that comes to those with the right disposition, but for stubbornly refusing to acknowledge what they already know. If they will only take that small step and continue to follow the light they are given, faith in the higher revealed truths will come. The logical order of events is as follows: We come to know the creator God through unaided reason. Then we find the true God by continuing to follow the light we have been given. At that point, we submit our intellect and will to the true God—the Triune God of Christianity. StephenB
39 Nope, none of it is required or follows by necessity in a realm we know nothing of: laws, forces, truths, etc. rhampton7
I'm no physicist so the following question may very well show my ignorance, but how can an "eternal universe" not be in a state of "heat death" — a state of no thermodynamic free energy? How can it be that in an eternal universe suns have not already died? Origenes
RH7, pardon but once distinct identity applies, instantly LOI, LEM & LNC are applicable. Otherwise, meaningful communication vanishes. KF kairosfocus
I really don't understand the persistent need to justify an extra-universal first cause using logic that derives from within the universe. To borrow a phrase, it's not even wrong. Truly, there is no way to reason what exists outside of space and time, nor by what logic, if any, applies. I doubt no one will ever equal Aquinas in his breadth of thought on the matter, but the whole edifice is built upon the assumption that our logic can perceive God and the Heavens. As beautiful as his work is, though, that is speculation at best. It is and always will be the greatest mystery that requires personal faith. For example, the impossibility of infinite regress or eternal existence derives from what we know to be true, given our experience in time and space. But it would be a great mistake to assume that the same rules must apply outside of it. Please, do not mistake this admission as weakness or retreat, for it takes great courage and fortitude to believe, to have faith. Ultimately the purpose of such reasoning is to provide an apologia for those minds that demand reasons to believe. But that is not the foundation nor the sustenance of faith. Faith stems from two things: Natural reason, necessarily constrained by nature (the universe), and Divine reason:
"That such Divine faith is necessary, follows from the fact of Divine revelation. For revelation means that the Supreme Truth has spoken to man and revealed to him truths which are not in themselves evident to the human mind. We must, then, either reject revelation altogether, or accept it by faith; that is, we must submit our intellect to truths which we cannot understand, but which come to us on Divine authority."
At best, the first cause rationalization (unaccompanied by revelation) is a deist exercise in argumentation and a stumbling block to forming true faith. rhampton7
HeKS @34 Excellent. harry
StephenB @33 Absolutely. I got the impression that Eric presented this point as an "aside argument" in the OP, but it seems I was mistaken. Origenes
StephenB @33 Exactly. HeKS
Eric @30
Consider that at one point the universe was thought to be static and eternal. In that case, the universe itself was considered something that always existed, without need for a causative explanation. And the universe has matter, energy, space, time – the various characteristics and parameters we observe around us. Positing that those always existed, indeed that the universe itself was the ultimate unchanging reality, was not problematic logically, and we had no need to argue that a First Cause had to exist outside of space, time and matter. The universe was thought to exist as a fundamental, eternal, physical reality. Now shift forward to the current understanding where we say that the universe had a beginning. Now we need to explain the cause of the observable universe.
Your description here seems to suggest that the universe was uniformly believed to be eternal until scientists discovered it had a beginning, and only then was it decided that we were in need of an explanation for the universe. The problem is that this simply isn't the case. The physical universe was thought to be eternal by those operating under a particular materialistic perspective, but philosophical arguments for the logical necessity of physical reality having a beginning at a finite time in the past and requiring an external cause predated any scientific discovery of the expanding universe. And, of course, these philosophical arguments were further predated by the Judeo-Christian claims that physical reality did have a beginning at a finite time in the past and did have an external cause. Science is literally thousands of years late to this party. That said, where the fact that materialistic scientists and philosophers thought the physical universal existed as a brute eternal fact becomes relevant is in pointing out that it is hypocritical of them to have been willing to accept the infinite temporal existence of the physical universe (which is logically impossible) without demanding any external cause for it but then demanding an external cause for God, being unwilling to accept his eternal non-temporal existence (which is not logically impossible) as a brute fact. Take care, HeKS HeKS
Eric, I like this discussion and it is evident that you don’t mind tugging away at these ideas. So here goes: In response to Origenes, you wrote,
Now shift forward to the current understanding where we say that the universe had a beginning. Now we need to explain the cause of the observable universe. There is no reason in principle why there cannot be something physical – just like our old static universe – outside of our observable universe or dimension or whatever we want to call it.
Logically, this is possible, but this hypothetic static universe also consists of time/space/ matter and needs explaining for all the same reasons. If it is part of a larger multiverse, (or a product of the multiverse) then the multiverse network becomes the ultimate universe and we are back to where we started. It hardly matters whether we attach the prefix “uni “or “multi” to the ultimate cosmic time/space/continuum. The question remains the same: Does the ultimate time/space/continuum of impersonal forces (that once didn’t exist and now does exist) need a first cause and what must that first cause be like. If the cosmic time/space/matter continuum once didn’t exist and now does exist, there are only two possible candidates for a first cause: It must be either be an immaterial cause or a material cause. (We can rule out abstract principles because ideas and numbers cannot act as causes). Logically, time, space, and matter, however favorably they might be distributed or organized, can play no role in bringing time, space, and matter into existence. Otherwise, they would have had to exist before they existed, which is absurd. It would seem that only a timeless, spaceless, immaterial cause (being) can bring time, space, and matter into existence. StephenB
Eric Anderson @ 30
Logically, some form of existence must be in play for the cause of the universe. What is the nature of that existence? We might be able to say that the cause existed outside of our current observable parameters of space and time and matter that exist in our universe. That could be accurate.
Whatever it is, the nature of that existence isn't natural according to our understanding of the natural. It obviously transcends the natural. Therefore, it is entirely correct to say that it is supernatural. To then say that that supernatural reality might not be the uncaused first cause, sounds a lot to me like one takes seriously Dawkins' silly question, "Then who created God?" harry
Eric, I may have misunderstood you.
EA: 6. Ultimately, when traced back, there must be a First Cause – that which existed in and of itself, without a beginning.
I took it that you were referring to the 'ultimate' First Cause. The First Cause of everything — the multiverse if need be —, the ultimate foundation of all of reality. If instead you were referring to the First Cause of our universe, which can be an intermediate cause, then I have misunderstood you and my comment #15 is rather irrelevant. Origenes
Origenes @15: You make an interesting case, and one worth considering further. May I suggest, however, that it is not quite as water-tight as we might initially think. Consider that at one point the universe was thought to be static and eternal. In that case, the universe itself was considered something that always existed, without need for a causative explanation. And the universe has matter, energy, space, time – the various characteristics and parameters we observe around us. Positing that those always existed, indeed that the universe itself was the ultimate unchanging reality, was not problematic logically, and we had no need to argue that a First Cause had to exist outside of space, time and matter. The universe was thought to exist as a fundamental, eternal, physical reality. Now shift forward to the current understanding where we say that the universe had a beginning. Now we need to explain the cause of the observable universe. There is no reason in principle why there cannot be something physical – just like our old static universe – outside of our observable universe or dimension or whatever we want to call it. Yes, we can say that the cause of our universe had to exist outside of our universe. That is one of the deductions we can make by force of logic, and up to that point we are on solid ground. We might be tempted to go one step further and argue that the cause had to exist outside of space and time and matter as well. Yet – and here we must pay very close attention because it is critical to understand the chain of thinking – such a claim rests upon and is conditioned upon the proposition that these very parameters (space, time and matter) exist only in our universe. But that is only because we have defined them as such. ----- Logically, some form of existence must be in play for the cause of the universe. What is the nature of that existence? We might be able to say that the cause existed outside of our current observable parameters of space and time and matter that exist in our universe. That could be accurate. But that is a more careful and nuanced claim than the ones typically put forward. Eric Anderson
HeKS @8:
As such, we must conclude that the First Cause was non-extensional, and therefore did not exist within any external dimension of time or space.
This may indeed be a reasonable conclusion. I would just point out that it is more related to the First Cause, than the cause of the universe. Per my comments to Querius and harry @11-12, the latter claim would run the risk of appealing to circular definitions.
He can attack the logic of the argument if he likes, but to simply offer some physical alternative to the non-physical First Cause necessitated by the argument is to be non-responsive to the argument.
Fair enough. I agree that vague appeals to some unidentified materialistic cause are unpersuasive. As between the alternatives, I would not choose the materialistic explanation. I remain more cautious, however, about defining the First Cause in terms of specific parameters – particularly if those parameters are simply negations of whatever parameters we have defined for the “physical” universe.
And so the argument educates “science” by informing it that we are forced to conclude that there was some point at which there was either a) absolutely nothing at all or b) some non-extensional reality with the characteristics described in my OP, and logic forces us to conclude that it was (b).
I think you make a good point and a strong case, with perhaps the cautionary caveats I am trying to flesh out a bit.
P.S. Just to be clear, if you sense any antagonism in my tone, it is not remotely directed at you. It is entirely a result of the frivolity of sean samis’ arguments in combination with his arrogant demeanor in the comment thread to my OP. If only I had more time on my hands.
No need to apologize. I appreciate your comments and additional clarifications and certainly have not taken offense. Eric Anderson
rvb8 @7: Thanks for bringing it down a notch.
The fact that a human endeavour, mathematics, needs these dimensions to balance equations is not physical proof these dimensions exist . . .
Agreed.
. . . but they are rational proofs, which, I am sorry to say, is several giant steps of proof beyond philosophising.
I'm not sure where your negative accusations of philosophizing come in. Even assuming mathematics has "proved" extra dimensions beyond our known universe, it doesn't help answer the question of what the cause of the universe is. Nor does it help answer the question of the First Cause. Yet there are a few things we can say, based on a rational analysis of the situation. The OP contains very modest claims. Claims based upon the few things that can be deduced and the few things that can be reasonably inferred. I have even called upon those who posit God as the cause of the universe or as the First Cause to exercise some caution in that claim, due to the lack of empirical evidence or deductive necessity. If you had read the OP with some measure of objectivity, rather than coming in here with guns blazing, it would seems that you would be applauding my cautionary tone, my specific call to reign in the religion and the philosophy, rather than making abusive accusations of "philosophizing."
I’m sorry if my tone sounds arrogant. I lay no claim to superior knowledge, indeed most of the contributors here, appear far more knowledgeable, about far more subjects than I do. However the, ‘extraordinary claims, demand extraordinary evidence’, is justified here, and I make no apologies for that.
One might point out that the issues we are discussing are more related to logic and first principles of reason, than observational evidence, given the acknowledged paucity of the latter. Even so, I am certainly happy to accept the "extraordinary claims, demand extraordinary evidence" concept. Presumably you are willing to apply this principle to the extraordinary naturalistic claims about the origin of the universe? Eric Anderson
JAD,
Sagan thought it was at least possible. He thought it was possible we lived in an oscillating universe that has gone through an infinite number of cycles, each cycle beginning with a new Big Bang which then ultimately collapses on itself. However that idea has since been discredited. It is now known that the universe is expanding too quickly to ever collapse back on itself. So, we do not live in an oscillating universe.
That's interesting. I didn't know that Sagan had written on the issue of infinite regress. daveS
To follow up on my comment @ #19: Many of our interlocutors here try to use the “so many religions” canard as an objection against theism. But the basic forms of religious theism agree on a fundamental metaphysical level when it comes to the origin of the universe, life, mind and consciousness. Indeed, one doesn’t need to be religious, like the late Anthony Flew, to accept theism as the best explanation why the world exists and why it exists as it does. Furthermore, notice the ontological demarcation between basic theism and every other world view. Only theism does not rely on an infinite regress to explain our existence. At least I can’t think of another worldview that doesn’t require an infinite regress… can you? john_a_designer
..any more than I need to prove that a moving train of box cars needs an engine to move them. StephenB
rvb8 @3
You cannot prove a first cause, just as I cannot disprove one. However the burden of proof is upon the, ‘extraordinary claim’. (Marcello Truzzi.)
That is pure, unadulterated nonsense. I don't need to prove that a causal chain needs a first cause any more than I need to prove that a train of moving train of box cars needs an engine to move them. Every reasonable person already knows that. You, on the other hand, do need to prove that a chain of box cars can move even when there is no engine to do the moving. It is your claim that is extraordinary. StephenB
PS: As parts of that onward discussion, we could highlight:
1 --> absent responsible, rational freedom, we cannot trust ourselves to argue reasonably. 2 --> such responsible, rational freedom allows us to be self-moved initiating actors, significantly free and rational, enconscienced agents. 3 --> In that context, we see abundant cases -- past a trillion -- of designs of functionally specific complex organisation and associated information [FSCO/I] and in every observed case the cause is such a designing agent as is being discussed. 4 --> We have no good reason to infer we exhaoust the possible cases of such designers. 5 --> Nor can we insist that design is a result of purely material action, indeed the evidence in hand points otherwise. For example, here is Reppert:
. . . let us suppose that brain state A, which is token identical to the thought that all men are mortal, and brain state B, which is token identical to the thought that Socrates is a man, together cause the belief that Socrates is mortal. It isn’t enough for rational inference that these events be those beliefs, it is also necessary that the causal transaction be in virtue of the content of those thoughts . . . [[But] if naturalism is true, then the propositional content is irrelevant to the causal transaction that produces the conclusion, and [[so] we do not have a case of rational inference. In rational inference, as Lewis puts it, one thought causes another thought not by being, but by being seen to be, the ground for it. But causal transactions in the brain occur in virtue of the brain’s being in a particular type of state that is relevant to physical causal transactions.
6 --> So we have strong reason to infer from FSCO/I and/or similar signs to design working by art. Here, I cite again Plato, earlier in The Laws Bk X:
Ath. . . . we have . . . lighted on a strange doctrine. Cle. What doctrine do you mean? Ath. The wisest of all doctrines, in the opinion of many. Cle. I wish that you would speak plainer. Ath. The doctrine that all things do become, have become, and will become, some by nature, some by art, and some by chance. Cle. Is not that true? Ath. Well, philosophers are probably right; at any rate we may as well follow in their track, and examine what is the meaning of them and their disciples. Cle. By all means. Ath. They say that the greatest and fairest things are the work of nature and of chance, the lesser of art, which, receiving from nature the greater and primeval creations, moulds and fashions all those lesser works which are generally termed artificial . . . . . fire and water, and earth and air, all exist by nature and chance . . . The elements are severally moved by chance and some inherent force according to certain affinities among them . . . After this fashion and in this manner the whole heaven has been created, and all that is in the heaven, as well as animals and all plants, and all the seasons come from these elements, not by the action of mind, as they say, or of any God, or from art, but as I was saying, by nature and chance only [--> evolutionary materialism is ancient] . . . . Nearly all of them, my friends, seem to be ignorant of the nature and power of the soul [i.e. mind], especially in what relates to her origin: they do not know that she is among the first of things, and before all bodies, and is the chief author of their changes and transpositions. And if this is true, and if the soul is older than the body, must not the things which are of the soul's kindred be of necessity prior to those which appertain to the body? . . . . if the soul turn out to be the primeval element, and not fire or air, then in the truest sense and beyond other things the soul may be said to exist by nature; and this would be true if you proved that the soul is older than the body, but not otherwise . . .
7 --> In short since Plato 2360 years ago near enough, it has been understood that we may contrast blind natural mechanism of necessity and/or blind chance with the ART-ificial. (The common rhetorical gambit to try to pin down to "natural vs supernatural" is question-begging, the proper contrast is nature working by chance and necessity vs the ART-ificial, working by design. When it comes form those who are at the top of the academic game, it is grossly irresponsible, at best and willfully deceitful at worst.) 8 --> So, we may freely infer to design on FSCO/I or the like, as tested, reliable inductive signs. 9 --> One of those signs is that often designed complex systems exhibit fine-tuning, close adaptation and/or adjustment of parts to work together when properly organised. This is obvious with concrete entities such as vehicles or electronic circuits, it also applies to text in writing, and to complex frames of argument or thought, i.e. to abstract entities, which may be the plans so to speak for concrete things. In short, functionally specific complex information is designed -- where we may use a threshold of 500 - 1,000 bits. 10 --> As Sir Fred Hoyle and many others have had to highlight, the physics and parameters of our observed cosmos set us to a deeply isolated operating point that facilitates C-chemistry, aqueous medium, protein using, cell based life on terrestrial planets. 11 --> This is a sign that points to design of our cosmos by a highly intelligent and powerful, deeply knowledgeable agent or set of such agents. 12 --> Multiply this by the logic of being and non-being, and we see that at the root of our world, even through a multiverse speculation, is a required necessary being of ensouled order. 13 --> This becomes particularly evident when we see the need to account for ourselves as responsibly free intelligences of ensouled/ minded/ agent order, who are morally governed. There must be a world root IS capable of grounding OUGHT.
The issue, is serious candidates to meet these requisites. At world root level, after centuries of debates, there is but one serious candidate, the inherently good creator God, a necessary and maximally great [= supreme] being, worthy of ultimate loyalty and the responsible, reasonable service of doing the good in accord with our evident nature. If you doubt this, simply put up a candidate to face comparative difficulties: _____________________ (Prediction, on long experience, no such serious alternative will be forthcoming.) kairosfocus
Before that exchange with Carnap, Einstein was denied a Nobel prize for relativity largely because of a debate he had with another philosopher over basically the same line of thought. i.e. over 'The Now' of the mind:
Einstein vs Bergson, science vs philosophy and the meaning of time - Wednesday 24 June 2015 Excerpt: The meeting of April 6 was supposed to be a cordial affair, though it ended up being anything but. ‘I have to say that day exploded and it was referenced over and over again in the 20th century,’ says Canales. ‘The key sentence was something that Einstein said: “The time of the philosophers did not exist.”’ It’s hard to know whether Bergson was expecting such a sharp jab. In just one sentence, Bergson’s notion of duration—a major part of his thesis on time—was dealt a mortal blow. As Canales reads it, the line was carefully crafted for maximum impact. ‘What he meant was that philosophers frequently based their stories on a psychological approach and [new] physical knowledge showed that these philosophical approaches were nothing more than errors of the mind.’ The night would only get worse. ‘This was extremely scandalous,’ says Canales. ‘Einstein had been invited by philosophers to speak at their society, and you had this physicist say very clearly that their time did not exist.’ Bergson was outraged, but the philosopher did not take it lying down. A few months later Einstein was awarded the Nobel Prize for the discovery of the law of photoelectric effect, an area of science that Canales noted, ‘hardly jolted the public’s imagination’. In truth, Einstein coveted recognition for his work on relativity. Bergson inflicted some return humiliation of his own. By casting doubt on Einstein’s theoretical trajectory, Bergson dissuaded the committee from awarding the prize for relativity. In 1922, the jury was still out on the correct interpretation of time. So began a dispute that festered for years and played into the larger rift between physics and philosophy, science and the humanities. Bergson was fond of saying that time was the experience of waiting for a lump of sugar to dissolve in a glass of water. It was a declaration that one could not talk about time without reference to human consciousness and human perception. Einstein would say that time is what clocks measure. Bergson would no doubt ask why we build clocks in the first place. ‘He argued that if we didn’t have a prior sense of time we wouldn’t have been led to build clocks and we wouldn’t even use them ... unless we wanted to go places and to events that mattered,’ says Canales. ‘You can see that their points of view were very different.’ In a theoretical nutshell this expressed perfectly the division between lived time and spacetime: subjective experience versus objective reality.,,, Just when Einstein thought he had it worked out, along came the discovery of quantum theory and with it the possibility of a Bergsonian universe of indeterminacy and change. God did, it seems, play dice with the universe, contra to Einstein’s famous aphorism. Some supporters went as far as to say that Bergson’s earlier work anticipated the quantum revolution of Niels Bohr and Werner Heisenberg by four decades or more. Canales quotes the literary critic Andre Rousseaux, writing at the time of Bergson’s death. ‘The Bergson revolution will be doubled by a scientific revolution that, on its own, would have demanded the philosophical revolution that Bergson led, even if he had not done it.’ Was Bergson right after all? Time will tell. http://www.abc.net.au/radionational/programs/philosopherszone/science-vs-philosophy-and-the-meaning-of-time/6539568 Einstein, Bergson, and the Experiment that Failed: Intellectual Cooperation at the League of Nations! - Jimena Canales page 1177 Excerpt: Bergson temporarily had the last word during their meeting at Société française de philosophie. His intervention negatively affected Einstein’s Nobel Prize, which was given “for his services to theoretical physics, and especially for his discovery of the law of the photoelectric effect” and not for relativity. The reasons behind this decision, as stated in the prize’s presentation speech, were related to Bergson’s intervention: “Most discussion [of Einstein’s work] centers on his Theory of Relativity. This pertains to epistemology and has therefore been the subject of lively debate in philosophical circles. It will be no secret that the famous philosopher Bergson in Paris has challenged this theory, while other philosophers have acclaimed it wholeheartedly.”51 For a moment, their debate dragged matters of time out of the solid terrain of “matters of fact” and into the shaky ground of “matters of concern.”52 https://dash.harvard.edu/bitstream/handle/1/3210598/canales-Einstein,%20Bergson%20and%20the%20Experiment%20that%20Failed%282%29.pdf?sequence=2 Einstein vs. "The Now" of Philosophers and Quantum Mechanics – video https://www.facebook.com/philip.cunningham.73/videos/vb.100000088262100/1129789497033982/?type=2&theater
As alluded to in the preceding video, the statement from Einstein to Carnap on the train, '"The experience of 'the now' cannot be turned into an object of physical measurement’, was an interesting statement for Einstein to make to the philosopher since 'the now of the mind' has, from many recent experiments in quantum mechanics, undermined the space-time of Einstein's General Relativity as to being the absolute frame of reference for reality.
LIVING IN A QUANTUM WORLD - Vlatko Vedral - 2011 Excerpt: Thus, the fact that quantum mechanics applies on all scales forces us to confront the theory’s deepest mysteries. We cannot simply write them off as mere details that matter only on the very smallest scales. For instance, space and time are two of the most fundamental classical concepts, but according to quantum mechanics they are secondary. The entanglements are primary. They interconnect quantum systems without reference to space and time. If there were a dividing line between the quantum and the classical worlds, we could use the space and time of the classical world to provide a framework for describing quantum processes. But without such a dividing line—and, indeed, with­out a truly classical world—we lose this framework. We must explain space and time (4D space-time) as somehow emerging from fundamentally spaceless and timeless physics. http://phy.ntnu.edu.tw/~chchang/Notes10b/0611038.pdf Experiment confirms quantum theory weirdness - May 27, 2015 Excerpt: The bizarre nature of reality as laid out by quantum theory has survived another test, with scientists performing a famous experiment and proving that reality does not exist until it is measured. Physicists at The Australian National University (ANU) have conducted John Wheeler's delayed-choice thought experiment, which involves a moving object that is given the choice to act like a particle or a wave. Wheeler's experiment then asks - at which point does the object decide? Common sense says the object is either wave-like or particle-like, independent of how we measure it. But quantum physics predicts that whether you observe wave like behavior (interference) or particle behavior (no interference) depends only on how it is actually measured at the end of its journey. This is exactly what the ANU team found. "It proves that measurement is everything. At the quantum level, reality does not exist if you are not looking at it," said Associate Professor Andrew Truscott from the ANU Research School of Physics and Engineering. Despite the apparent weirdness, the results confirm the validity of quantum theory, which,, has enabled the development of many technologies such as LEDs, lasers and computer chips. The ANU team not only succeeded in building the experiment, which seemed nearly impossible when it was proposed in 1978, but reversed Wheeler's original concept of light beams being bounced by mirrors, and instead used atoms scattered by laser light. "Quantum physics' predictions about interference seem odd enough when applied to light, which seems more like a wave, but to have done the experiment with atoms, which are complicated things that have mass and interact with electric fields and so on, adds to the weirdness," said Roman Khakimov, PhD student at the Research School of Physics and Engineering. http://phys.org/news/2015-05-quantum-theory-weirdness.html
Moreover, in trying to ascertain what are the characteristics the Mind that has brought, and continually brings, material reality into existence, it is important to note that a photon, while it is in its uncollapsed state, is mathematically defined as being in an infinite dimensional state. Moreover, it is in an infinite dimensional state that requires an infinite amount of information to define properly:
The Unreasonable Effectiveness of Mathematics in the Natural Sciences - Eugene Wigner - 1960 Excerpt: We now have, in physics, two theories of great power and interest: the theory of quantum phenomena and the theory of relativity.,,, The two theories operate with different mathematical concepts: the four dimensional Riemann space and the infinite dimensional Hilbert space, http://www.dartmouth.edu/~matc/MathDrama/reading/Wigner.html Explaining Information Transfer in Quantum Teleportation: Armond Duwell †‡ University of Pittsburgh Excerpt: In contrast to a classical bit, the description of a (photon) qubit requires an infinite amount of information. The amount of information is infinite because two real numbers are required in the expansion of the state vector of a two state quantum system (Jozsa 1997, 1) http://www.cas.umt.edu/phil/faculty/duwell/DuwellPSA2K.pdf Double Slit, Quantum-Electrodynamics, and Christian Theism – video https://www.facebook.com/philip.cunningham.73/videos/vb.100000088262100/1127450170601248/?type=2&theater
i.e. Whatever, or more precisely, Whomever is collapsing the infinite dimensional-infinite information photon to a single bit state must possess infinite knowledge and also be infinitely present! Which certainly sounds like almighty God to me!
Job 38:19-20 “What is the way to the abode of light? And where does darkness reside? Can you take them to their places? Do you know the paths to their dwellings?”
Supplemental quotes:
"The most fundamental definition of reality is not matter or energy, but information–and it is the processing of information that lies at the root of all physical, biological, economic, and social phenomena." Vlatko Vedral - Professor of Physics at the University of Oxford, and CQT (Centre for Quantum Technologies) at the National University of Singapore, and a Fellow of Wolfson College - a recognized leader in the field of quantum mechanics. “It is operationally impossible to separate Reality and Information” (48:35 minute mark) “In the beginning was the Word” John 1:1 (49:54 minute mark) Prof Anton Zeilinger speaks on quantum physics. at UCT https://youtu.be/s3ZPWW5NOrw?t=2984
Verse:
John 1:1-4 In the beginning was the Word, and the Word was with God, and the Word was God. He was with God in the beginning. Through him all things were made; without him nothing was made that has been made. In him was life, and that life was the light of all mankind.
bornagain77
In ascertaining what brought the universe, i.e. space-time matter-energy, into being it is interesting to focus on the characteristics of the two entities that space-time matter-energy have utterly failed to explain the origin of. Namely, as everybody who has been around ID for any amount of time knows, matter-energy space-time have utterly failed to explain the origin of both information and of mind. Moreover, both information and mind have the characteristic of being timeless and both also have the characteristic of not being constrained by space. For instance, Berlinski comments on the timeless-spaceless characteristic of mathematical information in the following excerpt:
An Interview with David Berlinski - Jonathan Witt Berlinski: There is no argument against religion that is not also an argument against mathematics. Mathematicians are capable of grasping a world of objects that lies beyond space and time…. Interviewer:… Come again(?) … Berlinski: No need to come again: I got to where I was going the first time. The number four, after all, did not come into existence at a particular time, and it is not going to go out of existence at another time. It is neither here nor there. Nonetheless we are in some sense able to grasp the number by a faculty of our minds. Mathematical intuition is utterly mysterious. So for that matter is the fact that mathematical objects such as a Lie Group or a differentiable manifold have the power to interact with elementary particles or accelerating forces. But these are precisely the claims that theologians have always made as well – that human beings are capable by an exercise of their devotional abilities to come to some understanding of the deity; and the deity, although beyond space and time, is capable of interacting with material objects. http://tofspot.blogspot.com/2013/10/found-upon-web-and-reprinted-here.html
And although information can be represented by a material medium, information is still its own independent entity that is 'real' and that is not reducible to, or 'emergent' from, a material medium as Darwinists falsely presuppose. That information is real and produces real effects in the material world is highlighted by George Ellis in the following article.
Recognising Top-Down Causation - George Ellis Excerpt: page 5: A: Causal Efficacy of Non Physical entities: Both the program and the data are non-physical entities, indeed so is all software. A program is not a physical thing you can point to, but by Definition 2 it certainly exists. You can point to a CD or flashdrive where it is stored, but that is not the thing in itself: it is a medium in which it is stored. The program itself is an abstract entity, shaped by abstract logic. Is the software “nothing but” its realisation through a specific set of stored electronic states in the computer memory banks? No it is not because it is the precise pattern in those states that matters: a higher level relation that is not apparent at the scale of the electrons themselves. It’s a relational thing (and if you get the relations between the symbols wrong, so you have a syntax error, it will all come to a grinding halt). This abstract nature of software is realised in the concept of virtual machines, which occur at every level in the computer hierarchy except the bottom one [17]. But this tower of virtual machines causes physical effects in the real world, for example when a computer controls a robot in an assembly line to create physical artefacts. Excerpt page 7: The assumption that causation is bottom up only is wrong in biology, in computers, and even in many cases in physics, for example state vector preparation, where top-down constraints allow non-unitary behaviour at the lower levels. It may well play a key role in the quantum measurement problem (the dual of state vector preparation) [5]. One can bear in mind here that wherever equivalence classes of entities play a key role, such as in Crutchfield’s computational mechanics [29], this is an indication that top-down causation is at play. http://fqxi.org/data/essay-contest-files/Ellis_FQXI_Essay_Ellis_2012.pdf
Here are a few more quotes along the same line:
"Those devices (computers) can yield only approximations to a structure (of information) that has a deep and "computer independent" existence of its own." - Roger Penrose - The Emperor's New Mind - Pg 147 “The mechanical brain does not secrete thought “as the liver does bile,” as the earlier materialists claimed, nor does it put it out in the form of energy, as the muscle puts out its activity. Information is information, not matter or energy. No materialism which does not admit this can survive at the present day. “ Norbert Weiner - MIT Mathematician - (Cybernetics, 2nd edition, p.132)
That information is a real entity that produces real effects in the world is made evident by modern technology. Mathematical and/or logical information 'underlies virtually all of our technology today"
Describing Nature With Math By Peter Tyson - Nov. 2011 Excerpt: Mathematics underlies virtually all of our technology today. James Maxwell's four equations summarizing electromagnetism led directly to radio and all other forms of telecommunication. E = mc2 led directly to nuclear power and nuclear weapons. The equations of quantum mechanics made possible everything from transistors and semiconductors to electron microscopy and magnetic resonance imaging. Indeed, many of the technologies you and I enjoy every day simply would not work without mathematics. When you do a Google search, you're relying on 19th-century algebra, on which the search engine's algorithms are based. When you watch a movie, you may well be seeing mountains and other natural features that, while appearing as real as rock, arise entirely from mathematical models. When you play your iPod, you're hearing a mathematical recreation of music that is stored digitally; your cell phone does the same in real time. "When you listen to a mobile phone, you're not actually hearing the voice of the person speaking," Devlin told me. "You're hearing a mathematical recreation of that voice. That voice is reduced to mathematics." http://www.pbs.org/wgbh/nova/physics/describing-nature-math.html
That information can be represented by a material medium, but is not reducible to material medium, is also made self-evident by the fact that the same exact information can be represented by a practically unlimited set of material substrates, (CDs, DVDs, Floppy Disks, RAM, Magnetic Tape, etc.. etc..), and yet the meaning and/or content of the information never changes no matter which type of material substrate the information may be encoded on.
“In the nineteenth century we thought that there were two fundamental entities in science; matter, and energy. At the beginning of the twenty first century, we now recognize that there’s a third fundamental entity; and its ‘information’. It’s not reducible to matter. It’s not reducible to energy. But it’s still a very important thing that is real; we buy it, we sell it, we send it down wires. Now, what do we make of the fact, that information is present at the very root of all biological function? In biology, we have matter, we have energy, but we also have this third, very important entity; information. I think the biology of the information age, poses a fundamental challenge to any materialistic approach to the origin of life.” -Dr. Stephen C. Meyer earned his Ph.D. in the History and Philosophy of science from Cambridge University for a dissertation on the history of origin-of-life biology and the methodology of the historical sciences. Intelligent design: Why can't biological information originate through a materialistic process? - Stephen Meyer - video http://www.youtube.com/watch?v=wqiXNxyoof8
The same timeless-spaceless characteristics found for information are also found for mind. For instance, although the material particulars of my brain, and body, have certainly changed many times over since I was born, "I" have stayed the same. That is to say, "I" have a persistence of self identity that simply is not reducible to any material substrate. In fact, there are many characteristics of mind that simply are not reducible to any material substrate:
How Consciousness Points to the Existence of God - J. Warner Wallace - video - Sept. 2015 (5 attributes of mind that are distinct from the material brain therefore, via the law of identity, the mind is not the same thing as the material brain) https://www.youtube.com/watch?v=8Ff1jiRpjko
That mind has a timeless/spaceless characteristic to it, that is not reducible to matter and/or energy, is also highlighted by an infamous debate that Einstein had with philosophers. (A debate which, due to advances in quantum mechanics, Einstein ultimately lost). Einstein was once asked (by a philosopher):
"Can physics demonstrate the existence of 'the now' in order to make the notion of 'now' into a scientifically valid term?"
Einstein's answer was categorical, he said:
"The experience of 'the now' cannot be turned into an object of physical measurement, it can never be a part of physics."
Quote was taken from the last few minutes of this following video.
Stanley L. Jaki: "The Mind and Its Now" https://vimeo.com/10588094
Here is an article on the exchange between Einstein and the philosopher:
The Mind and Its Now - May 22, 2008 - By Stanley L. Jaki Excerpt: ,,, Rudolf Carnap, and the only one among them who was bothered with the mind's experience of its now. His concern for this is noteworthy because he went about it in the wrong way. He thought that physics was the only sound way to know and to know anything. It was therefore only logical on his part that he should approach, we are around 1935, Albert Einstein, the greatest physicist of the day, with the question whether it was possible to turn the experience of the now into a scientific knowledge. Such knowledge must of course be verified with measurement. We do not have the exact record of Carnap's conversation with Einstein whom he went to visit in Princeton, at eighteen hours by train at that time from Chicago. But from Einstein's reply which Carnap jotted down later, it is safe to assume that Carnap reasoned with him as outlined above. Einstein's answer was categorical: The experience of the now cannot be turned into an object of physical measurement. It can never be part of physics. http://metanexus.net/essay/mind-and-its-now
What is meant by 'the Now' can be read in full context in the following article:
The Mind and Its Now - Stanley L. Jaki, May 2008 Excerpts: There can be no active mind without its sensing its existence in the moment called now.,,, Three quarters of a century ago Charles Sherrington, the greatest modern student of the brain, spoke memorably on the mind's baffling independence of the brain. The mind lives in a self-continued now or rather in the now continued in the self. This life involves the entire brain, some parts of which overlap, others do not. ,,,There is no physical parallel to the mind's ability to extend from its position in the momentary present to its past moments, or in its ability to imagine its future. The mind remains identical with itself while it lives through its momentary nows. ,,, the now is immensely richer an experience than any marvelous set of numbers, even if science could give an account of the set of numbers, in terms of energy levels. The now is not a number. It is rather a word, the most decisive of all words. It is through experiencing that word that the mind comes alive and registers all existence around and well beyond. ,,, All our moments, all our nows, flow into a personal continuum, of which the supreme form is the NOW which is uncreated, because it simply IS. http://metanexus.net/essay/mind-and-its-now
bornagain77
F/N: Pardon, a clip from Plato in The Laws, Bk X, in which he discusses first causes [note plural] and first cause of the cosmos [note distinction]:
[Athenian Stranger]. Then, by Heaven, we have discovered the source of this vain opinion of all those physical investigators; and I would have you examine their arguments with the utmost care, for their impiety is a very serious matter; they not only make a bad and mistaken use of argument, but they lead away the minds of others: that is my opinion of them. Cle. You are right; but I should like to know how this happens. Ath. I fear that the argument may seem singular. Cle. Do not hesitate, Stranger; I see that you are afraid of such a discussion carrying you beyond the limits of legislation. But if there be no other way of showing our agreement in the belief that there are Gods, of whom the law is said now to approve, let us take this way, my good sir. Ath. Then I suppose that I must repeat the singular argument of those who manufacture the soul according to their own impious notions; they affirm that which is the first cause of the generation and destruction of all things, to be not first, but last, and that which is last to be first, and hence they have fallen into error about the true nature of the Gods. Cle. Still I do not understand you. Ath. Nearly all of them, my friends, seem to be ignorant of the nature and power of the soul [[ = psuche], especially in what relates to her origin: they do not know that she is among the first of things, and before all bodies, and is the chief author of their changes and transpositions. And if this is true, and if the soul is older than the body, must not the things which are of the soul’s kindred be of necessity prior to those which appertain to the body? Cle. Certainly. Ath. Then thought and attention and mind and art and law will be prior to that which is hard and soft and heavy and light; and the great and primitive works and actions will be works of art; they will be the first, and after them will come nature and works of nature, which however is a wrong term for men to apply to them; these will follow, and will be under the government of art and mind. Cle. But why is the word “nature” wrong? Ath. Because those who use the term mean to say that nature is the first creative power; but if the soul turn out to be the primeval element, and not fire or air, then in the truest sense and beyond other things the soul may be said to exist by nature; and this would be true if you proved that the soul is older than the body, but not otherwise. [[ . . . .] Ath. . . . when one thing changes another, and that another, of such will there be any primary changing element? How can a thing which is moved by another ever be the beginning of change? Impossible. But when the self-moved changes other, and that again other, and thus thousands upon tens of thousands of bodies are set in motion, must not the beginning of all this motion be the change of the self-moving principle? . . . . self-motion being the origin of all motions, and the first which arises among things at rest as well as among things in motion, is the eldest and mightiest principle of change, and that which is changed by another and yet moves other is second. [[ . . . .] Ath. If we were to see this power existing in any earthy, watery, or fiery substance, simple or compound-how should we describe it? Cle. You mean to ask whether we should call such a self-moving power life? Ath. I do. Cle. Certainly we should. Ath. And when we see soul in anything, must we not do the same-must we not admit that this is life? [[ . . . . ] Cle. You mean to say that the essence which is defined as the self-moved is the same with that which has the name soul? Ath. Yes; and if this is true, do we still maintain that there is anything wanting in the proof that the soul is the first origin and moving power of all that is, or has become, or will be, and their contraries, when she has been clearly shown to be the source of change and motion in all things? Cle. Certainly not; the soul as being the source of motion, has been most satisfactorily shown to be the oldest of all things. Ath. And is not that motion which is produced in another, by reason of another, but never has any self-moving power at all, being in truth the change of an inanimate body, to be reckoned second, or by any lower number which you may prefer? Cle. Exactly. Ath. Then we are right, and speak the most perfect and absolute truth, when we say that the soul is prior to the body, and that the body is second and comes afterwards, and is born to obey the soul, which is the ruler? [[ . . . . ] Ath. If, my friend, we say that the whole path and movement of heaven, and of all that is therein, is by nature akin to the movement and revolution and calculation of mind, and proceeds by kindred laws, then, as is plain, we must say that the best soul takes care of the world and guides it along the good path. [[Plato here explicitly sets up an inference to design (by a good soul) from the intelligible order of the cosmos.]
So, Plato suggests that we should distinguish first causes --
(minded, ensouled, self-moved reflexive entities in effect that show freedom to initiate chains of events, as when I key certain keys in patters to compose this post . . . if such is driven and wholly controlledby chance and/or mechanical necessity it undermines argument through self referentiality)
. . . from the first cause of the cosmos in our discussion. I think this will allow us to proceed more effectively. KF kairosfocus
In the modern west the debate for the past two centuries has centered around two world view. On one hand, there is naturalism; on the other, there is theism. Naturalism, indeed as well as any other world view other than theism, leads to an infinite regress of causes (“turtles all the way down.”) For example, the late Cornell university astronomer Carl Sagan who took the side of naturalism over theism, or as he described it, between science and religion included in his book Broca’s Brain, a chapter titled, “A Sunday Sermon,” where he appears to vacillate about the relationship of science and religion. At times he seems to be sounding a conciliatory note, but then, at other times, he’s confrontational. For example, he writes, “A universe that is infinitely old and a God who is infinitely old are, I think, equally deep mysteries.” However, a few pages earlier he praises a book by Cornell universities’ founder and president, Andrew Dickson White, entitled, A History of the Warfare of Science with Theology in Christendom. Despite Sagan’s enthusiastic endorsement, White’s book has almost universally been discredited by historians of science as being more an anti-religious propaganda piece rather than a work of serious scholarship. Most historians of science reject the so-called warfare thesis put forth in White’s book as a myth. The relationship between science and the Christian faith is much more complicated and nuanced than White implies. Sagan, however, appears to uncritically swallow White’s thesis hook-line-and-sinker. As a Christian-theist, who has thought long and deeply about the basic assumptions underlying my world view, I don’t think Sagan, along with other likeminded naturalist’s, really understand the fundamental differences between the two world views. They are not really equal. Again, the naturalistic worldview that Sagan seemed to prefer, requires an infinite regress of causes. However, is such an infinite regress something that is scientifically provable? Is it even possible? Sagan thought it was at least possible. He thought it was possible we lived in an oscillating universe that has gone through an infinite number of cycles, each cycle beginning with a new Big Bang which then ultimately collapses on itself. However that idea has since been discredited. It is now known that the universe is expanding too quickly to ever collapse back on itself. So, we do not live in an oscillating universe. However, Sagan also thought that mathematics was on his side. He writes,
Humans seem to have a natural abhorrence of an infinite regression of causes, and this distaste is at the root of the most famous and most effective demonstrations of the existence of God by Aristotle and Thomas Aquinas. But these thinkers lived before the infinite series was a mathematical commonplace. If the differential and integral calculus or transfinite arithmetic had been invented in Greece in the fifth century B.C., and not subsequently suppressed, the history of religion in the West might have been very different-or at any rate we would have seen less of the pretension that theological doctrine can be convincingly demonstrated by rational argument to those who reject alleged divine revelation, as Aquinas attempted in the Summa Contra Gentiles.(p.335)
The famous German mathematician David Hilbert would have disagreed. He wrote, “The in?nite is nowhere to be found in reality. It neither exists in nature nor provides a legitimate basis for rational thought… The role that remains for the in?nite to play is solely that of an idea.” So, according to Hilbert an infinite sequence of real causes does not exist. Of course, other mathematicians would disagree. But the fact that mathematicians disagree about the existence of actual infinities cast doubt on the idea that the theological arguments would have been easily undermined. Indeed one could just as well argue that it would have had little effect over the status quo. It certainly doesn’t provide the knockout argument that Sagan thought it would. But there is more bad news for Sagan… One of the inventors of differential and integral calculus, Gottfried Wilhelm Leibniz, also had problems with an infinite regress. However, his argument wasn’t really mathematical but philosophical. According to Leibniz:
For a sufficient reason for existence cannot be found merely in any one individual thing or even in the whole aggregate and series of things. Let us imagine the book on the Elements of Geometry to have been eternal, one copy always being made from another; then it is clear that though we can give a reason for the present book based on the preceding book from which it was copied, we can never arrive at a complete reason, no matter how many books we may assume in the past, for one can always wonder why such books should have existed at all times; why there should be books at all, and why they should be written in this way. What is true of books is true also of the different states of the world; every subsequent state is somehow copied from the preceding one (although according to certain laws of change). No matter how far we may have gone back to earlier states, therefore, we will never discover in them a full reason why there should be a world at all, and why it should be such as it is.
Gottfried Wilhelm Leibniz, Philosophical Papers and Letters, ed. by Leroy E. Loemker (Kluwer Academic, 1989); p. 486. Basically, Leibniz’s argument is that with an infinite regress you never reach the ultimate explanation. Of course, the naturalist could still argue that maybe there is no ultimate explanation. Bertrand Russell conceded as much in his 1948 BBC radio debate with Fr. Fredrick Copleston about the existence of God. However, even if that could be proven as true (which it can’t) it nevertheless has dire consequences, which we have argued about on other threads, when we begin to consider meaning and morals. (Briefly, if God is not the ultimate explanation for our existence then there is no kind of ultimate explanation in the area of meaning and morals.) john_a_designer
F/N: The selectively hyperskeptical rhetorical gambit that "extraordinary claims require extraordinary evidence" is little more than an announcement of intent to beg the question in favour of one's present views and sense of what is plausible -- of course, coloured by those views. Instead, a more correct approach is:
extraordinary claims require extraordinary evidence [--> cogent, adequate evidence as with claims of similar type]
Simon Greenleaf, a founding father of the modern theory of evidence in anglophone jurisprudence, was far sounder:
Evidence, in legal acceptation, includes all the means by which any alleged matter of fact, the truth of which is submitted to investigation, is established or disproved . . . None but mathematical truth is susceptible of that high degree of evidence, called demonstration, which excludes all possibility of error [--> Greenleaf wrote almost 100 years before Godel], and which, therefore, may reasonably be required in support of every mathematical deduction. Matters of fact are proved by moral evidence alone; by which is meant, not only that kind of evidence which is employed on subjects connected with moral conduct, but all the evidence which is not obtained either from intuition, or from demonstration. In the ordinary affairs of life, we do not require demonstrative evidence, because it is not consistent with the nature of the subject, and to insist upon it would be unreasonable and absurd. The most that can be affirmed of such things, is, that there is no reasonable doubt concerning them. The true question, therefore, in trials of fact, is not whether it is possible that the testimony may be false, but, whether there is sufficient probability of its truth; that is, whether the facts are shown by competent and satisfactory evidence. Things established by competent and satisfactory evidence are said to be proved. By competent evidence, is meant that which the very-nature of the thing to be proved requires, as the fit and appropriate proof in the particular case, such as the production of a writing, where its contents are the subject of inquiry. By satisfactory evidence, which is sometimes called sufficient evidence, is intended that amount of proof, which ordinarily satisfies an unprejudiced mind, beyond reasonable doubt. The circumstances which will amount to this degree of proof can never be previously defined; the only legal test of which they are susceptible, is their sufficiency to satisfy the mind and conscience of a common man; and so to convince him, that he would venture to act upon that conviction, in matters of the highest concern and importance to his own interest. [A Treatise on Evidence, Vol I, 11th edn. (Boston: Little, Brown, 1888) ch 1., sections 1 and 2. Shorter paragraphs added. (NB: Greenleaf was a founder of the modern Harvard Law School and is regarded as a founding father of the modern Anglophone school of thought on evidence, in large part on the strength of this classic work.)]
KF kairosfocus
Eric Anderson @12
But it is definitional exercise and does not help us get to a substantive answer about the cause itself.
What would qualify as a substantive answer about the cause itself? What could help us to get to such an answer? Dionisio
F/N: I cross-post from the other thread (bearing in mind this longstanding post at UD): Last para of OP in that thread is telling. HeKS speaks:
. . . Coyne’s responses to the Fine-Tuning argument are no more compelling than his attempted rebuttal of the First-Cause argument and they have been answered in depth by others (see, for example, almost any debate with William Lane Craig). Coyne tries to downplay what we do know scientifically about the physical requirements for life in an attempt to weaken the force of the argument, wrongly identifies it as an argument from ignorance when it is actually a positive argument for design based on our universal experience of cause and effect and the principles by which we all consistently infer design, and he finally makes appeal to the possibility of a multiverse, but all of these are merely attempts to block a conclusion of theistic design that can be held with 100% certainty. [ --> no inductive inference is 100% certain, this is selective hyperskepticism, and given local isolation of our cosmos' physics setting up conditions for life, multiverse does not blunt the inference] Even if they were successful (and there’s no good reason to think they are), they would do nothing to change the fact that, based on what we do know at this point in time, theistic design is currently the best explanation of the fine-tuning of the universe for complex intelligent life, and by a large margin at that. [--> inference to best explanation on reliable sign]
This brings out the force of the issue of inferring design on reliable sign. Design is a familiar observed phenomenon, and it is replete with empirically reliable indicia. For instance, functionally specific complex organisation of components and associated information that results in a locally isolated operating point. (Thus, fine tuning, to set a system to that point.) Nor is it a serious objection to say that the designs we observe are done by humans. First, false, beaver dams show design adapted to situation (as has long since been argued here at UD). Second, we are contingent and there is no good reason to infer that we exhaust potential designers. Nor is there a good non-question begging reason to a priori insist that we cannot infer design on signs in circumstances that may point to an incorporeal being or mind. (In fact, there is no good reason to hold that our own minds are only manifestations of processing in the brain's neural networks, not least, such struggles to the point of futility in explaining responsible, rational freedom. Without which, freedom to undertake responsible reasoned argument is utterly undermined.) There are in fact abundant, even notorious, signs of cosmological design, that get stronger and stronger as the decades roll by. Already, at the turn of the 1980's, they were so strong that leading astrophysicist and lifelong agnostic, Sir Fred Hoyle, observed regarding the connecting block Carbon Atom:
>>From 1953 onward, Willy Fowler and I have always been intrigued by the remarkable relation of the 7.65 MeV energy level in the nucleus of 12 C to the 7.12 MeV level in 16 O. If you wanted to produce carbon and oxygen in roughly equal quantities by stellar nucleosynthesis, these are the two levels you would have to fix, and your fixing would have to be just where these levels are actually found to be. Another put-up job? . . . I am inclined to think so. A common sense interpretation of the facts suggests that a super intellect has "monkeyed" with the physics as well as the chemistry and biology, and there are no blind forces worth speaking about in nature. [F. Hoyle, Annual Review of Astronomy and Astrophysics, 20 (1982): 16.]>> also, in the same talk: >>The big problem in biology, as I see it, is to understand the origin of the information carried by the explicit structures of biomolecules. The issue isn't so much the rather crude fact that a protein consists of a chain of amino acids linked together in a certain way, but that the explicit ordering of the amino acids endows the chain with remarkable properties, which other orderings wouldn't give. The case of the enzymes is well known . . . If amino acids were linked at random, there would be a vast number of arrange-ments that would be useless in serving the pur-poses of a living cell. When you consider that a typical enzyme has a chain of perhaps 200 links and that there are 20 possibilities for each link,it's easy to see that the number of useless arrangements is enormous, more than the number of atoms in all the galaxies visible in the largest telescopes. [ --> 20^200 = 1.6 * 10^260] This is for one enzyme, and there are upwards of 2000 of them, mainly serving very different purposes. So how did the situation get to where we find it to be? This is, as I see it, the biological problem - the information problem . . . . I was constantly plagued by the thought that the number of ways in which even a single enzyme could be wrongly constructed was greater than the number of all the atoms in the universe. So try as I would, I couldn't convince myself that even the whole universe would be sufficient to find life by random processes - by what are called the blind forces of nature . . . . By far the simplest way to arrive at the correct sequences of amino acids in the enzymes would be by thought, not by random processes . . . . Now imagine yourself as a superintellect working through possibilities in polymer chemistry. Would you not be astonished that polymers based on the carbon atom turned out in your calculations to have the remarkable properties of the enzymes and other biomolecules? Would you not be bowled over in surprise to find that a living cell was a feasible construct? Would you not say to yourself, in whatever language supercalculating intellects use: Some supercalculating intellect must have designed the properties of the carbon atom, otherwise the chance of my finding such an atom through the blind forces of nature would be utterly minuscule. Of course you would, and if you were a sensible superintellect you would conclude that the carbon atom is a fix. >> and again: >>I do not believe that any physicist who examined the evidence could fail to draw the inference that the laws of nuclear physics have been deliberately designed with regard to the [--> nuclear synthesis] consequences they produce within stars. ["The Universe: Past and Present Reflections." Engineering and Science, November, 1981. pp. 8–12]>>
That, and much more, is what needs to be addressed. KF kairosfocus
On the nature of the First Cause. The First Cause is fundamental, which means that it does not consist of parts. If the First Cause consists of parts, then those parts are fundamental to the first cause — which is incoherent. Therefore, (1) The First Cause is one and indivisible. The First Cause is fundamental, which means that it cannot be encapsulated by a larger context of space, time and laws. If something encapsulates and directs the First Cause, then something is fundamental to the First Cause — which is incoherent. Therefore, (2) The First Cause is outside time and space. (3) A material thing consists of parts and is encapsulated by a context of time, space and laws. Therefore, (4) The First Cause is immaterial. Origenes
rvb8: Still spouting the same old tired atheist drivel. Please up your game so I don't completely waste my time reading your comments. Sebestyen@13: Excellent points. The universe arising on its own accord out of nothing is an "extraordinary claim" without any extraordinary evidence to support it, and certainly no empirical evidence. It is an atheist creation myth disguised as science. Truth Will Set You Free
However the burden of proof is upon the, ‘extraordinary claim’.
All possible explanations for the big bang are extraordinary in their own ways. Of course you can be convinced that one of them is 'more extraordinary' than the other and that you can then shift the burden of proof, but you're really just fooling yourself. Sebestyen Sebestyen
harry @10: Thanks for your comments. I think you are driving in a similar vein as Querius and you make a good case. However, the problem still remains that I described to Querius @11, and it applies to the concepts of time and energy as well. As long as we define each of these parameters (space, time, matter, energy) as existing only within the context of the universe, then obviously whatever cause caused the universe must exist outside of those parameters. But it is definitional exercise and does not help us get to a substantive answer about the cause itself. ----- Again, please note, in the immediate context I am talking about the cause of the universe, not the First Cause. Some people want to conclude those must be identical, but it is important to recognize that the conclusion is based on circular definitions. It might be true that that cause of the universe is the First Cause, but it cannot be deduced. That is the point. Eric Anderson
Querius: Good thoughts. If we define the universe as everything that exists, then, yes, I think your conclusions hold. On the other hand, if we define the universe as everything that we can see, or everything we can observe, or everything that we currently are able to ascertain, then the possibility remains open that something else existed before the universe. Indeed, already in positing a First Cause we are positing that something existed prior to the universe. And that something existed, by definition, outside of the universe. I presume you agree up to this point. Next, we could go on to claim that this something existed outside of "space and time," because space and time only exist in the context of the universe. But that is essentially a circular argument. Specifically, if we define "space" and "time" as coming into existence at the origin of the universe (or as existing only in the context of the universe) then -- by definition -- space and time did not exist prior to the universe. But that is more of a semantic exercise, than a logical or an empirical one. After all, the cause of the universe existed prior to the universe. Everyone agrees on that point. And we can describe this existence however we want: another dimension, another reality, another state of being, whatever. But it existed in some fashion. Saying that the cause of the universe existed outside of space and time, while at the same time defining "space" and "time" as only existing within the universe, may very well be true as a tautological matter, but it is trivially so and doesn't help address the question substantively. Both materialists and theists hold that what came into being as the "universe" is not all that there is. There is some other dimension, some other reality, some other state of being that existed outside of and prior to the universe. In other words, both models must by force of logic conclude that there was something that existed prior to the existence of the universe. And both models struggle mightily to describe how or what or in what state that cause existed. There is little to distinguish one model from the other, or to favor one model over the other, up to this point. Unless we engage in circular definitions, the fact that the universe had a beginning does not allow us to conclude, by force of deduction, that either (a) the cause was the First Cause, or (b) that the First Cause was God. The real question boils down to an inference of which proposed cause is more rational and more consonant with the evidence. Eric Anderson
It is all very simple to me. Events have causes. Time, space, matter and energy came into being with the Big Bang; its cause was some reality that is not confined to space and time since they didn't exist yet. Its cause couldn't have been any kind of natural reality because natural events require matter and energy which didn't exist yet. Natural events take place in time, no matter how infinitesimally small the amount of time that is required for them to occur may be. Something must change for an event to have taken place. Change takes time. No natural event can take place outside of time. So the Big Bang must have been a supernatural event, since natural events only take place in time, which didn't exist yet. If one claims pre-Big Bang matter in some form existed eternally, it still could never have brought forth the Big Bang because time didn't exist yet. That matter would have remained eternally the same without any time in which a change in that matter could take place. This is the reason theories of pre-Big Bang matter are faulty -- time began with the Big Bang before which no natural event could have taken place, including the Big Bang. (Not to mention the dubious nature of the notion that matter existed without any space in which to exist. ;o) The reality that is not confined to space and time that was the cause of the necessarily supernatural event that was the Big Bang, is what is commonly referred to as God. That this reality is a "who" and not a "what" is made evident by the knowledge that was required for the fine-tuning of the Universe for life and the fact that the physical dimension of life is digital informaton-based nanotechnology the functional complexity of which is light years beyond anything modern science knows how to build from scratch. Technology, by definition, is the application of knowledge for a purpose. Only intelligent agents apply knowledge. The primary reality is a "Who," the uncaused first cause, Who unsurprisingly identified Himself as "I AM WHO AM." harry
There's pretty much of a scientific consensus that the universe had a beginning. Since every effect that we study in science has a cause, the universe would also have a cause, otherwise causeless effects would obviate scientific inquiry. The universe includes space-time, dark matter, dark-energy, virtual particles, and likely other stuff and their interactions and deformations. Extrapolating these in reverse leaves us with Nothing. No natural laws, no "empty" space. Simply non-existence. The cause for the universe cannot have been due to quantum fluctuations or the magic of Vast Amounts of Time because Time came into existence with the "Big Bang." First there was Nothing, then there was Everything, and there was a cause. This would of necessity have to have been the First Cause for our universe because you would otherwise need to sneak in Time to have a succession of causes, and Time doesn't exist yet. (Note that postulating a universe-spawning multiverse is logically identical with a giant, trans-dimensional cosmic turtle that lays eggs that become universes.) From Quantum Physics, we know that the fundamental nature of mass-energy is Information (phi). Before it's observed, Information doesn't have any mass or volume, so it's an excellent candidate for something that can be observed.
In the beginning was the Word (Logos in Greek), and the Word was with God, and the Word was God. The same was in the beginning with God. All things were made by him; and without him was not any thing made that was made. In him was life; and the life was the light of men. And the light shineth in darkness; and the darkness comprehended it not. -John 1:1-5, KJV
-Q Querius
Hi Eric, I definitely look forward to reading the rest of your article. Just to offer some clarification, though, I wasn't intending to comment on what you might go on to say in the part of the article I didn't read (you may very well agree with everything I'm about to say). Rather, I was commenting on the lack of validity in Sean Samis' response to the issues addressed in my OP. As you say in your article:
In his comments, samis urges caution in drawing any conclusion from the Big Bang about deity’s existence or involvement.
Any attempt to counter the argument described in my OP by appealing to the possibility of something like a mulitverse or any other higher level spacetime or material state preceding the Big Bang is doomed to failure because the argument says nothing about the Big Bang in particular. The argument is that anything that is extended in time and/or space, which includes both time and space themselves, cannot have existed infinitely into the past. As such, we must conclude that the First Cause was non-extensional, and therefore did not exist within any external dimension of time or space. Sean's appeals to the idea that the First Cause must have just been some other "thing" made of "stuff", just not stuff from our universe, utterly fails to respond to the argument. The logic of the argument eliminates the possibility of the suggestions he puts forth as a matter of logical necessity. He can attack the logic of the argument if he likes, but to simply offer some physical alternative to the non-physical First Cause necessitated by the argument is to be non-responsive to the argument. And then we come to this other direct statement from sean samis:
The proper response to the creation ex nihilo argument is that science does not believe or claim that our universe was created ex nihilo. The argument is a red herring.
The fact that Sean doesn't understand the argument in my OP does not mean that the argument (or any part of the argument) is a red herring. I'm not sure I even know what he thinks "the creation ex nihilo argument" is, since there is no such argument that I'm aware of. As I explained in my OP (and as I'm sure you're aware), "creation ex nihilo" is merely a statement about the method of creation God used by God and just says that he did not create using some pre-existing matter but actually produced the matter out of which physical reality is made. It is not some kind of self-contained argument about God's existence. Furthermore, the issue is not whether or not "science" claims that the universe was created ex nihilo. Scientists are highly prone to saying philosophically stupid things, and so they will appeal to an infinite multiverse, or to some infinitely existing physical state prior to some cosmic inflationary period that was suddenly galvanized into action to explode and create the universe as we now have it. But as I've said above, these require some extensional reality existing infinitely into the past, which is simply not possible. And so the argument educates "science" by informing it that we are forced to conclude that there was some point at which there was either a) absolutely nothing at all or b) some non-extensional reality with the characteristics described in my OP, and logic forces us to conclude that it was (b). There seems to be a certain irony in sean samis talking about red herrings. P.S. Just to be clear, if you sense any antagonism in my tone, it is not remotely directed at you. It is entirely a result of the frivolity of sean samis' arguments in combination with his arrogant demeanor in the comment thread to my OP. If only I had more time on my hands. HeKS
Apparently in the field of Pure Mathematics, there are several dimensions beyond, up, down, sideways, and time. These dimensions are needed to balance equations; understanding this pure mathematics is beyond me; ask BA77! The fact that a human endeavour, mathematics, needs these dimensions to balance equations is not physical proof these dimensions exist, but they are rational proofs, which, I am sorry to say, is several giant steps of proof beyond philosophising. I'm sorry if my tone sounds arrogant. I lay no claim to superior knowledge, indeed most of the contributors here, appear far more knowledgeable, about far more subjects than I do. However the, 'extraordinary claims, demand extraordinary evidence', is justified here, and I make no apologies for that. rvb8
StephenB: Very interesting comment. I like your idea of thinking in terms of forces. If we posit a purely materialistic cause for the universe then, yes, we must be dealing with some kind of natural force. I'm not quite convinced that your argument about a personal agent is water tight, at least insofar as it stands in juxtaposition to a natural cause. After all, (a) the naturalistic explanation is such a vague, unknown, undefined, unexplainable universe-generating force, that it is easy for the materialist to argue that it is some unusual kind of force that could generate a contingent outcome, or perhaps a confluence of forces that together generate contingent outcomes; and (b) in the case of a personal agent, the critic can always point out that the capabilities and power and methods of the agent also remain conveniently undefined and somewhat unexplainable. But I think you may well be on to something as part of the general design-inference line of reasoning for the universe. The design line of reasoning, as I have detailed previously on UD, is more about intentionality, rather than methodology. So I think there is definite merit to your suggestion. I like your idea and will definitely think about it some more . . . Eric Anderson
rvb8: I note your acerbic disdain for anything beyond the measurable capabilities of science. It is somewhat ironic, however, because, as has been rightly pointed out several times by others in these pages, your very ability to reason and engage in the discussion necessarily involves principles that go beyond the measurable capabilities of science. If you are not interested in the origin of the universe or questions about fundamental causes, that is fine, no-one is obligating you to engage in the discussion. But you might bring your tone down a notch.
Anything, and I mean absolutely anything beyond this is speculation, philosophy, religion: Or, to put it into words you might understand, ‘beyond measurability, beyond the capabilities of science.’
I take it you would agree then, that the hypotheses we are regularly treated to in academia and the media about the origin of the universe, extra dimensions, the multiverse, and the like are "speculation, philosophy, religion"? Eric Anderson
Thanks, HeKS. I apologize for the length, but hope you will have a chance to read the rest. In the first part I am just setting up the issue. I am not necessarily addressing Rabbi Rowe's approach or any other specific approach, just analyzing sean samis' comments in and of themselves. If you have a chance to read the rest of my post I trust you will find that I am in agreement about the need for an ultimate First Cause -- independent of whether there is some other spacetime, some intermediate cause for our universe, or otherwise. I also think the possible materialist options for a First Cause are highly suspect in their own right and, frankly, are worse than the proposal of God as the First Cause. Considering God as the First Cause is, I think, supportable from a purely evidentiary and logical standpoint, without a need for scriptural or religious overtones, as valuable as those may be. Once you've had a chance to read the rest, let me know if you feel I am missing anything and I'd be happy to try to clarify. Thanks, Eric Anderson
These discussions on these questions (first causes, and who caused the first cause etc) leave me slack jawed, and feeling more than a little stupid. Until that is, I realise that the people discussing the topic are just as clueless as I am. We know the Universe is expanding, we know this because of the ‘Doppler' effect. We can extrapolate backward and determine a time for the universe's origin, about 13.75 billion years ago; this figure is being refined and, with the launch of the 'James Webb Space Telescope' in October 2018, we will be able to refine this figure down to mere hundreds of thousands of years, if not tens of thousands; amazing! Anything, and I mean absolutely anything beyond this is speculation, philosophy, religion: Or, to put it into words you might understand, 'beyond measurability, beyond the capabilities of science.’ Feel free to speculate, but please don't think you can win, prove, triumph, succeed, consumate, secure, or prove your religion. You cannot prove a first cause, just as I cannot disprove one. However the burden of proof is upon the, 'extraordinary claim'. (Marcello Truzzi.) rvb8
Hi Eric. Interesting thoughts. I would argue that only a personal agent can create a universe ex-nihilo. Here is my rationale: An impersonal force, such as gravity, is too rigid to do anything different from what it has always done, repeatedly and slavishly, directing matter in motion. It lacks the capacity or flexibility to change its behavior because of what it is—a law-like regularity. Thus, it cannot perform a creative act. Put another way, the first cause of the universe (or universes or whatever) must have the personal power to make a decision: Will it leave reality in a state of nothingness or will it create a contingent universe? The operative word here is "contingent." The universe, or initial conditions for a universe, or whatever other Creator substitute is posited, did not have to be. There could easily have been no universe (or multiverse) at all. There could have been nothing. But out of nothing came time, space, and matter. No cosmic law or set of initial conditions can decide to make that happen. Impersonal forces have no choice about how they will affect their environment. They cannot say yes or no to the option of creating or not creating. That choice was made by the personal Creator that brought them into being. StephenB
Hi Eric, I haven't had a chance to read your entire article yet, but having read the first portion there's something I need to point out. You are correct in saying that the cause of the universe does not necessarily need to be the First Cause, as we can logically allow for something like a multiverse. However, the argument presented by Rabbi Rowe was not the Kalam Cosmological Argument but the Argument from Extensionality. The latter works like the Kalam applied to any physical spacetime, like making the second premise of the Kalam ("the universe began to exist") operate on a variable in place of the term "universe", so that it becomes "X began to exist". As such, the Argument from Extensionality is agnostic regarding whether or not our universe is the only physical spacetime and its conclusion regarding the need for a non-extensional First Cause holds with equal force regardless of whether or not our universe exists within any kind of higher level super-space (and the smart bet would seem to be that it does not). I would love to engage Sean Samis on his arguments because I don't think any of them hold up at all, but as I said in comment #1 of my OP, I simply don't have the time to engage right now because of my work schedule. Maybe in a couple weeks I'll have some time to revisit the issue. Take care, HeKS HeKS

Leave a Reply